Você está na página 1de 288

Aritmetica Elemental: Una

introduccion (otra mas)


Sergio Plaza
1
1
Depto. de Matematica, Facultad de Ciencias, Universidad de Santiago de
Chile, Casilla 307Correo 2. Santiago, Chile. email splaza@lauca.usach.cl,
Homepage http://lauca.usach.cl/splaza
i
Version preliminar, en progreso. Se agradecen los comentarios,
correcciones, indicaciones, ...
ii
Contenidos
1 Divisibilidad 1
1.1 Principio de Induccion . . . . . . . . . . . . . . . . . . . . 5
1.2 Problemas . . . . . . . . . . . . . . . . . . . . . . . . . . . 6
1.3 Descenso Innito . . . . . . . . . . . . . . . . . . . . . . . 7
1.4 Divisibilidad en los n umeros enteros . . . . . . . . . . . . 9
1.5 Algoritmo de la Division . . . . . . . . . . . . . . . . . . . 12
1.5.1 Ejercicios . . . . . . . . . . . . . . . . . . . . . . . 14
1.6 Maximo Com un divisor . . . . . . . . . . . . . . . . . . . 19
1.7 Algoritmo para calcular el maximo com un divisor . . . . 25
1.7.1 Metodo de Blankinship . . . . . . . . . . . . . . . 27
1.7.2 Ejercicios . . . . . . . . . . . . . . . . . . . . . . . 30
1.8 N umeros coprimos . . . . . . . . . . . . . . . . . . . . . . 32
1.8.1 Ejercicios . . . . . . . . . . . . . . . . . . . . . . . 34
1.9 Mnimo Com un M ultiplo . . . . . . . . . . . . . . . . . . . 34
1.9.1 Ejercicios . . . . . . . . . . . . . . . . . . . . . . . 36
2 N umeros primos 37
2.0.2 Algoritmo para determinar si un n umero entero
dado es primo . . . . . . . . . . . . . . . . . . . . . 46
iii
iv
2.0.3 Ejercicios . . . . . . . . . . . . . . . . . . . . . . . 56
2.1 Postulado de Bertrand . . . . . . . . . . . . . . . . . . 59
2.2 N umeros de Fermat . . . . . . . . . . . . . . . . . . . . . 63
2.3 N umeros de Mersenne . . . . . . . . . . . . . . . . . . . . 67
2.4 N umeros Triangulares . . . . . . . . . . . . . . . . . . . . 68
2.4.1 Ejercicios . . . . . . . . . . . . . . . . . . . . . . . 72
2.4.2 Algunos comentarios sobre la distribucion de los
n umeros primos. . . . . . . . . . . . . . . . . . . . 80
2.5 Lista de los primeros 1000 n umeros primos . . . . . . . . 83
3 Ecuaciones diofantinas 89
3.1 Problemas . . . . . . . . . . . . . . . . . . . . . . . . . . . 102
3.2 Curiosidades Numericas . . . . . . . . . . . . . . . . . . . 109
4 Congruencias 113
4.1 N umero de pasos en el algoritmo de la division . . . . . . 122
4.2 Peque no Teorema de Fermat . . . . . . . . . . . . . . . . 123
4.3 Test de Primalidad Probabilstico . . . . . . . . . . . . . 124
4.4 Ejemplos . . . . . . . . . . . . . . . . . . . . . . . . . . . 127
4.4.1 Ejercicios . . . . . . . . . . . . . . . . . . . . . . . 138
4.5 Factorizacion . . . . . . . . . . . . . . . . . . . . . . . . . 139
4.6 Teorema de Fermat de los dos cuadrados . . . . . . . . . . 141
4.6.1 Orden de un Elemento . . . . . . . . . . . . . . . . 145
4.7 Races Primitivas . . . . . . . . . . . . . . . . . . . . . . 147
4.8 N umeros de Fermat . . . . . . . . . . . . . . . . . . . . . 149
4.9 Teorema chino de los restos . . . . . . . . . . . . . . . . . 151
4.9.1 Ejercicios . . . . . . . . . . . . . . . . . . . . . . . 156
4.10 Ecuaciones de Fermat . . . . . . . . . . . . . . . . . . . . 157
v
4.11 Ternas Pitagoricas . . . . . . . . . . . . . . . . . . . . . . 160
4.11.1 Ecuacion de Fermat para n = 4 . . . . . . . . . . 163
4.12 Ultimo Teorema de Fermat . . . . . . . . . . . . . . . . . 169
4.13 Teorema de Wilson y Teorema de Fermat . . . . . . . . . 173
4.14 Matematicas e Historia . . . . . . . . . . . . . . . . . . . . 177
4.15 Funcion . . . . . . . . . . . . . . . . . . . . . . . . . . . 178
4.16 La Funcion de Mobius . . . . . . . . . . . . . . . . . . . . 180
4.17 N umeros Perfectos. . . . . . . . . . . . . . . . . . . . . . . 190
4.18 Problemas . . . . . . . . . . . . . . . . . . . . . . . . . . . 197
5 Funciones aritmeticas y sucesiones 203
5.1 Funcion . . . . . . . . . . . . . . . . . . . . . . . . . . 203
5.2 Funcion de Euler . . . . . . . . . . . . . . . . . . . . . 206
5.3 Sucesiones . . . . . . . . . . . . . . . . . . . . . . . . . . . 210
5.4 Problemas . . . . . . . . . . . . . . . . . . . . . . . . . . . 212
6 Representaciones Numericas y Aproximaciones 215
6.1 Representacion decimal . . . . . . . . . . . . . . . . . . . 215
6.2 Representacion en base p , p > 1 . . . . . . . . . . . . . . 223
6.2.1 Representacion Binaria . . . . . . . . . . . . . . . 224
6.2.2 Representacion triadica . . . . . . . . . . . . . . . 225
6.3 Ejemplos . . . . . . . . . . . . . . . . . . . . . . . . . . . 228
6.4 e es irracional . . . . . . . . . . . . . . . . . . . . . . . . . 232
6.5 Conjunto de Cantor . . . . . . . . . . . . . . . . . . . . . 233
6.5.1 Longitud del Conjunto de Cantor . . . . . . . . . 234
6.6 k Volumen . . . . . . . . . . . . . . . . . . . . . . . . . . 236
6.6.1 Triangulo de Sierpinski . . . . . . . . . . . . . . . 238
6.6.2 Triangulo de Sierpinski y expansion en base 2 . . . 240
vi
6.7 Problemas . . . . . . . . . . . . . . . . . . . . . . . . . . . 241
7 Fracciones continuadas 243
7.1 Fracciones Continuadas . . . . . . . . . . . . . . . . . . . 247
7.2 Aplicacion de Gauss . . . . . . . . . . . . . . . . . . . . . 258
7.3 Aproximaciones racionales por fracciones continuadas . . 260
7.4 Fracciones Continuas y Geometra . . . . . . . . . . . . . 266
7.5 Ecuaciones de Pell . . . . . . . . . . . . . . . . . . . . . . 267
8 Problemas resueltos 271
8.1 Problemas Propuestos . . . . . . . . . . . . . . . . . . . . 278
9 Problemas Clasicos No Resueltos 279
Captulo 1
Divisibilidad
En lo que sigue suponemos que el lector esta familiarizado con los
n umeros naturales, N, y los n umeros enteros, Z. Tambien asumire-
mos conocidas las operatorias de suma y producto en estos conjuntos.
Denotaremos muchas veces el producto de dos n umeros a y b por ab ,
en vez de a b . Ademas, suponemos conocidas las propiedades de orden
en los n umeros enteros.
Como toda historia tiene un comienzo y todo libro esta basado en
alg un principio, nosotros nos basaremos en el siguiente principio.
Principio de la Buena Ordenacion (P.B.O.) Todo subconjunto no
vaco S de los n umeros enteros no negativos (enteros mayores o iguales
que cero) posee un menor elemento.
En otras palabras, existe un elemento q en S tal que q s para
todo elemento s en S.
En matematicas los principios no se demuestran, su validez se acepta
sin discusion. Por otra parte, los teoremas deben ser demostrados y
para desarrollar una demostracion se pueden usar deniciones, resulta-
1
2 Teora de N umeros
dos previamente demostrados y principios. En este captulo el unico
principio supuesto ser a el P.B.O.
Como aplicacion del P.B.O. veamos la irracionalidad de

2 .
Ejemplo 1.1 El n umero

2 es irracional.
En efecto, Supongamos contrariamente que

2 es un n umero racional,
digamos

2 = a/b , con a y b enteros, los cuales podemos suponer son


positivos, con b = 0 . Tenemos entonces que a = b

2 es un entero
positivo y podemos denir el conjunto
S = { n

2 : con n y n

2 enteros positivos } .
Es claro que S es no vaco pues b

2 S . Por el P.B.O., S tiene un


menor elemento, digamos, j = k

2 .
Como

21 > 0 , se sigue que j(

21) = j

2j = j

2k

2 =
(j k)

2 es un entero, por ser diferencia de dos enteros. Por otra parte,


como 2 < 2

2 vemos que 2

2 <

2 , y como j = k

2 se tiene que
j

2 = 2k . De esto,
(j k)

2 = j

2 k

2 = 2k k

2 = k(2

2) < k

2 = j ,
de donde (j k)

2 es un entero positivo que pertenece a S y es menor


que j . Esto contradice la eleccion de j como el menor entero positivo
en S .
Comentario. Hay muchas maneras de probar que

2 es irracional.
Una de ella, las mas divulgada y mas directa, es la siguiente. Igual que
antes supongamos que

2 = a/b con a , b enteros sin factores comunes


y b = 0 . Elevando al cuadrado se obtiene 2b
2
= a
2
. Por lo tanto 2
Sergio Plaza 3
divide a a
2
, y se sigue que 2 divide a a , por lo que conclumos que a
debe ser un n umero par, digamos a = 2h, donde h es un n umero entero,
elevando al cuadrado esa igualdad nos queda a
2
= 4h
2
. Reemplazando
este valor en la igualdad 2b
2
= a
2
, se obtiene b
2
= 2h
2
, de lo cual
deducimos que 2 divide a b
2
y como antes vemos que 2 divide a b , lo
cual es una contradiccion con el hecho de que a, b no tenan divisores
comunes.
Ejemplo 1.2 El cuadrado de un n umero entero es de la forma 8n,
8n + 1 u 8n + 4 .
Solucion. Sea z un n umero entero, entonces z puede ser par o impar.
Probaremos que en cada uno de estos casos se tiene lo pedido.
Primer caso z par.
Sea z = 2k con k un n umero entero. Luego z
2
= 4k
2
. Para k
2
hay
dos posibilidades, que sea par o impar.
1. Si k
2
es par entonces k
2
= 2p , donde p es un entero, y por lo
tanto z
2
= 4k
2
= 8p , cumpliendose lo pedido.
2. Si k
2
es impar entonces k
2
= 2q + 1 , donde q es un n umero
entero. Luego z
2
= 4k
2
= 4(2q + 1) = 8q + 4 , obteniendose lo
pedido.
Segundo caso z impar.
En este caso z = 2r + 1 , con r un n umero entero. Desarrollando el
cuadrado del binomio se obtiene que z
2
= 4(r
2
+r) +1 = 4r(r +1) +1 .
Puesto que el producto de dos n umeros enteros consecutivos es siempre
par (excluyendo el caso cero) se concluye que r(r +1) = 2m para alg un
n umero entero m, obteniendose que z
2
= 8m+ 1 .
4 Teora de N umeros
A continuacion probaremos el siguiente resultado, que se deduce del
Principio de la Buena Ordenacion.
Teorema 1.1 (Propiedad Arquimediana)
1
Si a y b son n umeros en-
teros positivos, entonces existe un n umero entero positivo n tal que
na b .
Demostracion. Supongamos que no existe tal entero positivo n, esto
signica que na < b para todo entero positivo n. Notese que a y b
son n umeros enteros positivos dados y jos. Construyamos el conjunto
S siguiente
S = {b na : n Z, n > 0} = {b a, b 2a, . . .}.
1
Arqumedes. Nacio 287 AC en Siracusa, Sicilia, Italia. Fallecio : 212 AC en
Siracusa, Sicilia. Las mayores contribuciones de Arqumedes fueron en geometra.
Sus metodos anticipados de calculo integral 2.000 a nos antes de Newton y Leibniz.
Arqumedes era un nativo de Siracusa, Sicilia y estudio en Alejandra, volviendo
enseguida a su patria. Dedico su genio a la Geometra, Mecanica, Fsica e Ingeniera.
Su geometra es una geometra de la medida. Efect ua cuadraturas de supercies
planas y curvas. Escribio varias obras las cuales se han ordenado seg un la epoca
en que fueron escritas: 1. Esfera y cilindro. 2. Medida del crculo. 3. Gnoides y
esferoides. 4. Espirales. 5. Equilibrio de los planos y sus centros de gravedad. 6.
Cuadratura de la parabola. 7. El arenario. 8. Cuerpos otantes. 9. Los lemas. 10.
El metodo. Arqumedes demostro que la supercie de una esfera es cuatro veces la
de uno de sus crculos m aximos. Calculo areas de zonas esfericas y el volumen de
segmentos de una esfera. Demostro que El area de un casquete esferico es igual a la
supercie de un crculo que tiene por radio la recta que une el centro del casquete con
punto de la circunferencia basal. El problema al cual le atribua gran importancia
era el de demostrar que El volumen de una esfera inscrita en un cilindro es igual a
2/3 del volumen del cilindro. Como postrer homenaje se coloco una esfera inscrita
en un cilindro. Asimismo demostro Arqumedes que la supercie de esta esfera era
tambien los 2/3 de la supercie del cilindro.
Sergio Plaza 5
Como hemos supuesto que b na > 0 para todo entero positivo n,
se tiene que S es un conjunto no vaco de n umeros enteros positivos.
Aplicando el Principio de Buena Ordenacion se obtiene que S posee un
menor elemento. Luego existe m
0
Z tal que bm
0
a bnapara todo
n entero positivo. De esta ultima desigualdad se obtiene que n m
0
para todo n entero positivo, lo cual evidentemente es falso, pues basta
tomar n = m
0
+1 > m
0
. Esta contradiccion se obtuvo por la suposicion
de que na < b para todo entero positivo n. Por lo tanto el teorema ha
sido probado.
1.1 Principio de Induccion
Aunque el Principio de Induccion (P.I.) es llamado as, es de hecho un
teorema, el cual pasamos a enunciar y probar.
Teorema 1.2 (Principio de Induccion) Sea P(n) una propiedad de
enteros no negativos n. Suponga que
(a) P(0) ( o P(1) ) es verdadera, y
(b) P(n + 1) es verdadera si P(n) lo es, para todo n 0 (n 1) .
Entonces P(n) es verdadera para todos los enteros no negativos (po-
sitivos).
Demostracion. Haremos la prueba por contradiccion.
Supongamos que P(n) es falsa para alg un entero no negativo n.
Por el P.B.O., existe un menor entero no negativo n tal que P(n) es
falsa, pero P(m) es verdadera para todo entero no negativo m, con
0 m < n. Como P(0) es verdadera por ( a ), se sigue que n > 0 .
6 Teora de N umeros
Luego, P(n 1) es verdadera, pues 0 n 1 < n. Desde ( b ) se
sigue entonces que P(n) es verdadera, lo cual nos da la contradiccion
deseada.
Teorema 1.3 (Principio de Induccion Completa) Sea P(n) una propiedad
de enteros no negativos n. Supongamos que para cualquier entero no
negativo n, la propiedad P(n) es verdadera cuando P(m) es verdadera
para todo entero no negativo m menor que n. Entonces P(n) es ver-
dadera para todo n 0 .
Demostracion. La armacion es equivalente a su contrapositiva, esto
es, para cualquier n, si P(n) es falsa entonces P(m) es falsa para
alg un entero no negativo m, con m < n. Luego el conjunto de todos
los enteros no negativos para P(n) es falsa no tiene un menor elemento,
y por el P.B.O., este conjunto debe ser vaco.
1.2 Problemas
Problema 1.1 Para todo n 1 , pruebe lo siguiente por induccion
matematica
1
1
2
+
1
2
2
+
1
3
2
+ +
1
n
2
2
1
n
.
Problema 1.2
Problema 1.3
Problema 1.4
Problema 1.5
Sergio Plaza 7
Problema 1.6
Problema 1.7
Problema 1.8
Problema 1.9
Problema 1.10
Problema 1.11
Problema 1.12
Aqu voy, 14 de Mayo de
2009
1.3 Descenso Innito
En el siglo XVII, el gran matematico frances Pierre de Fermat introdujo
un metodo de prueba, el cual llamo el metodo descenso innito, sobre
el que armo uso en todos sus descubrimientos en Teora de N umeros.
Este metodo esta basado en la siguiente armacion obvia: una sucesion
decreciente de n umeros naturales no puede continuar indenidamente.
Con esto Fermat fue capaz de establecer algunos de sus resultados
mas relevantes referentes a propiedades de los n umeros enteros.
Supongamos, como en el caso con induccion matematica, que son
dadas una cantidad innita de armaciones P
1
, P
2
, P
3
, . . . , una para
cada n umero entero positivo n, y que deseamos probar que todas ellas
son verdaderas. Eso puede ser hecho mostrando que cada armacion
8 Teora de N umeros
falsa es precedida por otra armacion falsa. Mas precisamente, supon-
gamos que cualquier armacion dada P
n
es falsa, entonces otra ar-
macion P
m
puede ser encontrada, con m < n, la cual tambien es falsa.
Note que asumimos s olo que m es alg un valor menor que n, y no nece-
sariamente el entero inmediatamente menor. Como n fue arbitrario,
la plaicacion repetida de este argumento nos lleva a otro valor k < m
para el cual P
k
es falsa, y as sucesivamente ad innitum. Pero esto
es imposible, pues existe solo una cantidad nita de n umeros enteros
positivos menores que n, y conclumos por reduccion al absurdo que las
armaciones deben ser verdaderas.
El metodo de descenso innito es simplemente otra forma del Princi-
pio de Induccion, este ultimo bien podra llamarse Metodo de Ascenso
Innito. Ambos metodos de demostracion son equivalentes al P.B.O.
Si podemos demostrar, sin restricciones, que cada armaci on falsa es
precedida por otra armacion falsa, entonce se sigue a priori que es
imposible que la primera armacion sea falsa. La segunda armacion
debe ser tambien verdadera, pues su falsedad debera implicar la de la
primera. De modo analogo, la tercera armacion debe ser precedida por
armaciones verdaderas, y as sucesivamente ad innitum.
Invirtiendo la logica hemos ganado una ventaja. La transicion desde
n a n + 1 , que es la parte importante de una prueba por induccion, es
reemplazada por una transicion desde n a alg un valor menor, pero no
necesariamente n1 , lo cual permite hacer pruebas sin tanta restriccion.
Ademas en una prueba por descenso innito. ??????
Ejemplo 1.3 Prueba de la irracionalidad de

2 usando el metodo de
descenso innito. Por reduccion al absurdo. Supongamos que existe una
fraccion positiva a/b tal que (a/b)
2
= 2 , es decir, a
2
= 2b
2
. De esto se
Sergio Plaza 9
sigue que a
2
es par y en consecuencia a debe ser par. Luego, podemos
escribir a = 2c , y tenemos 4c
2
= a
2
= 2b
2
, de donde b
2
= 2c
2
. Pero
claramente, b es menor que a , y hemos encontrado otra fraccion b/c ,
igual a

2 , pero que tiene denominador menor. Esto es el inicio del


descenso innito, lo cual nos lleva a una contradiccion.
Observacion 1.1 Existe otra prueba interesante en la cual la relacion
la relacion hipotetica a
2
= 2b
2
puede ser escrita en la forma equivalente
(2b a)
2
= 2(a b)
2
, luego (2b a)|(a b) es otra fraccion igual a
a/b y que tiene un denominador menor. La contradiccion se sigue por
el descenso innito o asumiendo de inicio que a/b esta en su forma
simplicada.
1.4 Divisibilidad en los n umeros enteros
Denici on 1.1 Dados dos enteros a, b Z, con a = 0 . Decimos que
a divide a b si existe un n umero entero c tal que b = c a . Si a divide
a b lo denotaremos por el smbolo a|b . Al n umero entero a lo llamamos
un divisor de b . Tambien decimos que b es un m ultiplo de a , o bien
que a es un factor de b . Si a no divide a b usamos la notacion a | / b .
Ejemplo 1.4 Claramente 3|27 , pues existe el n umero 9, tal que 27 =
3 9 , es decir, 3 es un factor de 27 o 3 es un divisor de 27.
Ejemplo 1.5 Para todo n umero entero n, se cumple que n|0 . Esto es
inmediato de la propiedad de que todo n umero entero, incluido el cero,
multiplicado por cero es igual a cero.
A continuacion enumeramos algunas propiedades de la division que
aplicaremos muchas veces en este texto.
10 Teora de N umeros
Teorema 1.4 (Propiedades de la divisibilidad)
i) Si a|b y b|a , entonces a = b .
ii) Si a|b y c|d , entonces ac|bd .
iii) Si a|b y b|c , entonces a|c .
iv) Si a|b , a|c y u, v Z, entonces a|(bu +cv) .
Demostracion. Probaremos solamente la propiedad iv) y dejaremos
las restantes como ejercicios para el lector.
Si a|b y a|c entonces existen n umeros enteros k
1
y k
2
tales que
b = k
1
a y c = k
2
a . Luego bu +cv es de la forma
bu +cv = k
1
au +k
2
av = (k
1
u +k
2
v)a,
lo cual signica que a|(bu +cv) .
Nota 1.1 Una factorizacion algebraica util en Teora de N umeros es la
siguiente.
Para todo par n umeros de a, b y para todo n n umero natural se tiene
que
a
n
b
n
= (a b)(a
n1
+a
n2
b +a
n3
b
2
+ +ab
n2
+b
n1
) .
Esta factorizacion resulta directamente de dividir el polinomio a
n

b
n
por a b . Veamos un ejemplo de la utilidad de esta formula de
factorizacion.
Ejemplo 1.6 Pruebe que para cualquier n umero natural n, 17 divide
a 2
n
3
2n
1 .
Sergio Plaza 11
Solucion. Debemos mostrar que 2
n
3
2n
1 puede ser escrito como un
m ultiplo de 17. Tenemos
2
n
3
2n
1 =
_
2 3
2
_
n
1
= 18
n
1
= (18 1)(18
n1
+ 18
n2
+ + 18 + 1)
= 17 N
donde N = 18
n1
+ 18
n2
+ + 1 . Luego, 17|
_
2
n
3
2n
1
_
.
Problema 1.1 Pruebe que si a, b, c son enteros que no son divisibles
por 5 entonces a
2
+b
2
c
2
tampoco es divisible por cinco.
Solucion. Este problema puede abordarse directamente haciendo una
tabla. La idea es utilizar el hecho que al dividir por 5 cada uno de
ellos se tiene un resto no nulo, es decir, a = 5k
1
+ r
1
, b = 5k
2
+ r
2
,
c = 5k
3
+ r
3
, con los restos 0 < r
i
4 . Por lo tanto, calculando
a
2
+ b
2
c
2
se obtiene la expresion a
2
+ b
2
c
2
= 5l + r
2
1
+ r
2
2
r
2
3
.
Luego basta con examinar la expresion r
2
1
+r
2
2
r
2
3
con la condicion de
que los restos son n umeros 0 < r
i
4 . Como no son muchos, podemos
intentar calcular todos los posibles valores de r
2
1
+r
2
2
r
2
3
.
r
2
1
r
2
1
r
2
1
r
2
1
+r
2
2
r
2
3
1 1 1 1
4 4 4 4
9 9 9 9
16 16 16 16
Experimentando con esta tabla de valores se puede calcular todos los
casos y deducir que la expresion r
2
1
+r
2
2
r
2
3
en la ultima columna nunca
12 Teora de N umeros
es cero ni tampoco un m ultiplo de 5. Con ellos se ha probado lo que se
pide. Una vez logrado esta etapa de experimentacion numerica se puede
tratar de encontrar una demostracion formal que solamente tiene por
objetivo abstraer lo que numericamente se ha corroborado.
1.5 Algoritmo de la Division
Ejemplo 1.7 Vimos que 3|27 , pero 3 no divide a 20. Sin embargo el
n umero 20 se puede escribir como 20 = 3 6 + 2 . Esta propiedad es
generalizable a un par de n umeros enteros arbitrarios como lo indica el
resultado siguiente.
Este algoritmo es debido a Euclides
2
.
Teorema 1.5 Sean a, b Z, con a > 0 . Entonces existen dos n umeros
enteros q y r , unicos, tales que
b = qa +r, con 0 r < a (1.1)
2
Nacio 365 AC en Alejandra, Egipto. Fallecio: Alrededor del 300 AC. Muy poco
se sabe con certeza de su vida. Probablemente, fue llamado a Alejandra en el a no 300
AC. Sin duda que la gran reputacion de Euclides se debe a su famosa obra titulada
Los Elementos Geometricos, conocida simplemente por Los Elementos. Tal es la
importancia de esta obra que se ha usado como texto de estudios por cerca de 2000
a nos, veinte siglos, sin que se le hicieran correcciones de importancia, salvo peque nas
modicaciones. Los Elementos estan constituidos por trece libros. A aquellos se ha
agregado un XIV libro que comprende un trabajo de Hipsicles del siglo II de nuestra
era, y a un un XV libro con un trabajo de menor importancia. Esta obra de Euclides
es el coronamiento de las investigaciones realizadas por los geometras de Atenas, como
as mismo de los anteriores. Euclides no hace sino volver a tomar con mas perfeccion
los ensayos anteriores; hace una seleccion de las proposiciones fundamentales y las
ordena convenientemente desde el punto de vista logico. La forma que emplea es la
logica deductiva.
Sergio Plaza 13
Al n umero r en (1.1) se le llama el resto de la division de b por a.
Demostracion. Primero mostraremos que tales n umeros q, r Z ex-
isten. Para ellos consideramos el conjunto
S = {b as 0 : s Z} .
Es facil ver que S es un subconjunto no vaco de N {0} . Se sigue
del P.B.O. que S tiene un primer elemento.
Sea r el menor elemento de S , y sea q Z tal que b aq = r . Es
claro que r 0 , por lo tanto resta mostrar que r < a . Supongamos
contrariamente que r a . Entonces ba(q+1) = (baq)a = ra 0
de donde b a(q +1) S . Por otra parte es claro que b a(q +1) < r ,
lo que contradice el hecho de que r es el menor elemento de S .
Ahora pasamos a probar la unicidad de los n umeros q, r Z para
los cuales se tiene b = qa + r , con 0 r < a . Supongamos que
b = aq
1
+ r
1
= aq
2
+ r
2
, de esto se sigue que |r
1
r
2
| = a|q
1
q
2
| . Si
q
1
= q
2
, entonces se tiene que a|q
1
q
2
| a , mientras que |r
1
r
2
| < a ,
lo cual es una contradiccion. Por lo tanto q
1
= q
2
y de esto se sigue que
r
1
= r
2
, lo que completa la prueba del teorema.
Este teorema nos dice que si a > 0 , entonces a|b si solo si el resto de
la division de b por a es cero.
Ejemplo 1.8 Sabemos que un a no no bisiesto tiene 365 das. Si el
primero de Enero de un a no fue da Domingo Cuantos Domingos tiene
ese a no? Cuantos Lunes tiene ese a no? Cuantos Martes tiene ese
a no?
14 Teora de N umeros
Solucion. Aqu utilizamos la division por 7, que corresponde a los siete
das de la semana. No es dcil de ver que 365 = 52 7+1 . Esto signica
que el 30 de Diciembre de ese a no es un da Domingo y por lo tanto el
31 de Diciembre es un da Lunes. Por lo tanto podemos responder la
pregunta en forma mas general, hay exactamente 52 das de cada semana
excepto el Lunes que hay uno mas, es decir 53 das Lunes.
1.5.1 Ejercicios
Problema 1.13 Una maquina fabrica alternadamente un perno de pre-
cision cada 14 minutos y una tuerca cada 5 minutos. Si la maquina tra-
baja 24 horas que conviene mas que la maquina comience fabricando
una tuerca o que comience fabricando un perno?
Problema 1.14 Sean a, b enteros positivos tales que ab + 1 divide a
a
2
+b
2
. Pruebe que
a
2
+b
2
ab+1
es el cuadrado de un n umero entero.
Problema 1.15 Sean a, b, c, d enteros. Pruebe que el producto
(a b) (a c) (a d) (b c)(b d)(c d)
es divisible por 12.
Problema 1.16 Sean x, y enteros positivos tales que xy divided a
x
2
+y
2
+ 1 . Pruebe que
x
2
+y
2
+1
xy
= 3
Problema 1.17 (R.K. Guy, R.J. Nowakowki)
a) Encuentre innitos pares de enteros a y b , con 1 < a < b , tal
que ab divide a a
2
+b
2
1 .
b) Sean a, b como en (a) Cuales son los posibles valores de
a
2
+b
2
+1
ab
?
Sergio Plaza 15
Problema 1.18 Pruebe que para cada entero n 2 , se tiene que n
no divide a 2
n
1 .
Problema 1.19 Sea k 2 un entero, y sean n
1
, n
2
, . . . , n
k
1 enteros
con la propiedad
n
2
|2
n
1
, n
3
|2
n
2
1, . . . , n
k
|2
n
k1
1, n
1
|2
n
k
1.
Pruebe que n
1
= n
2
= = n
k
= 1 .
Problema 1.20 Sean m, n enteros positivos tales que mn + 1 es di-
visible por 24. Pruebe que m+n es divisible por 24.
Problema 1.21 Sea f(x) = x
3
+ 17 . Pruebe que para cada entero
n 2 , existe un entero positivo para el cual f(x) es divisible por 3
n
pero no lo es por 3
n+1
.
Problema 1.22 Pruebe que para cada entero no negativo n,
n

k=0
_
2n + 1
2k + 1
_
2
3k
es divisible por 5
Problema 1.23 Sea n un entero positivo. Pruebe que las siguientes
armaciones son equivalentes.
1. n no es divisible por 4.
2. Existen a, b Z tal que a
2
+b
2
+ 1 es divisible por n.
Problema 1.24 Muestre que existen innitos n umeros enteros com-
puestos n tal que 3
n1
2
n1
es divisible por n.
16 Teora de N umeros
Problema 1.25 Sean a, b y c enteros tales que a + b + c divide a
a
2
+b
2
+c
2
. Pruebe que existen innitos enteros positivos n tales que
a +b +c divide a a
n
+b
n
+c
n
.
Problema 1.26 Pruebe que para cada n N se tiene que 7 divide a
3
n
+ n
3
si y solo si 7 divide a 3
n
n
3
+ 1 .
Problema 1.27 Determine todos los enteros n > 1 tal que
2
n
+ 1
n
es un entero.
Problema 1.28 Encuentre todos los triples de enteros positivos (a, c, b) ,
tal que (2
c
1) divide a 2
a
+ 2
b
+ 1 .
Problema 1.29 Encuentre todos los enteros a, b, c con 1 < a < b < c
tales que (a 1)(b 1)(c 1) es un divisor de abc 1 .
Problema 1.30 Determine todos los triples enteros positivos (a, m, n)
tales que a
m
+ 1 divided a (a + 1)
n
.
Problema 1.31 Encuentre todos los n N tal que 2
n1
|n! .
Problema 1.32 Encuentre todos los enteros positivos n tales que 2
n
divide a 3
n
1 .
Problema 1.33 Determine todos los pares (a, b) de enteros para los
cuales a
2
+b
2
+ 3 es divisible por ab .
Problema 1.34 Encuentre el menor entero positivo n tal que 2
1989
|(m
n

1) para todo entero positivo impar m > 1 .


Sergio Plaza 17
Problema 1.35 (Shaulesh Shiradi) Si a, b, c son enteros positivos tales
que
a < a
2
+b
2
abc c .
Pruebe que a
2
+b
2
abc es un cuadrado perfecto.
Nota: Si a
2
+b
2
abc = c entonces
a
2
+b
2
ab+1
= c N.
Problema 1.36 Sean a y b enteros positivos tales que a|b
2
, b
2
|a
3
,
a
3
|b
4
, b
4
|a
5
, . . . Pruebe que a = b .
Problema 1.37 Sean m, n enteros positivos, tales que
A =
(m + 3)
n
+ 1
3m
es un entero. Pruebe que A es impar.
Problema 1.38 Pruebe que si 3 d 2
n+1
, entonces d no divide a
(a
2n
+ 1) para todo entero positivo a .
Problema 1.39 Si n 6 es un entero compuesto, pruebe que n|(n
1)! .
Problema 1.40 Pruebe que existen innitos enteros positivos n tales
que n
2
+ 1 divide a n! .
Problema 1.41 Sea a y b enteros con la propiedad que para cada
entero no negativo (es decir, mayor o igual que cero) n, el n umero
2
n
a +b es el cuadrado de un entero. Pruebe que a = 0 .
18 Teora de N umeros
Problema 1.42 Para cada entero positivo n, pruebe que
(n + 1)(n + 2) (n + 10)
no es un cuadrado perfecto.
Problema 1.43 Determine todos los n umeros N de 3 dgitos, que tiene
la propiedad que N es divisible por 11, y
N
11
es igual a la suma de los
cuadrados de los dgitos de N .
Problema 1.44 Pruebe que 1980
1981
1982
+1982
1980
1980
es divisible por
1981
1981
.
Problema 1.45 Sean m, n enteros positivos.
1. Pruebe que n!|(m+ 1)(m + 2) (m+n).
2. Pruebe que
(3m)!(4n)!
(m!)
3
(n!)
4
es un entero.
Problema 1.46 Sean n
1
, . . . , n
k
enteros positivos. Pruebe que n
1
!n
2
! n
k
!
divide a (n
1
+n
2
+ +n
k
)!
Problema 1.47 Pruebe que 2
n
|(n+1)(n+2) (2n) para cada n N.
Problema 1.48 Encuentre todos los enteros positivos d tales que d|(n
2
+
1) y d|((n + 1)
2
+ 1) para alg un entero n.
Problema 1.49 Encuentre, con demostracion, todos los valores de n
tal que
n
2
+1
n+2
es un entero.
Problema 1.50
Problema 1.51
Sergio Plaza 19
Problema 1.52
Problema 1.53
Problema 1.54
Problema 1.55
Problema 1.56
Problema 1.57
Problema 1.58
1.6 Maximo Com un divisor
El maximo com un divisor entre dos n umeros enteros a , b es un n umero
entero positivo d , que denotamos por d = mcd (a, b) , el cual satisface
las siguientes propiedades.
1. d > 0 , d|a y d|b .
2. Si c Z es tal que c|a y c|b entonces c d .
En otras palabras, el maximo com un divisor mcd(a, b) es un n umero
que divide a a y divide a b , y si existe otro n umero que divide a a
y a b entonces necesariamente tal n umero es menor o igual a d . Por
ejemplo se tiene que mcd (n, 0) = 0 para todo n n umero entero.
Ejemplo 1.9 mcd (45, 78) = 3 , pues los divisores de 45 son: 1, 3, 5, 9,
15, y 45, y los divisores de 78 son: 1, 2, 6, 13, 26, 39, y 78. Observando
los divisores de ambos n umeros vemos que el mayor divisor com un a
ellos es 3.
20 Teora de N umeros
Ejemplo 1.10 de manera analoga al jemplo anterior se tiene que mcd(12, 21) =
3 y mcd(943, 414) = 23 .
Dos preguntas surgen inmediatamente de la denicion de maximo
com un divisor:
1. Existe siempre el maximo com un divisor de un par de n umeros
dados?
2. Es posible construir un algoritmo para calcularlo?
La respuesta a ambas preguntas es armativa. La respuesta a la
primera esta basada en el P.B.O.
Con respecto a la segunda pregunta, una manera de calcular mcd (a, b)
es encontrar todos los divisores de a y de b y elegir el mayor divisor
com un a ambos n umeros. Este metodo resulta ser muy engorroso para
n umeros grandes.
Un metodo mas ecaz es descrito en el septimo libro de la obra de
Euclides, Los Elementos. Este algoritmo para el calculo de mcd (a, b)
se basa en el siguiente resultado.
Lema 1.1 Si b = qa +r , entonces mcd(a, b) = mcd (a, r).
Demostracion. Es inmediata de la denicion de maximo com un di-
visor. Sean k = mcd (a, b)) y = mcd (a, r) los maximos comunes
divisores de a, b y de a, r , respectivamente.
Despejando r se tiene que r = b qa . Como k|a y k|b , se obtiene,
por propiedad iv) de la division, que k tambien divide a r y luego,
por denicion de , k . Ademas es tambien un divisor de b y,
Sergio Plaza 21
por denicion de k , k . Por lo tanto no queda otra alternativa que
k = .
Para calcular mcd(a, b) procedemos de la siguiente manera. Apli-
cando el algoritmo de la division sucesivamente obtenemos la siguiente
cadena de igualdades:
b = q
1
a +r
1
, 0 r
1
< a,
a = q
2
r
1
+r
2
, 0 r
2
< r
1
,
r
1
= q
3
r
2
+r
3
, 0 r
3
< r
2
,
.
.
.
r
n2
= q
n
r
n1
+r
n
, 0 r
n
< r
n1
r
n1
= q
n+1
r
n
+r
n+1
, r
n+1
= 0.
(1.2)
Detenemos el proceso al encontrar el primer resto nulo. Esto siempre
sucede, puesto que el resto de una etapa es estrictamente menos que el
resto de la etapa anterior y r
1
, el primer resto, es estrictamente menor
que a . Aplicando el Lema 1.1 se obtiene que
mcd (a, b) = mcd (a, r
1
) = mcd (r
1
, r
2
) = = mcd (r
n1
, r
n
) = mcd (r
n
, 0) = r
n
.
Ejemplo 1.11 Calculemos mcd (414, 943) . Tenemos
943 = 2 414 + 115
414 = 3 115 + 69
115 = 1 69 + 46
69 = 1 46 + 23
46 = 2 23
22 Teora de N umeros
En este ejemplo, a = 414 , b = 943 y los correspondientes restos son
r
1
= 115 , r
2
= 69 , r
3
= 46 , r
4
= 23 y r
5
= 0 . Luego mcd (414, 943) =
23 .
Ejemplo 1.12 Calculemos mcd (5486, 3254) . Tenemos
5486 = 1 3254 + 2234
3254 = 1 2234 + 1020
2234 = 2 1020 + 194
1020 = 5 194 + 50
194 = 3 50 + 44
50 = 1 44 + 6
44 = 7 6 + 2
6 = 3 2 + 0
Luego, mcd (5486, 3254) = 2 .
Este algoritmo muestra tambien que para un par de n umeros enteros
a y b existen n umeros enteros y tales que
mcd (a, b) = a +b . (1.3)
Esta ecuacion es llamada representacion de Bezout
3
para el maximo
com un divisor.
En el ejemplo 1.11 anterior se obtienen y eliminado consecuti-
vamente los restos r
1
, r
2
, . . . , r
n
, empezando por la pen ultima igualdad
3
Etienne B`ezout, nacio en Nemours, Francia 1730 y murio en Basses-Loges, Francia
1783.
Sergio Plaza 23
23 = 69 1 46
= 2 69 115
= 2 (141 3 115) 115
= 2 414 7 115
= 2 414 7 (943 2 414)
= 16 414 7 943 ,
es decir, = 16 y = 7 .
Este ejemplo muestra como proceder para el caso en que a y b son
dos enteros cualesquiera, es decir, se empieza por la ultima igualdad de
la de cadena en (1.2) y se contin ua imitando lo realizado en el ejemplo.
Ademas de la existencia del maximo com un divisor de dos enteros
a, b Z, tenemos una representacion de B`ezout, esto es, si d = mcd (a, b) ,
entonces existen enteros m, n Z tal que d = ma +nb .
Teorema 1.6 (Lema de B`ezout) Sean a, b Z. Si d = mcd (a, b) ,
entonces d tiene una presentacion de B`ezout para d , en otars palabras,
existen enteros m, n Z tales que d = ma +nb .
Demostracion. Considere el conjunto D = {am + bn : a, b Z} .
Claramente, D es un conjunto no vaco de n umeros enteros. Notemos
que si c D entonces tambien c D. En consecuencia, D contiene
enteros positivos. Sea d el menor entero positivo en D; armamos que
d = mcd(m, n) . Para demostrar esto existen dos cosas a probar.
Armacion 1. d es un divisor com un de m y n.
En efecto, por simetra basta probar que d|m. Escribamos m = rd+s
con 0 s < d ; tenemos que d = am + bn, luego s = rd m =
r(am+bn) m = (ra 1)m +bn D.
24 Teora de N umeros
La minimalidad de d implica que s = 0 , luego d|m.
Armacion 2. Si e es un divisor com un de m y n, entonces e|d .
En efecto, supongamos que e|m y e|n. Como d = am + bn se
concluye que e|d .
La existencia de la representacion de B`ezout para d es inmediata pues
d D.
Note que la clave de la prueba es la existencia de una division con
resto.
El lema de B`ezout puede ser usado para dar una generalizaci on im-
portante de la propiedad: p|ab p|a o p|b de los primos p .
Teorema 1.7 Si m|ab y mcd(m, b) = 1 , entonces m|a .
Demostracion. Escribamos ab = mn, por el lema de B`ezout, existen
x, y Z tal que mx + by = 1 . Multiplicando esta igualdad por a ,
obtenemos a = max +aby = max +mny = m(ax +ny) , esto es, m|a .
Un ejercicio interesante consiste en probar que el maximo com un di-
visor d = mcd (a, b) tambien es determinado por las condiciones
1. d > 0 , d|a y d|b .
2. Si c|a y c|b entonces c|d .
Para probar que estas condiciones (que no usan el concepto de orden)
son equivalentes a la denicion de mcd (a, b) dada anteriormente, se
aplica la propiedad (1.3) del maximo com un divisor.
Sergio Plaza 25
1.7 Algoritmo para calcular el maximo com un
divisor
El algoritmo de Berlekamp para determinar el maximo com un divisor
entre dos enteros, tambien entrega la representacion de B`ezout para el
maximo com un divisor, esto es, d = mcd (a, b) = ax +by .
1. haga a
1
a, a
2
b; x
1
1, x
2
0; y
1
0, y
2
1.
2. haga q
_
a
1
a
2
_
( [x] denota la parte entera de x, es decir, [x] es
el mayor entero menor o igual que x)
3. haga a
3
a
1
qa
2
; x
3
x
1
+qx
2
; y
3
y
1
+qy
2
.
4. haga a
1
a
2
, a
2
a
3
; x
1
x
2
, x
2
x
3
; y
1
y
2
, y
2
y
3
.
5. Si a
2
> 0 vaya a 2.
6. Si ax
1
by
1
> 0 retorne como resultado (d, x, y) = (a
1
, x
1
, y
1
) ,
en otro caso retorne como resultado (d, x, y) = (a
1
x
1
, y
1
) .
Ejemplo 1.13 N umeros de Fibonaci. Una manera de generar n umeros
de Fibonaci
4
es denir la sucesion de n umeros construidos por la recu-
rrencia f
0
= 0 , f
1
= 1 y f
n+1
= f
n
+f
n1
, para n 1 . As podemos
4
Nacio en 1170 probablemente en Pisa, Italia. Fallecio en 1250 probablemente en
Pisa. Leonardo Pisano es mas conocido por su apodo Fibonacci. Jugo un rol muy
importante al revivir las matematicas antiguas y realizo importantes contribuciones
propias. Fibonacci nacio en Italia pero fue educado en Africa del Norte donde su
padre ocupaba un puesto diplomatico. Viajo mucho acompa nando a su padre, as
conocio las enormes ventajas de los sistemas matematicos usados en esos pases. En
su libro Liber Abaci, publicado en el 1202 despues de retornar a Italia, esta basado
en trozos de aritmetica y algebra que Fibonacci acumulo durante sus viajes. En
Liber Abaci Fibonaci introduce el sistema decimal Hind uArabico y usa los n umeros
arabicos dentro de Europa. Un problema en Liber Abaci permite la introduccion de
26 Teora de N umeros
formar los siguientes n umeros 17 primeros n umeros de Fibonaci
f
n
: 0, 1, 1, 2, 3, 5, 8, 13, 21, 34,
55, 89, 144, 233, 377, 610, 987, . . . ,
Notemos la siguiente curiosidad que se extrae examinando directa-
mente los n umeros f
n
. Se tiene que 3|6 y f
3
|f
6
; 4|8 y f
4
|f
8
; 3|9 y
f
3
|f
9
; 5|10 y f
5
|f
10
; 6|12 y f
6
|f
12
, . . . .
Podemos conjeturar que si k | n entonces f
k
|f
n
.
Tratemos de demostrar esta armacion, que en un principio asoma solo
por simple observacion. Para ellos establecemos el siguiente resultado.
Teorema 1.8 Sean f
n
, con n 1 los n umeros de Fibonaci como
fueron generados arriba. Si k|n entonces f
k
|f
n
.
Demostracion. Primero probemos la siguiente identidad
f
s+t
= f
s1
f
t
+f
s
f
t+1
.
Despues, tomando s = n y t = kn, reemplazando en la identidad
nos queda
f
n+kn
= f
(k+1)n
= f
n1
f
kn
+f
n
f
kn+1
.
los n umeros de Fibonacci y la serie de Fibonacci por las cuales es recordado hoy en
da. El Diario Trimestral de Fibonacci es un moderno periodico cientco dedicado al
estudio de las matematicas que llevan estas series. Otro libro de Fibonacci de mayor
importancia es Practicas de Geometra publicado en el a no 1220. Este contiene
una extensa coleccion de problemas de geometra y de trigonometra. Tambien en
su Liber Quadratorum publicado en el a no 1225 aproximo las races c ubicas obte-
niendo una respuesta que en la notacion decimal es correcta en 9 dgitos. En el libro
Mis Practicas de Geometra, publicado el a no 1220 entrega una compilacion de la
geometra al mismo tiempo que introduce algo de trigonometra.
Sergio Plaza 27
De aqu, si f
n
|f
kn
entonces f
n
|f
(k+1)n
. Como trivialmente se tiene
que f
n
|f
n1
y el resultado se sigue por induccion.
1.7.1 Metodo de Blankinship
Este metodo fue descubierto por W.A Blankinship y permite obtener
los enteros s y t en el Lema de Bezout y al mismo tiempo produce
mcd (a, b) .
Dados dos enteros a > b > 0 , comenzamos con el arreglo
_
a 1 0
b 0 1
_
continuamos sumando m ultiples de una la a otra la, eligiendo alter-
nadamente la la, hasta obtener un arreglo de la forma
_
0 x
1
x
2
d y
1
y
2
_
_
d y
1
y
2
0 x
1
x
2
_
Entonces d = mcd (a, b) = y
1
a +y
2
b .
Ejemplo 1.14 Consideremos a = 35 y b = 15 . Tenemos entonces el
arreglo
_
35 1 0
15 0 1
_
28 Teora de N umeros
Notemos que 35 = 15 2 + 5 , luego 35 + 15 (2) = 5 . Ahora,
multiplicando la la 2 por -2 y sumando a la la 1, obtenemos
_
5 1 2
15 0 1
_
Ahora, 15 = 5 3 , de donde 15 + 5(3) = 0 , luego multiplicando la
la 1 por 3 y sumandola a la la 2 obtenemos
_
5 1 2
0 3 1
_
Luego, mcd (35, 15) = 5 y 5 = 1 35 + (2) 15 .
Ejemplo 1.15 Sean a = 1876 y b = 365 . Tenemos el arreglo
_
1876 1 0
365 0 1
_
Como 1876 = 365 5 +51 , multiplicando la la 2 por -5 y sumandola
a la la 1, obtenemos
_
51 1 5
365 0 0
_
Ahora, 365 = 51 7 + 8 , luego multiplicando la la 1 por 7 y
sumandola a la la 2, obtenemos
_
51 1 5
8 7 36
_
Sergio Plaza 29
Como 51 = 8 6 + 3 , multiplicando la la 2 por 6 y sumandola a
la la 1 nos queda
_
3 43 221
8 7 36
_
Ahora, como 8 = 3 2 + 2 , sumamos 2 veces la la 1 a la la 2, y
obtenemos
_
3 43 221
2 93 478
_
Como 3 = 2 1+1 , sumamos 1 veces la la 2 a la la 1 y obtenemos
_
1 136 699
2 93 478
_
Finalmente, como 2 = 1 2 , si multiplicamos la la 1 por 2 y la
sumamos a la la 2, obtenemos
_
1 136 699
0 365 1876
_
Por lo tanto, mcd (1876, 365) = 1 y 1 = 136 1876 + (699) 365 .
Notemos que al momento de producir un cero en la columna 1, no
es necesario calcular los otros elementos de esa la, pues nos interesan
los coecientes en que aparece d = mcd(a, b) , por ejemplo, en el ultimo
ejemplo, no es necesario calcular los coecientes 136(2)93 y (699)
(2) + 478 .
30 Teora de N umeros
Examinemos un poco el algoritmo de Blankinship. Note que esta-
mos viendo lo que ocurre en la primera columna, y esto corresponde
exactamente a lo que hacemos en el algoritmo de Euclides.
Tenemos
_
a 1 0
b 0 1
_
y a = 1 a +a b , b = 0 a + 1 b
Veamos un paso intermedio cualesquiera, en este caso tenemos un
arreglo de la forma
_
a
1
x
1
x
2
b
1
y
1
y
2
_
y a
1
= a
1
a +x
2
b , b
1
= y
1
a +y
2
b , y el resultado se sigue de esto.
1.7.2 Ejercicios
Problema 1.59 Determine el maximo com un divisor de los elementos
del conjunto {n
13
n : n Z} .
Problema 1.60 Sean a , m y n enteros positivos. Pruebe que mcd (a
m

1, a
n
1) = x
mcd (m,n)1
.
Problema 1.61 Suponga que n tiene al menos dos representaciones
distintas como suma de dos cuadrados, es decir, n = 5
2
+t
2
= u
2
+v
2
,
donde s t 0 , u v 0 y s > u. Muestre que mcd (su tv, n) es
un divisor propio de n.
Sergio Plaza 31
Problema 1.62 Pruebe que
mcd (m, n)
n
_
n
m
_
es un entero para todo par de enteros positivos (m, n) con n m 1 .
Problema 1.63 (Teorema de los cuatro n umeros). Sean a, b, c y d
enteros positivos tales que ab = cd . Entonces existen enteros positivos
p , q , r y s tales que
a = pq, b = rs, c = pt y d = su.
Problema 1.64 (Kiran S. Kedlaya) Sean x, y , z enteros positivos.
Pruebe que (xy + 1)(yz + 1)(zx + 1) es un cuadrado perfecto si y solo
si xy + 1 , yz + 1 , zx + 1 son todos cuadrados perfectos.
Problema 1.65 Encuentre los triples (a, b, c) de enteros positivos, tales
que a , b , c estan en progresion aritmetica y ab + 1 , bc + 1 y ca + 1
son cuadrados perfectos.
Problema 1.66 Sean a, b, c, d N. Pruebe cada una de los siguientes
armaciones.
1. Si a|bc y (mcd (a, b) = 1 , entonces a|c .
2. (a; b) = d si y solo si
_
a
d
;
b
d
_
= 1 .
3. mcd (ac, bc) = d = c mcd (a, b)
4. mcd (a, bc) = mcd (a, mcd (a, b)c)
32 Teora de N umeros
5. mcd (a
2
, b
2
) = (mcd (a, b))
2
.
Problema 1.67 Todos los dgitos de un n umero N con 1998 dgitos
son 1. Encuentre el maximo com un divisor de N y 1111.
Problema 1.68
Problema 1.69
Problema 1.70
Problema 1.71
Problema 1.72
1.8 N umeros coprimos
Denicion 1.2 Diremos que los n umeros enteros no nulos a y b son
coprimos (relativamente primos) si no poseen divisores comunes dife-
rentes de 1. En otras palabras, a y b son coprimos si mcd (a, b) = 1 .
Ejemplo 1.16 Los n umeros 18 y 35 son coprimos, mientras que 18 y
15 no lo son, puesto que 3 es un divisor com un.
En particular, por la representacion de B`ezout (1.3), si a y b son
coprimos entonces existen dos n umeros enteros y tales que
a +b = 1 .
Ahora probaremos un resultado frecuentemente empleado, y que apli-
caremos en la proxima seccion.
Sergio Plaza 33
Lema 1.2 Si d = mcd(a, b) es el maximo com un divisor de a y b , en-
tonces existen enteros r , s tales que a = rd y b = sd , con mcd (r, s) =
1.
Demostracion. Por denicion de maximo com un divisor, se tiene que
d es un divisor positivo de a y b . Luego es posible encontrar un par
de n umeros r, s tal que a = rd y b = sd . Si r , s no son coprimos,
entonces tienen un divisor com un t > 1 , de donde td es un divisor
com un de a y de b . Ahora, como td > d , se obtiene una contradiccion
con la hipotesis de que d es el maximo com un divisor de a y de b .
Se puede enumerar una gran cantidad de propiedades para este tipo
de n umeros. Mencionamos solamente algunas y dejamos como ejercicios
sus demostraciones.
Teorema 1.9 Sean a, b dos n umeros coprimos, y c Z.
1. Si c|a y d|b , entonces mcd(c, d) = 1 .
2. Si a|bc , entonces a|c .
3. Si a|c y b|c , entonces ab|c .
4. Si mcd (a, c) = 1 , entonces mcd (a, bc) = 1 .
Ejemplo 1.17 Las expresiones 2x + 3y y 9x + 5y son divisibles por
17 para los mismos valores enteros x e y .
Solucion. Llamemos w = 2x+3y y z = 9x+5y a las expresiones dadas.
Se tiene que 4w + z = 17(x + y) . Luego 17 divide a 4w + z . Ademas,
si 17|4w , entonces 17|w, puesto que mcd (17, 4) = 1 . Finalmente, es
claro de la igualdad anterior que 17|w si y solamente si 17|z .
34 Teora de N umeros
Ejemplo 1.18 Sean a y b n umeros naturales tales que su maximo
com un divisor es d . Entonces hay exactamente d n umeros del conjunto
S = {a, 2a, 3a, . . ., (b 1)a, ba} que son divisibles por b .
Solucion. Sea d = mcd (a, b) , entonces d|a y d|b , es decir, existen
enteros r y s tales que a = rd y b = sd , con mcd (r, s) = 1 . Luego el
conjunto S puede describirse como sigue
S = {rd, 2rd, 3rd, . . . , (b 1)rd, brd} = {krd : k = 1, 2, . . . , b}.
Al dividir cada n umero del conjunto S por b = sd , se obtiene resto
cero si y solamente si s divide a k , pues mcd(r, s) = 1 . Como b = sd ,
esto sucede exactamente d veces.
1.8.1 Ejercicios
Problema 1.73 Pruebe que entre cualesquiera diez enteros positivos y
consecutivos al menos uno es coprimo con el producto de los otros.
1.9 Mnimo Com un M ultiplo
Introducimos ahora el concepto de mnimo com un m ultiplo para dos en-
teros a y b . Este n umero es el menor entero positivo que es un m ultiplo
com un de a y b , y lo denotamos por mcm(a, b) = [a, b] . Por ejemplo,
[9, 12] = 36 , pues los m ultiplos de 9 son: 9, 18, 27, 36, 45, 54, . . . y los
m ultiplos de 12 son: 12, , 24, 36, 48, 60, . . . , y el menor m ultiplo com un
de ambos n umeros es 36. De manera analoga, se ve que [25, 9] = 225 y
[49, 14] = 98 . Cuando calculamos
a
b
+
c
d
primero buscamos el mnimo
com un m ultiplo entre b y d , enseguida procedemos a realizar la suma.
Sergio Plaza 35
Una forma relativamente facil para calcular el mnimo com un m ultiplo
para a y b es dada por la formula
mcm(a, b) =
|ab|
mcd(a, b)
.
En los ejemplos anteriores, tenemos [9, 12] =
|912|
mcd (9,12)
=
108
3
= 36 ,
[25, 9] =
|259|
mcd (25,9)
=
225
1
= 225 y [49, 14] =
|4914|
mcd (49,14)
=
686
7
=
98 .
Como una aplicacion de ambos conceptos, veamos el siguiente proble-
ma.
Ejemplo 1.19 El n umero 739ABC es divisible por 7, 8 y 9 Que val-
ores pueden tomar A, B y C ?
Solucion. Recuerde que dos n umeros naturales a y b son coprimos si
su maximo divisor com un es 1. Si a , b y c son coprimos dos a dos,
es decir, cada par distintos de ellos son coprimos, entonces su mnimo
m ultiplo com un, mcm(a, b, c), es igual a su producto abc .
Ahora como 739ABC es divisible por 7, 8 y 9, debemos elegir 739ABC
de modo sea un m ultiplo de 7, 8 y 9, es decir, debemos elegir 739ABC
que es divisible por mcm(7, 8, 9) = 504 . Ahora 739000 deja un resto
igual a 136 cuando es dividido por 504. Luego, los n umeros 739ABC
que andamos buscando deben ser de la forma 739136+k 504 , donde
k es un entero. Vemos que k solo puede tomar los valores 1 o 2. Si
k = 1 , obtenemos el n umero 739368 que es una solucion para A = 3 ,
B = 6 , C = 8 , por otra parte si k = 2 obtenemos el n umero 739872
que es otra solucion con A = 8 , B = 7 y C = 2 .
36 Teora de N umeros
1.9.1 Ejercicios
Problema 1.74 Para todo a, b Z, se tiene mcd (a, b) mcm(a, b) =
ab .
Problema 1.75 Para cada entero a , demuestre lo siguiente
1. mcd (2a + 1, 9a + 4) = 1 ;
2. mcd (5a + 2, 7a + 3) = 1 ;
3. Si a es impar, entonces mcd (3a, a3 + 2) = 1 .
Problema 1.76 Pruebe que n divided a 1
5
+3
5
+5
5
+ +(2n1)
5
.
Problema 1.77
Problema 1.78
Problema 1.79
Problema 1.80
Problema 1.81
Problema 1.82
Captulo 2
N umeros primos
Diremos que un n umero entero p > 1 es un n umero primo (o simple-
mente primo) si sus unicos divisores son 1 , 1 , p y p . Si un n umero
a > 1 no es primo diremos que a es un n umero compuesto.
Teorema 2.1 Un entero 2 es compuesto si y solo si existen enteros
a y b tales que n = a b , y 1 < a < n, 1 < b < n.
Demostracion. Sea n 2 un entero. Si n es compuesto, entonces
existe un entero positivo a tal que a = 1 , a = n y a|n. Esto signica
que n = a b para alg un entero b . Como a y n son positivos, se sigue
que b tambien es positivo. Luego 0 1 y b 1 . Ademas, tambien
se tiene que a n y b n. Como a = 1 y a = n, se tiene que
1 < a < n. Si b = 1 , entonces a = n, lo cual es imposible, por lo
tanto b = 1 . Si b = n, entonces a = 1 , lo cual es imposible. Luego,
1 < b < n.
La recproca es obvia.
Trabajaremos solo con los primos positivos. Los primeros primos son
2, 3, 5, 7, . . . y los primeros compuestos son 4, 6, 8, 9, . . . . Notese que el
37
38 Teora de N umeros
n umero 1 no es primo ni compuesto.
Teorema 2.2 Si n > 1 es un entero compuesto, entonces n tiene un
factor primo p , con p

n.
Demostracion. Sea n > 1 un entero compuesto. Entonces n = a b ,
con 1 < a < n y 1 < b < n. Armamos que uno de ellos a o b es
menor o igual que

n. Si no, es decir, a >

n y b >

n, entonces
n = a b >

n

n = n, esto es, n > n, lo que es una contradiccion.


Por lo anterior, tenemos a

n o b

n. Supongamos que a

n.
Como a > 1 , del lema anterior, existe un primo p tal que p|a . Como
a|n, se sigue que p|n y como p|a , se tiene tambien que p a

n.
Teorema 2.3 Si n > 1 , entonces existe un primo p tal que p|n.
Demostracion. Supongamos por el contrario que existe un entero n
0
>
1 que no posee divisores primos. Sea A = {n N; n > 1 tal que n no
posee divisores primos } . Por lo que estamos asumiendo A = , pues
n
0
A. Por el P.B.O. existe un menor elemento para A, llamemos este
m, es decir, m A y m n para todo n A.
Ahora m > 1 y no tiene divisores primos. Luego m no puede ser
primo, de donde m es compuesto, por lo tanto por el lema anterior,
podemos escribir
m = ab, 1 < a < m, 1 < b < m
Como 1 < a < m, se sigue que a no permite a A. Por lo tanto a
tiene un divisor primo p . Ahora, como p|a y a|m se sigue que p|m.
Esto no tiene divisores primos. Esta contradiccion prueba el resultado.
Sergio Plaza 39
Examinaremos una propiedad elemental de los primos que es de mucha
utilidad.
Teorema 2.4 Si p es un primo y p|ab , entonces p|a o p|b , es decir,
si un n umero primo divide al producto de dos n umeros, entonces necesa-
riamente el debe dividir a uno de ellos (o a ambos).
Demostracion. Si p|a no hay nada mas que hacer. Si p no divide a a
entonces mcd (p, a) = 1 , puesto que p no posee ning un divisor aparte
de 1 y p , es decir, a y p son coprimos. Aplicando la propiedad 2) de
la coprimalidad se obtiene que necesariamente p|b .
Ahora estamos en condiciones de describir el resultado quizas mas
importante de la teora de n umeros:
Teorema 2.5 (Teorema Fundamental de la Aritmetica [T.F.A.]) Sea
n > 1 un n umero entero. Entonces existen primos p
1
, p
2
, . . . , p
r
, con
p
1
< p
2
< < p
r
, y n umeros enteros positivos
1
,
2
, . . . ,
r
, tales
que
n = p

1
1
p

2
2
p
r
r
. (2.1)
Ademas, esta presentacion 2.1, llamada descomposicion primaria de
n, es unica.
Demostracion. La demostracion del T.F.A esta basada en el P.B.O.
Daremos un esbozo de ella.
Como n > 1 , entonces hay solamente dos posibilidades para n:
1. es primo., en este caso no hay nada mas que hacer, es decir, p
1
=
n,
1
= 1 y r = 1 .
2. n es compuesto.
40 Teora de N umeros
Caso 1) En este caso no hay nada mas que hacer, es decir, p
1
= n,

1
= 1 y r = 1 .
Caso 2) En este caso se tenemos que n posee divisores distintos de 1
y n. Llamamos p
1
al menor de los divisores de n, el cual existe por el
P.B.O., puesto que el conjunto S denido por S = {c : c|n, c > 1} es
un conjunto no vaco de enteros positivos.
Armacion. p
1
es primo.
Si p
1
no es primo, entonces p
1
posee a un divisor c > 1 , con c < p
1
.
Como c|p
1
y p
1
|n, se sigue que c|n (propiedad iii) de la division), lo cual
contradice la minimalidad de p
1
, quedando demostrada la armacion.
Ahora bien, por denicion de divisor existe un n umero entero n
1
> 1
tal que n = n
1
p
1
. Para n
1
tenemos dos posibilidades, estas on las
ya descritas en 1. y 2. arriba. Aplicando el argumento anterior a n
1
,
se obtenemos que existe p
2
tal que p
2
|n
2
, con p
2
el menor divisor de
n
2
. Imitando lo hecho para p
1
obtenemos que p
2
tambien es primo.
Luego n = n
2
p
2
p
1
, con n
2
< n
1
< n. Continuando de esta forma,
obtenemos que para alg un r , el n umero n
r
es primo, pues ordenando los
r
j
obtenidos en cada etapa tenemos que n
r
< n
r1
< < n
2
< n
1
< n
y n
r
no puede ser menor que 1. Esto completa la prueba del teorema.
Ejemplo 2.1 Claramente, si no se impone la condicion p
1
< p
2
< <
p
r
, tal representacion no es unica. Por ejemplo, el n umero 12 posee las
descomposiciones siguientes 12 = 2
2
3 = 2 3 2 = 3 2
2
, pero solo la
primera de estas cumple la condicion p
1
= 2 < p
2
= 3 .
Para los n umeros 112 y 165, sus descomposiciones primarias son 112 =
2
4
7 y 165 = 3 5 11 .
Sergio Plaza 41
Varias preguntas se pueden plantear para los primos. Algunas de ellas
son
1. Es la cantidad de primos innita?
2. Existe alg un algoritmo para encontrar todos los n umeros primos?
La respuesta a la primera pregunta puede ser encontrada en el libro
IX de los Elementos de Euclides. El argumento descrito all es de una
simplicidad asombrosa.
Euclides, en Libro IX, proposicion 20 (de la version en ingles) formulo
su teorema sobre la innitud de los primos de la forma siguiente.
los n umeros primos son mas que cualquier multitud de n umeros pri-
mos asignados.
En otras palabras, dada cualquier lista de n umeros primos existe un
n umero primo que no esta en dicha lista. Note que Euclides cuidadosa-
mente evita la nocion de conjuntos innitos, de hecho problemas con
innitos, recuerde la paradoja de Zenon, por ejemplo, llevo a los griegos
solo a admitir cantidades nitas en matematica. Por ejemplo, lineas en
geometra no eran innitas, pero podan ser prolongadas tanto como se
deseara. Los conjuntos innitos ganaron su propio lugar en matematica
debido a trabajos de G. Cantor
1
Teorema 2.6 (Euclides) La cantidad de n umeros primos es innita.
Demostracion. Para demostrar este teorema Eulides supuso que hay
una cantidad nita de n umeros primos y logro mostrar que existe otro
1
Georg Cantor, nacio en San Petersburg en 1845 y Murio en Halle en 1918. Se debe
a el entre otras, la nocion de numerabilidad, los racionales son numerables mientras
que los irracionales no lo son.
42 Teora de N umeros
primo mas aparte de los ya considerados. Examinamos esta construccion
detalladamente.
Supongamos que p
1
, p
2
, . . . , p
n
son todos los primos posibles. De-
namos el n umero entero q como q = p
1
p
2
p
n
+1 . Puesto que q > p
i
para todo i = 1, 2, . . . , n, se tiene que q no es primo, es decir, debe ser
compuesto. Por el T.F.A. se obtiene que q posee un divisor primo p,
el cual debe ser uno de los n umeros p
1
, p
2
, . . . , p
n
. Por otra parte, es
claro que p|(p
1
p
2
p
n
) . Luego, por la propiedad iv) de la division,
p debe dividir al n umero q p
1
p
2
p
n
= 1 y, por lo tanto, p = 1 ,
lo cual contradice la denicion de primo. En resumen, se ha probado la
innitud de los n umeros primos.
Es interesante notar que si comenzamos con p = 2 , el primer primo,
la construccion anterior genera los siguiente n umeros:
q
1
= 2 + 1 = 3
q
2
= 2 3 + 1 = 7
q
3
= 2 3 5 + 1 = 31
q
4
= 2 3 5 7 + 1 = 211
q
5
= 2 3 5 7 11 + 1 = 2311 ,
los cuales son primos. Sin embargo, q
6
= 2 3 5 7 11 13+1 = 30031 =
59 509 , q
7
= 510511 = 19 97 277 , q
8
= 9699691 = 67 104473
no lo son. Uno de los problemas no resueltos en teora de n umeros es
determinar si existe una cantidad innita de primos que se pueda generar
con el algoritmo anterior. Se conocen muy pocas maneras, y en general
difciles de obtener, de generar primos. En resumen, la respuesta a la
segunda pregunta planteada no se conoce y es probable que tal algoritmo
no exista.
Sergio Plaza 43
Una variacion del argumento de Euclides es el siguiente
n
1
= 2
n
2
= n
1
+ 1 = 3
n
3
= n
2
n
1
+ 1 = 7
n
4
= n
3
n
2
n
1
+ 1 = 43
.
.
.
n
k
= n
k1
n
k2
n
1
+ 1
Problema 2.1 . Probar que dos n umeros cualesquiera seleccionados
del algoritmo anterior son coprimos, es decir, mcd(n
i
, n
j
) = 1 para
i = j .
La construccion anterior produce innitos n umeros n
k
coprimos entre
si y, ya que ellos no poseen ning un factor primo com un, obtenemos otra
demostracion de que hay una cantidad innita de primos.
A continuacion daremos otras pruebas de la innitud de los n umeros
primo.
Prueba de Hermite. Para n = 1, 2, . . . , sea q
n
el menor primo divisor
de n! + 1 . Se tiene que q
n
> n, luego existen innitos primos.
Otra prueba. Se denen los n umeros de Fermat como sigue: F
n
=
2
2
n
+ 1 es el nesimo n umero de Fermat. Armamos que si m < n,
entonces F
m
|(F
n
2) . En efecto, F
n
= 2
2
n
1 aqui hay algo mal es di-
visible por 2
2
m+1
1 = (2
2
m
1)F
m
. Luego mcd (F
m
, F
n
)|(F
n
, F
n
2) =
2 , pero los n umeros de Fermat son impares, luego ellos son coprimos.
Prueba de Stieltjes, 1890. Supongamos que existe solo una cantidad
nita de primos, denotemos por D su producto. Sea D = m n una
factorizacion de D con m, n N. Entonces para cualquier primo p ,
44 Teora de N umeros
tenemos que p|m o p|n, pero no a ambos; luego p no divide a m+n,
y por lo tanto m+n no puede tener divisores primos. Contradiccion.
Observacion. Si en la demostracion anterior tomamos la factorizacion
D = D 1 , entonces tenemos la prueba de Euclides de la innitud de
primos.
Prueba de Euler 1849. Esta prueba usa la funcion de Euler que
estudiaremos en un captulo mas adelante. Supongamos que existe solo
una cantidad nita de primos, y sea D su producto. Entonces
(D) =

p primo
(p 1) 2 4 > 2,
luego, debe haber un entero a en {2, . . . , D} coprimo con D. Este
entero a no puede tener ning un divisor primo, y por lo tanto debe ser
igual a 1 , lo que contradice el hecho que a 2 .
Observacion. Si tomamos a = D1 , basicamente tenemos la prueba
de Euclides con N 1 en vez de la original con D N + 1 .
Como ya hemos visto, los primeros primos son 2, 3, 5, 7, 11, 13, 17, . . . .
Denotemos por p
1
al primer primo, p
2
al segundo primo, p
3
al tercero
y as sucesivamente. En otras palabras, p
1
= 2 , p
2
= 3 , p
3
= 5 , y
p
n
sera el nesimo primo. Luego, por notacion, p
n
< p
n+1
. El mayor
n umero primo conocido hasta 1979 era 2
21.701
1 .
Teorema 2.7 Si p
n
denota el nesimo primo, entonces p
n
< 2
2
n
.
Este resultado prueba que al menos hay ( n + 1 ) primos menores que
2
2
n
.
Demostracion. Colocarla
Sergio Plaza 45
La demostracion es una clara y sencilla aplicacion del Principio de
Induccion Matematica y la dejamos para el apendice.
Teorema 2.8 Existen innitos primos de la forma 4n 1 .
Demostracion. Supongamos que existen solo una cantidad nita de
primos de la forma 4n 1 , y sean estos p
1
= 3 , p
2
= 7, . . . , p
n
, y
formemos el n umero N = 4p
1
p
2
p
n
1 . Este n umero no puede ser
primo, pues N = p
j
y por lo que asumimos los p
j
, para j = 1, 2, . . . , n
son todos los primos de la forma 4n1 . Luego N debe tener un divisor
primo. Si todos ellos son de la forma 4n+1 , entonces N debe tener tal
forma, pues (4n + 1)(4m+ 1) = 4(4mn +n +m) + 1 , pero N no tiene
esa forma, as al menos uno de los divisores primos de N , digamos p ,
dedbe tener la forma 4n 1 . Como p|4p
1
p
2
p
n
1 , este primo debe
ser diferente de los primos p
j
, esto contradice el hecho que existe solo
una cantidad nita de primos de la forma 4n 1 .
Observacion. La misma idea no funciona para primos de la forma
4n+1, p
1
= 5, p
2
= 13, p
3
= 17, . . . , pues n umeros de la forma 4n+1 ,
pues por ejemplo, 4 5 + 1 = 21 = 3 7 .
Teorema 2.9 Hay innitos n umeros primos de la forma 4n+3.
Demostracion. La prueba de este resultado es una inmediata variante
de la demostracion del resultado Completarla.
Entre las conjeturas acerca de la distribucion de los n umeros primos
que a un permanecen sin respuesta mencionamos
Problema 2.1 Para cada n N, hay siempre un n umero primo entre
n y 2n, para n > 1 ?
46 Teora de N umeros
Problema 2.2 Hay innitos primos de la forma n
2
+ 1 ?
Verquemos esta conjetura para unos pocos n umeros
n n
2
+ 1
1 2 primo
2 5 primo
3 10 compuesto
4 17 primo
5 26 compuesto
6 37 primo
.
.
.
.
.
.
.
.
.
Problema 2.3 Hay siempre un primo entre n
2
y (n + 1)
2
para todo
n 1 ? Una vericaci on con algunos pocos n umeros es dada a seguir
n n
2
(n + 1)
2
primos entre n
2
y (n + 1)
2
1 1 4 2 y 3
2 4 9 5 y 7
3 9 16 11 y 13
4 16 25 17 , 19 , y 23
.
.
.
.
.
.
.
.
.
.
.
.
2.0.2 Algoritmo para determinar si un n umero entero
dado es primo
1. input N; if N=1 print N es una unidad y termine;
2. if 2 | N print p=2 y termine;
3. put q:=3;
4. if q | N print p=q y termine;
Sergio Plaza 47
5. put q:=q+2; if q >

N print p=N y termine;


de otro modo go to paso 3.
Primero que nada notemos que este es un algoritmo pues termina. No
puede formarse un ciclo con el paso 3 para siempre, ya que eventualmente
q va a ser mayor que

N , y el programa termina con el paso 4.


El algoritmo determina el menor factor primo de un n umero dado N ,
en la practica, la condicion q >

N es reemplazada por otra como


la siguiente q >

N + 0.1 esto para evitar el problema de error de


redondeo que pueden ocurrir, por ejemplo cuando N = p
2
. Si N no es
divisible por ning un entero entre 2 y N1 entonces N es primo. En el
peor de los casos, es decir, cuando N es primo, este metodo requiere de

N
2
divisiones, lo cual es bastante mejor que las N 1 divisiones que
se requieren para determinar el mismo resultado cuando dividimos por
2, 3, 4, . . . , N 1 . Un problema interesante se encontrar un algoritmo
que sea altamente eciente para determinar si un n umero entero es primo
o no. Por ejemplo, podemos vericar que nuestro algoritmo no es nada
eciente cuando queremos determinar si el n umero 2
6.972.593
1 es primo
o no. Este n umero es enorme y era el mayor primo conocido hasta el a no
2000. A estas alturas probablemente, ya se conozca uno mucho mayor.
Una pregunta natural es la siguiente Como podemos decidir cuando
un n umero natural dado n, posiblemente muy grande, es un n umero
primo? Si n es compuesto entonces n = ab para algunos n umeros
naturales a y b con ninguno de ellos iguales a 1; y bien a = b =

n
o uno de ellos a o b es menor que

n. Luego, para mostrar que n
es un n umero primo necesitamos mostrar que no tiene divisores primos
menores o iguales que

n.
48 Teora de N umeros
Ejemplo 2.2 97 es un n umero primo. Para verlo primero calcule-
mos

97 = 9, 8488578 . . . , y basta entonces considerar solo los primos


menores o iguales que 9, los cuales son 2, 3, 5, 7, y como ninguno de
ellos divide a divide a 97, concluimos que 97 es primo.
Ejemplo 2.3 Los primos menores que 100 son
2, 3, 5, 7, 11, 13, 17, 19, 23, 29, 31, 37, 41, 43, 47, 53, 59, 61, 67, 71, 73,
79, 83, 89, 97,
esto es facil de vericar, pues solo necesitamos chequear la divisibilidad
de ellos por los primos 2, 3, 5 y 7.
Ejemplo 2.4 Sea n un n umero entero mayor que 1. Entonces 4
n
+n
4
no es primo.
En efecto, si n es par la expresion z = 4
n
+ n
4
es divisible por 2,
luego no es primo. Supongamos que n = 2k+1 , con k un entero k 1 .
Entonces z = (2
2k+1
)
2
+ (n
2
)
2
. Sumando 2n
2
2
2k+1
se completa el
cuadrado del binomio, es decir,
z + 2n
2
2
2k+1
= (2
2k+1
+n
2
)
2
.
Despejando z en esta igualdad se obtiene que
z =
_
2
2k+1
+n
2
_
2
2
2(k+1)
n
2
=
_
2
2k+1
+n
2
_
2

_
2
k+1
n
_
2
=
_
2
2k+1
+n
2
n
2
2
k+1
n
_

_
2
2k+1
+n
2
+ 2
k+1
n
_
.
Para nalizar basta con probar que las expresiones entre parentesis
de la derecha en la ultima igualdad son mayores que 1. Claramente
la segunda expresion es mayor que uno. Examinaremos la primera de
Sergio Plaza 49
ellas. Supongamos que 2
2k+1
+ n
2
2
k+1
n = 1 . Entonces se obtiene
que
_
n 2
k
_
2
+ 2
2k
= 1 , lo cual se cumple solamente si k = 0 , lo cual,
a su vez, no esta permitido por hip otesis.
Ejemplo 2.5 Sea n = p
e
1
1
p
e
2
2
p
e
k
k
la descomposicion primaria de un
n umero natural n, entonces el n umero de divisores de n, incluyendo a
1 y n mismo, es igual a (e
1
+ 1)(e
2
+ 1) (e
k
+ 1) .
En efecto, notemos que cada divisor de n es de la forma p
f
1
1
p
f
2
2
p
f
k
k
,
donde todos los f
1
, f
2
, . . . , f
k
son n umeros naturales y satisfacen
0 f
1
e
1
0 f
2
e
2
.
.
.
0 f
k
e
k
.
En particular, 1 = p
0
1
p
0
1
p
0
k
con f
1
= 0, f
2
= 0, . . . , f
k
= 0 , y
n es el divisor de n con f
1
= e
1
, f
2
= e
2
, . . . , f
k
= e
k
. Luego el
n umero de divisores de n es igual al n umero de elecciones de f
1
multi-
plicado por el n umero de elecciones de f
2
... multiplicado por el n umero
de elecciones de f
k
. Ahora el conjunto de elecciones posibles para f
1
es {0, 1, 2, . . . , e
1
} , y existen e
1
+ 1 posibilidades para elegir a f
1
, el
conjunto de elecciones posibles para f
2
es {0, 1, 2, . . . , e
2
} , y existen
e
2
+ 1 posibilidades para elegir a f
2
, y as sucesivamente, el conjunto
de elecciones posibles para f
k
es {0, 1, 2, . . . , e
k
} , y existen e
k
+1 posi-
bilidades para elegir a f
k
. Por lo tanto el n umero de divisores de n es
(e
1
+ 1)(e
2
+ 1) (e
k
+ 1) .
Ejemplo 2.6 Pruebe que un n umero natural n es un cuadrado perfecto
si y solo si tiene un n umero impar de divisores.
50 Teora de N umeros
En efecto, sea n = p
e
1
1
p
e
2
2
p
e
k
k
la descomposicion primaria de n.
Ahora es claro que n es un cuadrado perfecto si y solo si todos los
exponentes e
1
, . . . , e
k
son pares, en cuyo caso el producto (e
1
+1)(e
2
+
1) (e
k
+ 1) es un producto de n umeros impares, y por lo tanto es
un n umero impar. Sin embargo por el resultado anterior, el n umero de
divisores de n es igual a (e
1
+1)(e
2
+1) (e
k
+1) . Por lo tanto, n es
un cuadrado perfecto si y solo si el n umero de divisores de n es impar.
Ejemplo 2.7 Para todo n umero natural n 2 , se tiene que
1 +
1
2
+
1
3
+ +
1
n
no es n umero entero.
En efecto, denotemos por A(n) el conjunto de los primeros n n umeros
naturales, es decir,
A(n) = {1, 2, 3, ..., n} .
Podemos suponer que n es un n umero que se encuentra entre 2

y
2
+1
para alg un > 1 , es decir, n = 2

+k , con 0 k < 2

. Luego los
n umeros de la forma 2
m
, con m 1 , estan en dicho conjunto, y ademas
ellos son divisores de un n umeros z si y solo si en la descomposicion
primaria de z aparece 2

.
Para sumar la expresion pedida se necesita calcular el mnimo com un
m ultiplo de los elementos del conjunto A(n) . Este mnimo com un
m ultiplo es el menor entero divisible por todo elemento de A(n) , luego,
por el T.F.A., ese n umero debe tener la forma 2

b , con b impar.
Sumando obtenemos
1 +
1
2
+
1
3
+ +
1
n
=
a
2
b
.
Sergio Plaza 51
Basta con probar que a no es divisible por 2. Reescribiendo esta
igualdad y multiplicandola por 2

b , se tiene
2

b +
2

b
2
+
2

b
3
+ +
2

b
2

+
2

b
2

+ 1
+
2

b
2

+ 2
+ +
2

b
2

+k
= a .
Puesto que 2

b es divisible por todos los n umeros de A(n) , todas


las fracciones en el lado izquierdo de la ultima igualdad son n umeros
enteros. Ademas, son n umeros pares, pues la maxima potencia de 2 que
puede aparecer en la descomposicion primaria de cualquier n umero del
conjunto A(n) (que son los denominadores de tales fracciones) es .
Luego a debe ser impar.
Ejemplo 2.8 Encuentre el valor mnimo de la expresion z dado por
z =
p
q
+
q
p
,
donde p y q son n umeros enteros positivos.
Solucion. Debido a que la expresion z es simetrica en p y q , podemos
suponer, sin perdida de generalidad, que p q .
Aplicando el T.F.A. sabemos que existe un n umero entero k 1 tal
que q = kp + r , donde r es un n umero entero con 0 r < p . Por
tanto z puede escribirse como sigue
z = k +
r
p
+
p
q
.
Claramente, el valor mnimo de z se obtiene cuando k = 1 y r = 0 .
Luego q = 1 p + 0 = p y entonces el mnimo de z se obtiene cuando
p = q y tal valor es 2.
52 Teora de N umeros
Ejemplo 2.9 Sean p y q enteros positivos. Si 2
p
+ 1 = q
2
, pruebe
que p = q = 3 .
Solucion. Notemos que encontrando q se obtiene inmediatamente p
y que la igualdad puede ser escrita como 2
p
= (q 1)(q + 1) . Esto
signica que q 1 divide a 2
p
. Aplicando el Teorema Fundamental
de la Aritmetica (T.F.A) se obtiene que necesariamente q 1 es una
potencia de 2. En resumen, se tiene que q 1 = 2
n
, con n p , y por
lo tanto la igualdad se transforma en
2
p
= 2
n
(2
n
+ 2) = 2
n
2(2
n1
+ 1) = 2
n+1
(2
n1
+ 1) .
De esta igualdad se deduce que (2
n1
+1) debe ser una potencia de 2
(por T.F.A), y esto sucede si y solamente si n = 1 . Por lo tanto q = 3
y p = 3 .
Consideremos la lista de los primeros n umeros primos
2, 3, 5, 7, 11, 13, 17, 19, 23, 29, 31, 37, 41, 43, 47, 53, 59, 61, 67, 71, 73, 79,
83, 89, 97, 101, 103, 107, 109, 113, 127, 131, 137, 139, 149, 151, 157, 163, 167,
173, 179, 181, 191, 193, 197, 199, . . .
Observemos que
(3, 5), (5, 7), (11, 13), (17, 19), (29, 31), (41, 43), (59, 61), (71, 73), (101, 103),
(107, 109), (137, 139), (149, 151), (179, 181), (191, 193), (197, 199), . . .
son pares de n umeros primos que satisfacen que su diferencia, en valor
absoluto, es igual a 2. Tales n umeros primos son llamados primos geme-
los, formalmente dos primos p y q son gemelos si |p q| = 2 . Una
pregunta natural se reere a la existencia de una cantidad innita o no
Sergio Plaza 53
de primos gemelos. Tambien nos podemos preguntar acerca de su dis-
tribucion entre los n umeros enteros positivos, por ejemplo, vemos que
entre 1 y 99 hay 8 pares de primos gemelos, y entre 100 y 199 hay 7
pares de n umeros gemelos, ahora, si consideramos los primos entre 200
y 299 tenemos
211, 223, 227, 229, 233, 239, 241, 251, 257, 263, 269, 271, 277, 281, 283, 293
y formando las parejas de n umeros primos gemelos, obtenemos
(227, 229), (239, 241), (269, 271) (281, 283) ,
es decir, hay 4 parejas de primos gemelos. Una buena idea es seguir
buscando este tipo de pares de n umeros primos y tratar de comprender
algo mas sobre ellas.
Cuantos n umeros primos terminan con el dgito 7? Por ejemplo,
7, 17, 37, 47, 97, 107, 127, 137, 157, 167, 197, . . .
Se puede calcular que de los 664579 primos menores que 10.000.000 el
n umero de ellos que terminan con el dgito 1, 3, 7 y 9, respectivamente
son: 1663104, 166230, 166211 y 166032.
En porcentaje estos corresponden al 24,99%, 25,01%, 25,01 % y 24,98%,
respectivamente que nos sugiere este tipo de estimaciones?
Ejemplo 2.10 Dado un entero positivo n, sea de p(n) el producto de
los dgitos no ceros de n, si n tiene un solo dgito entonces p(n) es
igual a ese dgito. Sea s = p(1) +p(2) + +p(999) Cual es el mayor
factor primo de s ?
Solucion. Notemos que los dgitos no cero de un entero positivo son
los que importan, por ejemplo p(108) = p(180) = p(810) = p(800) =
54 Teora de N umeros
p(811) = p(81) = p(18) = p(8) = 8.
Obtenemos todos los n umeros de 3 dgitos 001 a 999 desarrollando
(0 + 1 + 2 + + 9)
3
0,
sustraemos el cero para eliminar el n umero 000 (= 0). Luego,
(10 + 1 + + 9)
3
0 = 001 + 002 + + 999.
Para obtener el valor p(n) , para un n umero n, basta sustituir los
posibles ceros en su representacion decimal por 1, as
p(1) +p(2) + +p(n) = 1 1 1 + 1 1 2 + + 999
=(1 + 1 + 2 + + 9)
3
1
= 46
3
1 .
En la suma anterior, 111 se repite varias veces, una para 001, una para
011, una para 100, una para 101, una para 110,. . . . Como 46
3
1 =
3
3
5 7 103 , el n umero primo buscado es 103.
Recordemos que si R, entonces la parte entera de , es [] Z,
y corresponde al mayor entero menor o igual que , esto es, [] = m
es el unico entero satisfaciendo m < m+ 1 .
Observacion. Sean , R.
(i) Tenemos 1 < [] y 0 [] < 1 .
(ii) Si 0 , entonces [] cuenta el n umero de enteros positivos que
no exceden a . En otras palabras
[] =

1n
1 .
Sergio Plaza 55
(iii) Para cada n Z, tenemos [ +n] = [] +n.
(iv) Tenemos [] + [] [ +] [] + [] + 1 .
(v) Si Z, entonces [] + [] = 0 .
vi Si Z, entonces [] + [] = 1 .
(vii) El n umero [] es el menor entero no menor que .
(viii) Si n N, entonces [[]/n] = [/n] .
(ix) El n umero [ + 1/2] es uno de los enteros mas proximo a .
Ademas, si esos dos enteros ambos dieren de por el mismo
valor, entoces [ + 1/2] es el mayor de esos n umeros.
(x) Si > 0 y n N, entonces [/n] es el n umero de enteros
positivos no excediendo a y los cuales son m ultiplos de n.
Sea p un primo. Para cualquier entero positivo n, un problema
interesante es encontrar el mayor entero k tal que p
k
|n! .
Teorema 2.10 Sean n N y p un primo. Entonces el mayor entero
k tal que p
k
|n! es dado por
k =

j1
_
n
p
j
_
.
Demostracion. Sean m N con 1 m n. Si p
r
|m y p
r+1
divide
a m, queremos contar una contribucion de r .
56 Teora de N umeros
En otras palabras, contamos una contribucion de 1 para cada j N
tal que p
j
|m. Luego,
k =
n

m=1

j 1
p
j
|m
1 =

j1
n

m 1
p
j
|m
1 =

j1
_
n
p
j
_
.
2.0.3 Ejercicios
Problema 2.4 Conjetura de Goldbach.
2
Goldbach propuso su conjetura, basado solamente en evidencias empricas,
esta establece que cada n umero par mayor 2 que puede ser representado
como suma de dos primos. Por ejemplo,
4 = 2 + 2
6 = 3 + 3
8 = 3 + 5
10 = 3 + 7 = 5 + 5
12 = 5 + 7
14 = 3 + 11 = 7 + 7
16 = 3 + 13
18 = 5 + 13
20 = 3 + 17
22 = 3 + 19 = 5 + 17
24 = 5 + 19
2
Es conjetura fue formulada por el empleado civil del gobierno Ruso, Christian
Goldbach, en 1742 en una carta al gran matematico suizo Leonhard Euler
Sergio Plaza 57
26 = 3 + 23
28 = 5 + 23
30 = 7 + 23
Febrero 1 de 2005. Se ha vericado computacionalmente la conjetura
de Goldbach hasta n = 2

10
17
.
La conjetura de Goldbach es equivalente a que cada entero mayor que
5 es la suma de tres primos.
Es interesante la respuesta de Euler a la carta de Goldbach.
Que cada n umero par es una suma de dos primos, lo considero un
teorema enteramente cierto en el espritu de que yo no soy capaz de
demostrarlo.
Algunos progresos se han hecho sobre este problema. Por ejemplo,
se ha probado que cada entero par es la suma de a lo mas seis primos
(Goldbach sugiere dos) y en 1966 Chen probo que cada entero par su-
cientemente grande es la suma de un primo mas de dos factores primos.
Vinogradov en 1937 mostro que cada entero impar sucientemente
grande puede ser escrito como la suma de a lo mas tres primos, y luego
cada entero sucientemente grande es la suma de a lo mas cuatro primos.
Un resultado del trabajo de Vinogradov es que conocemos la conjetura
de Goldbach vale para casi todo entero par.
Problema 2.5 Es cada n umero de la forma 4n + 2 (n > 1) la suma
de dos primos de la forma 4n + 1 ? (Euler).
Problema 2.6 Puede cada n umero par ser expresado como la diferen-
cia de dos primos? Por ejemplo,
2 = 5 3
58 Teora de N umeros
4 = 7 3
6 = 11 5
8 = 13 5
10 = 17 7
.
.
.
Problema 2.7 Puede un n umero par ser expresado como la diferencia
de dos primos de innitas maneras? Por ejemplo,
2 = 5 3 = 7 5 = 19 17 = 31 29 =
Problema 2.8 Existen innitos primos gemelos? Los primos gemelos
son aquellos primos p y q tales que |pq| = 2 , es decir, primos p tales
que p + 2 tambien es primo. Por ejemplo, (11, 13), (17, 19), (29, 31), . . .
son pares de primos gemelos.
Problema 2.9 Sean k , , m y n n umeros enteros positivos tales que
m + k + 1 es un primo mayor que n + 1 . Sea c
s
= s(s + 1) . Pruebe
que el producto
(c
m+1
c
k
) (c
m+2
c
k
) (c
m+n
c
k
)
es divisible por el producto c
1
c
2
c
n
.
Problema 2.10 Existen innitos primos de la forma n
2
+ 1 ?
Por ejemplo, 5 = 2
2
+ 1 , 17 = 4
4
+ 1, . . .
Problema 2.11 Existe un entero positivo n que n tiene exactamente
200 divisores primos y 2
n
+ 1 es divisible por n?
Sergio Plaza 59
Problema 2.12 Pruebe que todos los n umeros de forma 2
2
k
+ 1 son
mutuamente coprimos
Problema 2.13 Sean n N y R un n umero no negativo.
Pruebe que
n1

k=0
_
+
k
n
_
= [n] (identidad de Hermite).
Problema 2.14 Encuentre el mayor factor primo de
_
200
100
_
.
Problema 2.15 Suponga que n N, y que p >

2n es un primo tal
que p divide a
_
2n
n
_
. Pruebe que p
2
no divide a
_
2n
n
_
.
Problema 2.16
Problema 2.17
2.1 Postulado de Bertrand
Teorema 2.11 (Postulado de Bertrand, probado por Chebyschef) Para
cada entero positivo n > 1 existe un primo p tal que n < p < 2n.
Probaremos primero el siguiente resultado.
Teorema 2.12 Sea n 2 un entero, entonces

pn
p < 4
n
,
donde el producto del lado izquierdo tiene un factor por cada primo p
n.
60 Teora de N umeros
Demostracion. Por induccion sobre n. Para n = 2 , tenemos el unico
primo p = 2 , y es obvio que 2 < 4
2
. Supongamos que el resultado vale
para todo etero menor que n. Si n es para, entonces no es primo, luego
por induccion

pn

pn1
p < 4
n1
< 4
n
y el paso inductivo en este caso es trivial. Supongamos que n = 25+1 es
impar, es decir, s =
n 1
2
. Como

s+1<pn
es un divisor de
_
n
s + 1
_
,
obtenemos

pn
p =

ps+1
p

s+1<pnp
4
s+1
_
n
s + 1
_
< 4
s+1
2
n1
usando induccion para n = s + 1 y la desigualdad: si n es impar,
entonces
_
n
(n + 1)/2
_
2
n1
cuya demostracion se deja a cargo del lector.
Ahora,
4
s+1
2
n1
= 2
25+2
2
n1
= 2
45+2
= 4
25+1
= 4
n
Esto completa la prueba del teorema.
Prueba del Postulado de Bertrand.
Sergio Plaza 61
Supongamos que no existen primos entre n y 2n.
Consideremos la factorizacion primaria para
_
2n
n
_
, es decir,
_
2n
n
_
=

pn
p
sp
,
donde sp es el exponente y del factor primo p en esta factorizacion.
Ning un primo mayor que n puede ser encontrado en esta factorizacion.
En efecto, para ello, usamos el siguiente lema.
Lema 2.1 Sea n 3 un entero y p un primo. Sea s el mayor expo-
nente tal que p
s
|
_
2n
n
_
. Entonces
1. p
s
2n
2. Si

2n < p entonces s 1
3. Si 2n/3 < p n, entonces s = 0 .
Por la parte c) del lema, podemos escribir
_
2n
n
_
=

p2n/3
p
sp
y tenemos p
sp
2n y que s
p
= 1 para p >

2n. Luego
_
2n
n
_

2n
p
sp

p2n/3
Ahora, no tenemos mas que

2n
n
1 factores en el primer factor del
producto del lado derecho. Luego
62 Teora de N umeros
_
2n
n
_
< (2n)

2n
2
1
4
2n/3
(1)
Por otra parte, si n es par, se tiene que
_
n
n/2
_

2
n
2
(prueba a cargo del lector). Luego
_
2n
n
_

2
2n
2n
=
4
n
2n
(2)
Combinando (1) y (2), obtenemos
4
n/3
< (2n)

n/2
Tomando logaritmo a ambos lados de esta desigualdad, nos queda
2n
3
ln(2) <
_
n
2
ln(2n)
esto es,

8nln(2) 3 ln(2n) < 0 (3)


Reemplazando n = 2
2k3
para alg un k .
Sergio Plaza 63
Tenemos entonces 2
k
ln(2) 3(2k 2) ln(2) < 0 o 2
k
< 3(2k 2) lo
cual es verdadero para k 4 . Luego (3) no es verdadero para n = 2
7
=
128 .
Usando calculo, se puede ver que (3) no es verdadera para todo n
128 . Luego, no es verdadera para todo n 128 . Luego, hemos probado
el postulado de Bertrand para n 128 . Para n 128 , se puede hacer
por impeccion derecta de esos pocos casos.
2.2 N umeros de Fermat
Recuerde que los n umeros de Fermat, F
n
, son denidos por
F
n
= 2
2
n
+ 1
para n 0 .
Los primeros cinco n umeros de Fermat son
F
0
= 3, F
1
= 5, F
2
= 17, F
3
= 257, F
4
= 65537
ellos son primos. Esto llevo a Fermat a conjeturar que cada n umero F
n
es primo para todo n 0 .
Cerca de 100 a nos mas tarde, en 1732, Euler probo que esta conjetura
era falsa, para ello mostro que
F
5
= 2
32
+ 1 = 4294967297 = 641 6700417 .
Los n umeros de Fermat que son primos son llamados primos de Fer-
mat, y hasta hoy da no se sabe si existen innitos primos de Fermat.
64 Teora de N umeros
No se han encontrado primos de Fermat mas alla de F
4
. Sin embargo
el mayor n umero de Fermat conocido es F
234711
, y fue descubierto por
W. Keller en 1984.
La siguiente demostracion del hecho que F
5
es compuesto no se usa
ninguna division y es debida a G. Bennet.
Armacion. F
5
es compuesto.
En efecto, primero notemos que
641 = 5 2
7
+ 1 .
Ahora,
F
5
= 2
2
5
+ 1
= 2
32
+ 1
= 2
4
2
28
+ 1
= 16 2
28
+ 1
= (641 625)2
28
+ 1
= (641 5
4
)2
28
+ 1
= 641 2
28
(5 2
7
)
4
+ 1
= 641 2
28
(641 1)
4
+ 1
= 641 2
28
(641
4
4 641
3
+ 6 641
2
4 641 + 1) + 1
= 641(2
28
641
3
+ 4 641
2
6 641 + 4)
= 641 6700417
= 4294967297 .
Luego 641|F
5
.
Sergio Plaza 65
Quizas, una de las mas importantes propiedades de los n umeros de
Fermat, es la conexion encontrada por Gauss en 1796 entre los primos de
Fermat y la construccion con regla (sin marcas) y compas de polgonos
regulares.
Teorema 2.13 Un polgono regular con n lados es constructible us-
ando solamente una regla y compas si y solo si
n = 2
r
F
1
F
2
F
k
,
donde r 0 y F
1
, F
2
, . . . , F
k
son distintos primos de Fermat.
Los antiguos Griegos saban como construir polgonos regulares, usan-
do solamente regla y compas, con lados 2
k
, 32
k
, 52
k
y 152
k
= 2
k
35 .
Tambien saban como construir polgonos regulares con 3, 4, 5, 6, 8,
10, 12, 15 y 16 lados, pero no saban como construir un polgono
regular con 17 lados. La construccion de un polgono con 17 lados fue
hecha por Gauss cuando tena 19 a nos, y seg un cuenta la historia, esto
lo llevo a dedicar el resto de su vida a la matematica.
Damos a seguir una relacion de recurrencia que satisfacen los n umeros
de Fermat y usamos esto para probar algunos resultados interesantes.
Teorema 2.14 Para cada entero n 0 , se tiene
F
n+1
= F
0
F
1
F
2
F
n
+ 2 .
Demostracion. Como F
0
= 2
2
0
1 = 1 , tenemos
F
0
F
1
F
2
F
3
F
n
= 1 F
0
F
1
F
2
F
3
F
n
66 Teora de N umeros
= (2
2
0
1)(2
2
0
+ 1)(2
2
1
+ 1)(2
2
2
)(2
2
3
+ 1) (2
2
n
+ 1)
= (2
2
2
1)(2
2
2
+ 1)(2
2
3
+ 1) (2
2
n
+ 1)
= (2
2
3
1)(2
2
3
+ 1) (2
2
n
+ 1)
.
.
.
.
.
.
= (2
2
n
1)(2
2
n
+ 1)
= 2
2
n+1
1
= 2
2
n+1
+ 1 2
= F
n+1
2 .
Teorema 2.15 Dos n umeros de Fermat distintos F
m
y F
n
, con m >
n, son coprimos.
Demostracion. Sean F
m
y F
n
dos n umeros de Fermat distintos, con
m > n. Supongamos que d > 0 es un divisor de F
m
y F
n
, entonces d
divided a
2 = F
m
F
0
F
1
F
n
F
m1
.
Por lo tanto d = 1 o d = 2 , pero F
m
y F
n
son impares, luego
debemos tener que d = 1 . Lo que completa la prueab del teorema.
Damos a continuacion una prueba elemetal sobre la innitud de los
primos debida a G. Polya.
Teorema 2.16 Existen innitos primos.
Demostracion. (G. Polya) Existen innitos n umeros de Fermat dis-
tintos, cada uno de ellos es divisible por un primo impar, y como dos
n umeros de Fermat distintos son coprimos, esos primos impares deben
ser todos distintos. Luego, existen innitos primos.
Sergio Plaza 67
Teorema 2.17 Para cada entero n 0 , el entero positivo
N = 2
2
n
1
es divisible por al menos n primos diferentes.
Demostracion. Para cada n 0 , tenemos
2
2
n
1 = 2
2
n
+ 1 2
= F
n
2
= F
0
F
1
F
2
F
3
F
n1
,
y como todos los F
k
son coprimos, el resultado se sigue.
2.3 N umeros de Mersenne
Denotemos por M
n
= 2
n
1 , los n umeros de Mersenne.
Teorema 2.18 Si d|n , entonces M
d
|M
n
.
Demostracion. Escribamos n = dr entonces la identidad
x
dr
1 = (x
d
1)(x
r
d 1 +x
d2
+ +x
r
+ 1)
nos muestra que para x = 2 , se tiene que M
d
= 2
d
1|2
dr
1 = M
n
.
Corolario 2.1 Si M
n
es primo, entonces n es primo.
Demostracion. Por el teorema anterior si n es compuesto entonces
M
n
. Supongamos por lo tanto que n = a b con 1 < a, b < n.
Entonces M
a
|M
b
por el teorema anterior, M
a
> 1 pues a > 1 , y
M
a
< M
n
pues a < n.
68 Teora de N umeros
2.4 N umeros Triangulares
Un entero positivo n es un n umero triangular si es de la forma
n =
k(k + 1)
2
,
donde k N. Luego, el mesimo n umero triangular es
t
m
=
m(m+ 1)
2
.
Notemos que para cada m N, se tiene
t
m
= 1 + 2 + +m =
_
m+ 1
2
_
,
t
m+1
= t
m
+ (m + 1)
y
(t
n
)
nN
= {1, 3, 6, 10, . . . ,
n(n + 1)
2
, . . .} .
El nombre de n umeros triangulares se debe al hecho podemos colo-
carlos como sigue, formando triangulo equilateros
La siguiente caracterizacion de los n umeros triangulares entre los
n umeros naturales es debida a Plutarco (cerca 100 D.C.).
Sergio Plaza 69
Teorema 2.19 Un n umero n N es triangular si y solo si 8n +1 es
un cuadrado perfecto.
Demostracion. Supongamos que n es un n umero triangular, es decir,
n = t
k
para alg un k N. Luego,
8n + 1 = 8t
k
+ 1
= 8
k(k + 1)
2
+ 1
= 4k
2
+ 4k + 1
= (2k + 1)
2
que es un cuadrado perfecto.
Recprocamente, si n N es tal que 8n + 1 = k
2
es un cuadrado
perfecto. Tenemos entonces que k debe ser impar y k 3 . Luego
k1
2
N. Tomamos m =
k1
2
y tenemos
t
m
= t k1
2
=
k1
2
_
k1
2
+ 1
_
2
=
k
2
1
8
= n,
esto es, n es un n umero triangular, mas precisamente n es el
k1
2
esimo termino en la sucesion de los n umeros triangulares.
Teorema 2.20 Entre los n umeros de Fermat F
n
, no hay cuadrados,
no hay cubos, y no hay n umeros triangulares, excepto F
0
= 3 =
2 3
2
.
Teorema 2.21 Si n N es un cuadrado perfecto, entonces se tiene
a) Si n es par, entonces n es divisible por 4.
b) Si n es impar, entonces n es de la forma 8k + 1 , con k N,
esto es, cuando n es dividido por 8 deja resto 1.
Demostracion. Sea m N tal que n = m
2
.
70 Teora de N umeros
a) Si m = 2k es par, entonces n = m
2
= 4k
2
es divisible por 4.
b) Si m = 2l1 es impar, entonces n = m
2
= (2l1)
2
= 4l
2
4l+1 =
4(l 1)l + 1 .
Como el producto (l1)l de dos n umeros naturales consecutivos siempre
es par, se tiene que (l 1)l = 2k , y nos queda n = 4 2k + 1 = 8k + 1 .
Ejemplo 2.11 En la sucesion de n umeros 11, 111, 1111, . . . , 111 . . . 1111, . . .
no aparece ning un cuadrado perfecto.
En efecto, tenemos que 11 = 8+3 y n = 111 . . . 1111 = 111 . . . 1000+
111 = 8l +8 13+7 = 8k+7 para n 111 , esto signica que ninguno de
los n umeros en la sucesion es de la forma 8k + 1 , que es una condicion
necesaria para ser un cuadrado perfecto.
Ejemplo 2.12 a) n = 12 es divisible por 4, pero no es cuadrado
perfecto.
b) n = 17 = 2 8 + 1 , es de la forma 8k + 1 pero no es un cuadrado
perfecto.
N umeros naturales que son diferencia de dos cuadrados, es decir, n =
x
2
y
2
, con x N, y N{0} . Note por ejemplo que n = 2 y n = 6
no pueden ser escritos como diferencia de dos cuadrados como arriba.
Tenemos el siguiente resulado
Teorema 2.22 Sea n N, entonces n = x
2
y
2
para x N e
y N {0} si y solo si n no es de la forma 4k + 2, k N {0} .
Demostracion. Supongamos que n = x
2
y
2
, con x N, y N{0} .
Para probar que n no es de la forma 4k +2 , con k N{0} , podemos
Sergio Plaza 71
suponer que n es par (si n es impar esto es inmediato). De esto se
sigue que x e y ambos son pares o ambos son impares. Si x = 2k e
y = 2l , tenemos que n = x
2
y
2
= (2k)
2
(2l)
2
= 4(k
2
l
2
) que
no es de la forma 4m + 2 . Si x = 2k 1 e y = 2l 1 , tenemos
n = x
2
y
2
= (2k 1)
2
(2l 1)
2
es de la forma 4m + 2 .
Recprocamente, supongamos que n no es de la forma 4m + 2 , esto
implica que n es impar o divisible por 4.
Si n es impar, entonces n 1 es par y por lo tanto
n 1
2
,
n + 1
2

N{0} . Ahora,
_
n+1
2
_
2

_
n1
2
_
2
=
1
4
((n +1)
2
(n 1)
2
) = n, esto es
n =
_
n + 1
2
_
2

_
n 1
2
_
2
es una posible descomposicion de n como diferencia de dos cuadrados.
Si n = 4k , entonces n = (k + 1)
2
(k 1)
2
, es decir,
n =
_
n
4
+ 1
_
2

_
n
4
1
_
2
Observacion. Podemos escribir N como
N = {4k : k N}{4k+1 : k N{0}} {4k+2 : k {0}}{4k+3 : k N{0}}
y vemos que de estos, solamente los n umeros del conjunto {4k +2 : k
N {0}} no pueden ser escritos como diferencia de dos cuadrados.
72 Teora de N umeros
2.4.1 Ejercicios
Problema 2.18 (Teorema de Wolstenholme) Sea p 5 un primo. En-
tonces p
2
divide al numerador de la fraccion resultante del n umero
1 +
1
2
+
1
3
+ +
1
p 1
.
Problema 2.19 Sean p
1
, p
2
, . . . , p
n
primos distintos mayores que 3.
Pruebe que
2
p
1
p
2
pn
+ 1
tiene al menos 4
n
divisores Que ocurre si alg un p
i
es igual a 2 o 3 ?
Problema 2.20 Sea p un primo y n 1 un entero tal que n divide
a p 1 y p divide a n
3
1 . Pruebe que 4p + 3 es el cuadrado de un
n umero entero. Ilustre el resultado con algunos ejemplos.
Problema 2.21 Sea p un primo con p > 5 , y sea S = {p n
2
: n
N, n
2
< p} . Pruebe que S contiene dos elementos a y b tales que
1 < a < b y a|b .
Problema 2.22 Suponga que 2
n
+ 1 es un primo impar para alg un
entero positivo n. Pruebe que n debe ser una potencia de 2 .
Problema 2.23 Sea p un primo mayor que 3. Pruebe que 7
p
6
p
1
es divisible por 43.
Problema 2.24 Suponga que 4
n
+ 2
n
+ 1 es primo para alg un entero
positivo n. Muestre que n debe ser una potencia de 3 .
Sergio Plaza 73
Problema 2.25 Para cada entero positivo n > 1 , sea p(n) el mayor
divisor primo de n. Pruebe que existen innitos enteros positivos n
con p(n) < p(n + 1) < p(n + 2) .
Problema 2.26 Determine todos los pares (n, p) de enteros no nega-
tivos tales que
1. p es primo,
2. n < 2p , y
3. (p 1)
n
+ 1 es divisible por n
p1
.
Problema 2.27 Encuentre todos los n umeros naturales n tal que el
n umero n(n+1)(n+2)(n+3) tiene exactamente tres divisores primos.
Problema 2.28 El siguiente resultado es debido a Dirichlet. Para
a, b, c N, con mcd (a, b) = 1 , existen innitos primos de la forma
ak +b .
Usando este resultado pruebe que existen innitos primos que termi-
nan con el dgito 9.
Por ejemplo: 19, 29, 59, 79, 89, 139, 149, . . . .
Problema 2.29 Usando el teorema de Dirichlet (problema anterior)
pruebe que existen innitos primos de la forma 2pk +1 , donde p es un
primo impar.
Problema 2.30 Sean b , m y n enteros positivos, con b > 1 y m = n.
Si b
m
1 y b
n
1 tienen los mismos factores primos. Pruebe que b +1
debe ser una potencia de 2.
74 Teora de N umeros
Problema 2.31 Pruebe que existen innitos pares ordenados (a, b) de
enteros tal que para cada entero positivo t , el n umero at + b es un
n umero triangular si y solo si t es un n umero triangular.
Problema 2.32 Muestre que existen dos cuadrados consecutivos tales
que entre ellos existen al menos 1000 primos.
Problema 2.33 Existen innitos primos de Fermat, es decir, primos
de la forma? F
n
= 2
2
n
+ 1? Se sabe que F
0
, F
1
, F
2
, , F
3
, F
4
son
primos, pero F
5
nos lo es (Euler probo esto).
Problema 2.34 Existen innitos primos de Mersenne, es decir, primos
de la forma 2
p
1 , con p primo?
Problema 2.35 Existen innitos primos de la forma 2p +1 , donde p
es primo? Por ejemplo,
7 = 2 3 + 1, p = 3 primo
11 = 2 5 + 1, p = 5 primo
23 = 2 11 + 1, p = 11 primo
.
.
.
Problema 2.36 Existe al menos un primo entre cada par de cuadrados
consecutivos?
n = 1, 1
2
= 1 3 2
2
= 4
n = 2, 2
2
= 4 5, 7 3
2
= 9
n = 3, 3
2
= 9 11, 13 4
2
= 16
.
.
.
Sergio Plaza 75
Problema 2.37 Existen innitos primos p para los cuales 2
p1
es
divisible por p
2
?
Problema 2.38 Existen innitos primos p para los cuales (p1)! +1
es divisible por p
2
?
Problema 2.39
Problema 2.40
Problema 2.41
Problema 2.42
Problema 2.43
Problema 2.44
Problema 2.45
Problema 2.46 Sea f(n) = (n) n, donde (n) denota la suma de
los divisores de n. Por ejemplo, f(1) = 0 , f(2) = 1 + 2 2 = 1 ,
f(3) = 1 + 3 3 = 1 , f(4) = 1 + 2 + 4 4 = 3 , f(5) = 1 + 5 5 = 1 ,
f(6) = 1+2+3+66 = 6 , f(7) = 1 , f(8) = 1+2+4+88 = 7, . . . .
Note que si n es primo entonces f(n) = 1 , pues los divisores de n
son 1 y n.
Para n = 8 , tenemos f(8) = 7 , f(7) = 1 y f(1) = 1 , si n = 9 ,
f(9) = 1 + 3 + 9 9 = 4 , f(4) = 3 , f(3) = 1 , y f(1) = 1 . Para
n = 10 , f(10) = 1+2+55 = 3 , f(3) = 1 y f(1) = 1 . Caso n = 12 ,
f(12) = 1 + 3 + 4 + 6 + 12 12 = 14 , f(14) = 1 + 7 + 14 14 = 8 ,
f(8) = 7 , f(7) = 1 y f(1) = 1 . Haga mas ejemplos y formule una
76 Teora de N umeros
conjetura respecto al calculo , n, f(n) , f(f(n)) , f(f(f(n)) ),... (Si su
conjetura es la siguiente, n, f(n)) , f(f(n)) , f(f(f(n)) ),... se vuelve
periodica. Entonces a ella no se le conoce si es verdadera o falsa. Por
ejemplo, f(95) = 25 , f(25) = 6 , f(6) = 6 , f(6) = 6 ,...
Problema 2.47 Sea a
n
= 6
n
+ 8
n
. Calcule el resto de la division de
a
83
por 49.
Problema 2.48 Si 30x0y03 es divisible por 13 encuentre x e y .
Problema 2.49 Pruebe que su 9|(a
3
+b
3
+c
3
) entonces 3|(abc) , para
enteros positivos a, b, c .
Problema 2.50 Encuentre el ultimo dgito de 3
100
.
Problema 2.51 Pruebe que si 7|(a
2
+b
2
) entonces 7|a y 7|b .
Problema 2.52 Pruebe que para todo n, se tiene que n
9
6n
7
+9n
5

4n
3
es divisible por 8640.
Problema 2.53 Pruebe que para cada entero positivo n se tiene que
( n + 1) (n + 2) 2n es divisible por 2
n
.
Problema 2.54 Determine los ultimos dgitos de los n umeros en la
sucesion 23 23
23
, 23
(23
23
)
,. . .
Problema 2.55 Pruebe que tres n umeros enteros positivos n, n+2, n+
4 no pueden simultaneamente primos, salvo si n = 3 .
Problema 2.56 Sea p > 3 un primo.
1. Explique porque p = 6k + 1 o p = 6k 1 para alg un k N.
Sergio Plaza 77
2. Use (a) para probar que 24|(p
2
1) .
Problema 2.57 Pruebe que 24|n(n
2
1) para cada entero positivo
impar n.
Problema 2.58 Sean a, b, c N.
1. Pruebe que si 3|(a
2
+b
2
) , entonces 3|ab .
2. Pruebe que si 9|(a
3
+b
3
+c
3
) , entonces 3|abc .
Problema 2.59 Pruebe que 2
n
|(n+1)(n+2) (2n) para cada n N.
Problema 2.60 Pruebe que si p es primo entonces
_
_
p
i
_
_
0 (mod p)
para 1 i p 1 .
Use esto para probar que
(1 +x)
p
1 +x
p
(mod p)
Problema 2.61 Sea p 3 un primo y sea k =
_
2p
3
_
Pruebe que
_
p
1
_
+
_
p
2
_
+ +
_
p
k
_
es divisible por p
2
.
78 Teora de N umeros
Problema 2.2 Si a y b son enteros positivos coprimos, pruebe que
mcd (a +b, a
2
+ab +b
2
) = 1 .
Problema 2.62 Muestre que si q
1
, q
2
, , q
n
son primos, entonces ex-
iste un primo q , con q {q
1
, , q
n
} . (Indicacion. Considere el n umero
N = q
1
q
2
q
n
+ 1 . Por un lema anterior existe un primo q , tal que
q|N . Pruebe entonces que q {q
1
, q
2
, , q
n
} ).
Problema 2.63 Pruebe que, para cada primo impar, existen innitos
primos de la forma 2kp + 1 .
Problema 2.64 Sean p y q n umeros primos, tales que n = p q =
243449 y (n) = 241536 . Encuentre los primos p y q .
Problema 2.65 Pruebe que un n umero de la forma 2
n
+1 , con n > 1 ,
puede ser primo solo si n es una potencia de 2 .
Problema 2.66 Pruebe que un n umero de la forma 2
n
1 , con n > 1 ,
puede ser primo solo si n es primo.
Indicacion. 2
m1
divide a 2
n
1 cuando m divide a n.
Problema 2.67 Pruebe que existen innitos primos en la progresion
aritmetica innita 2, 5, 11, 14, 17, . . .
Problema 2.68 Sea p un n umero primo
1. Pruebe que
_
p
k
_
es divisible por p para k = 1, 2, . . . , p 1 .
2. Pruebe que 2
p
2 es divisible por p .
Problema 2.69
Sergio Plaza 79
Problema 2.70
Problema 2.71
Problema 2.72
Curiosidad Numerica
La siguiente tabla corresponde a los coecientes del desarrollo del
binomio de Newton
1
1 1
1 2 1
1 3 3 1
1 4 6 4 1
1 5 10 10 5 1
1 6 15 20 15 6 1
1 7 21 35 35 21 7 1
1 8 28 56 70 56 28 8 1
1 9 36 84 126 126 84 36 9 1
1 10 43 120 210 252 210 120 43 10
1 11 53 163 330 462 462 330 163 53 11
En esta tabla hemos considerado los coecientes del binomio de New-
ton modulo 2.
Si el lector tiene paciencia agregue mas las y columnas y vera apare-
cer una conocida forma fractal llamada triangulo de Sierpinski.
80 Teora de N umeros
1
1 1
1 0 1
1 1 1 1
1 0 0 0 1
1 1 0 0 1 1
1 0 1 0 1 0 1
1 1 1 1 1 1 1 1
1 0 0 0 0 0 0 0 1
1 1 0 0 0 0 0 0 1 1
1 0 1 0 0 1 0 0 1 0 1
1 1 1 1 0 0 0 0 1 1 1 1
2.4.2 Algunos comentarios sobre la distribucion de los
n umeros primos.
Una de las peculiaridades de los primos es la aparente ausencia de
un patron. Por ejemplo, hay exactamente nueve primos entre los 100
n umeros comprendidos entre 9.999.900 y 10.000.000. Sin embargo, hay
solamente dos primos entre 10.000.000 y 10.000.100. ( encontrarlos!).
Por otro lado hay 25 primos entre 1 y 100. Esta falta de regularidad
en la distribucion de los n umeros primos ha sido constante fuente de
discusion en la disciplina, que ha llevado al desarrollo de una amplia
variedad de ramas de la matematica. Este aparente desorden ya haba
tomado la atencion de Gauss (17771855
3
).
3
C.F. Gauss: Nacio el 30 de Abril 1777 en Brunswick, Alemania. Fallecio el 23
de Febrero 1855 en Gottingen, Hanover. Cuando Gauss tena diez a nos de edad, su
Sergio Plaza 81
maestro solicito a la clase que encontrara la suma de todos los n umeros comprendidos
entre uno y cien. El maestro, pensando que con ello la clase estara ocupada alg un
tiempo, quedo asombrado cuando Gauss, levanto en seguida la mano y dio la respuesta
correcta. Gauss revelo
que encontro la solucion usando el algebra, el maestro se dio cuenta de que el ni no
era una promesa en las matematicas. Hijo de un humilde alba nil, Gauss dio se nales
dio se nales de ser un genio antes de que cumpliera los tres a nos. A esa edad aprendio
a leer y hacer calculos aritmeticos mentales con tanta habilidad que descubrio un
error en los calculos que hizo su padre para pagar unos sueldos. Ingreso a la escuela
primaria antes de que
cumpliera los siete a nos. Cuando tena doce a nos, critico los fundamentos dela
geometra euclidiana; a los trece le interesaba las posibilidades de la geometra no
euclidiana. A los quince, entenda la convergencia y probo el binomio de Newton. El
genio y la precocidad de Gauss llamaron la atencion del duque de Brunswick, quien
dispuso, cuando el muchacho tena catorce a nos, costear tanto su educacion secun-
daria como universitaria. Gauss, a quien tambien le interesaban los clasicos y los
idiomas, pensaba que hara de la lologa la obra de su vida, pero las matematicas
resultaron ser una atraccion irresistible. Cuando estudiaba en Gotinga, descubrio que
podra construirse un polgono regular de diecisiete lados usando solo la regla y el
compas. Ense no la prueba a su profesor, quien se demostro un tanto esceptico y le dijo
que lo que sugera era imposible; pero Gauss demostro que tena la razon. El profesor,
no pudiendo negar lo evidente, armo que tambien el procedio de la misma manera.
Sin embargo, se reconocio el merito de Gauss, y la fecha de su descubrimiento, 30
de Marzo de 1796, fue importante en la historia de las matematicas. Posteriormente,
Gauss encontro la formula para construir los demas polgonos regulares con la regla
y el comp as. Gauss se graduo en Gotinga en 1798, y al a no siguiente recibio su doc-
torado en la Universidad de Helmstedt. Las matematicas no fueron el unico tema
que le intereso a este hombre; fue tambien astronomo, fsico, geodesta e inventor.
Hablaba con facilidad varios idiomas, e inclusive domino el ruso a la edad de sesenta
a nos. En 1807 fue nombrado director del observatorio y profesor de astronoma en la
Universidad de Gotinga. A principios del siglo XIX, Gauss publico sus Disquisiciones
aritmeticas, que ofrecan un analisis l ucido de su teora de n umeros, comprendiendo
las complicadas ecuaciones que conrmaban su teora y una exposicion de una con-
vergencia de una serie innita. Estudio la teora de los errores y dedujo la curva
82 Teora de N umeros
La idea genial fue no considerar los primos de a uno si no que tomarlos
de montones. Llamemos por (n) la cantidad de primos menor o igual
que n . Esta cantidad se puede calcular para varios valores de n. A
continuacion damos algunos valores para (n) .
El cociente n/(n) representa la proporcion de n umeros hasta n que
son primos. Examinando con detencion esta razon. Vemos que por
cada potencia de 10 la razon aumenta en 2,3. Este n umero es magico
para los matematicos, pues se parece a ln(10) , y por lo tanto, n/(n)
se debera parecer ln(n) o lo que es equivalente, (n) se aproxima
n/ ln(n) . Mas precisamente, podemos enunciar el famoso Teorema de
los N umeros Primos que arma que
lim
n
(n)
n/ ln(n)
= 1 .
normal de la probabilidad, llamada tambien curva de Gauss, que todava se usa en
los calculos estadsticos. En 1833 invento un telegrafo electrico que uso entre su
casa y el observatorio, a una distancia de unos dos kilometros. Invento tambien un
magnetometro biliar para medir el magnetismo y, con Weber, proyecto y construyo
un observatorio no magnetico. Tanto Gauss como Riemann, que fue discpulo suyo,
pensaban en una
teora electromagnetica que sera muy semejante a la ley universal de la gravitacion,
de Newton. Empero, la teora del electromagnetismo fue ideada mas tarde, en 1873,
por Maxwell, aunque Gauss ya posea los cimientos matematicos para la teora. En
1840, las investigaciones de Gauss sobre la optica tuvieron especial importancia debido
a sus deducciones por lo que toca a los sistemas de lentes. A la edad de setenta y
siete a nos, Gauss fallecio. Se ha dicho que la lapida que se nala su tumba fue escrita
con un diagrama, que construyo el mismo Gauss, de un polgono de diecisiete lados.
Durante su vida, se reconocio que era el matematico mas grande de los siglos XVIII
y XIX. Su obra en las matematicas contribuyo a formar una base para encontrar la
solucion de problemas complicadsimos de las ciencias fsicas y naturales.
Sergio Plaza 83
n (n) n/(n)
10 4 2,5
100 25 4,0
1000 168 6,0
10.000 1.229 8,1
100.000 9.592 10,4
1.000.000 78.498 12,7
10.000.000 664.579 15,0
100.000.000 5.761.455 17,4
1.000.000.000 50.847.534 19,7
10.000.000.000 455.052.512 22,0
Esto quiere decir que para n sucientemente grande la cantidad de
primos menor o igual que n es aproximadamente n/ ln(n) .
Ejercicio 2.1 Utilizando una calculadora o un computador, calcular los
valores de n/ ln(n) para n = 10
k
para valores de k = 1, 2, 3, 4, 5, 6, 7, 8, 9, 10
y compararlos con los valores exactos dados por la segunda columna de
la tabla para (n) .
2.5 Lista de los primeros 1000 n umeros primos
Los primeros 1000 primos, el primo n umero 1000 es 7919. Para mas in-
formacion sobre n umeros primos consultar la pagina siguiente http://www.utm.edu/research/primes
2 3 5 7 11 13 17 19 23 29
31 37 41 43 47 53 59 61 67 71
73 79 83 89 97 101 103 107 109 113
127 131 137 139 149 151 157 163 167 173
84 Teora de N umeros
179 181 191 193 197 199 211 223 227 229
233 239 241 251 257 263 269 271 277 281
283 293 307 311 313 317 331 337 347 349
353 359 367 373 379 383 389 397 401 409
419 421 431 433 439 443 449 457 461 463
467 479 487 491 499 503 509 521 523 541
547 557 563 569 571 577 587 593 599 601
607 613 617 619 631 641 643 647 653 659
661 673 677 683 691 701 709 719 727 733
739 743 751 757 761 769 773 787 797 809
811 821 823 827 829 839 853 857 859 863
877 881 883 887 907 911 919 929 937 941
947 953 967 971 977 983 991 997 1009 1013
1019 1021 1031 1033 1039 1049 1051 1061 1063 1069
1087 1091 1093 1097 1103 1109 1117 1123 1129 1151
1153 1163 1171 1181 1187 1193 1201 1213 1217 1223
1229 1231 1237 1249 1259 1277 1279 1283 1289 1291
1297 1301 1303 1307 1319 1321 1327 1361 1367 1373
1381 1399 1409 1423 1427 1429 1433 1439 1447 1451
1453 1459 1471 1481 1483 1487 1489 1493 1499 1511
1523 1531 1543 1549 1553 1559 1567 1571 1579 1583
1597 1601 1607 1609 1613 1619 1621 1627 1637 1657
1663 1667 1669 1693 1697 1699 1709 1721 1723 1733
1741 1747 1753 1759 1777 1783 1787 1789 1801 1811
1823 1831 1847 1861 1867 1871 1873 1877 1879 1889
1901 1907 1913 1931 1933 1949 1951 1973 1979 1987
1993 1997 1999 2003 2011 2017 2027 2029 2039 2053
2063 2069 2081 2083 2087 2089 2099 2111 2113 2129
Sergio Plaza 85
2131 2137 2141 2143 2153 2161 2179 2203 2207 2213
2221 2237 2239 2243 2251 2267 2269 2273 2281 2287
2293 2297 2309 2311 2333 2339 2341 2347 2351 2357
2371 2377 2381 2383 2389 2393 2399 2411 2417 2423
2437 2441 2447 2459 2467 2473 2477 2503 2521 2531
2539 2543 2549 2551 2557 2579 2591 2593 2609 2617
2621 2633 2647 2657 2659 2663 2671 2677 2683 2687
2689 2693 2699 2707 2711 2713 2719 2729 2731 2741
2749 2753 2767 2777 2789 2791 2797 2801 2803 2819
2833 2837 2843 2851 2857 2861 2879 2887 2897 2903
2909 2917 2927 2939 2953 2957 2963 2969 2971 2999
3001 3011 3019 3023 3037 3041 3049 3061 3067 3079
3083 3089 3109 3119 3121 3137 3163 3167 3169 3181
3187 3191 3203 3209 3217 3221 3229 3251 3253 3257
3259 3271 3299 3301 3307 3313 3319 3323 3329 3331
3343 3347 3359 3361 3371 3373 3389 3391 3407 3413
3433 3449 3457 3461 3463 3467 3469 3491 3499 3511
3517 3527 3529 3533 3539 3541 3547 3557 3559 3571
3581 3583 3593 3607 3613 3617 3623 3631 3637 3643
3659 3671 3673 3677 3691 3697 3701 3709 3719 3727
3733 3739 3761 3767 3769 3779 3793 3797 3803 3821
3823 3833 3847 3851 3853 3863 3877 3881 3889 3907
3911 3917 3919 3923 3929 3931 3943 3947 3967 3989
4001 4003 4007 4013 4019 4021 4027 4049 4051 4057
4073 4079 4091 4093 4099 4111 4127 4129 4133 4139
4153 4157 4159 4177 4201 4211 4217 4219 4229 4231
4241 4243 4253 4259 4261 4271 4273 4283 4289 4297
4327 4337 4339 4349 4357 4363 4373 4391 4397 4409
86 Teora de N umeros
4421 4423 4441 4447 4451 4457 4463 4481 4483 4493
4507 4513 4517 4519 4523 4547 4549 4561 4567 4583
4591 4597 4603 4621 4637 4639 4643 4649 4651 4657
4663 4673 4679 4691 4703 4721 4723 4729 4733 4751
4759 4783 4787 4789 4793 4799 4801 4813 4817 4831
4861 4871 4877 4889 4903 4909 4919 4931 4933 4937
4943 4951 4957 4967 4969 4973 4987 4993 4999 5003
5009 5011 5021 5023 5039 5051 5059 5077 5081 5087
5099 5101 5107 5113 5119 5147 5153 5167 5171 5179
5189 5197 5209 5227 5231 5233 5237 5261 5273 5279
5281 5297 5303 5309 5323 5333 5347 5351 5381 5387
5393 5399 5407 5413 5417 5419 5431 5437 5441 5443
5449 5471 5477 5479 5483 5501 5503 5507 5519 5521
5527 5531 5557 5563 5569 5573 5581 5591 5623 5639
5641 5647 5651 5653 5657 5659 5669 5683 5689 5693
5701 5711 5717 5737 5741 5743 5749 5779 5783 5791
5801 5807 5813 5821 5827 5839 5843 5849 5851 5857
5861 5867 5869 5879 5881 5897 5903 5923 5927 5939
5953 5981 5987 6007 6011 6029 6037 6043 6047 6053
6067 6073 6079 6089 6091 6101 6113 6121 6131 6133
6143 6151 6163 6173 6197 6199 6203 6211 6217 6221
6229 6247 6257 6263 6269 6271 6277 6287 6299 6301
6311 6317 6323 6329 6337 6343 6353 6359 6361 6367
6373 6379 6389 6397 6421 6427 6449 6451 6469 6473
6481 6491 6521 6529 6547 6551 6553 6563 6569 6571
6577 6581 6599 6607 6619 6637 6653 6659 6661 6673
6679 6689 6691 6701 6703 6709 6719 6733 6737 6761
6763 6779 6781 6791 6793 6803 6823 6827 6829 6833
Sergio Plaza 87
6841 6857 6863 6869 6871 6883 6899 6907 6911 6917
6947 6949 6959 6961 6967 6971 6977 6983 6991 6997
7001 7013 7019 7027 7039 7043 7057 7069 7079 7103
7109 7121 7127 7129 7151 7159 7177 7187 7193 7207
7211 7213 7219 7229 7237 7243 7247 7253 7283 7297
7307 7309 7321 7331 7333 7349 7351 7369 7393 7411
7417 7433 7451 7457 7459 7477 7481 7487 7489 7499
7507 7517 7523 7529 7537 7541 7547 7549 7559 7561
7573 7577 7583 7589 7591 7603 7607 7621 7639 7643
7649 7669 7673 7681 7687 7691 7699 7703 7717 7723
7727 7741 7753 7757 7759 7789 7793 7817 7823 7829
7841 7853 7867 7873 7877 7879 7883 7901 7907 7919
88 Teora de N umeros
Captulo 3
Ecuaciones diofantinas
Una ecuacion con coecientes enteros en una o mas incognitas es llamada
diofantina (en honor a Diofanto
1
quien fue el primero en estudiar este
tipo de ecuaciones). En particular, nos interesa examinar la ecuacion
diofantina, llamada ecuacion diofantina lineal en dos incognitas,
ax +by = c , (3.1)
donde a , b y c son enteros dados, con a y b no simultaneamente
nulos. Un par de n umeros x
0
, y
0
es una solucion de la ecuacion (3.1) si
y solo si ax
0
+by
0
= c .
Ejemplo 3.1 Consideremos la ecuacion diofantina 3x+6y = 18 . Clara-
mente x
0
= 4 e y
0
= 1 es una solucion, puesto que 3 4 + 6 1 = 18 .
Tambien los pares (6, 6) y (10, 2) son soluciones. Este ejemplo
muestra que una ecuacion diofantina lineal no necesariamente posee solu-
ciones unicas.
1
Diofantus, nacio alrededor del a no 200 y fallecio alrededor del a no 284. Su libro
Aritmetica inspiro a Fermat en sus investigaciones.
89
90 Teora de N umeros
Ejemplo 3.2 Puede suceder que una ecuacion diofantina no posea ninguna
solucion en Z. Por ejemplo, la ecuacion diofantina 2x 8y = 13
no puede tener soluciones en los n umeros enteros, puesto que el lado
izquierdo de la ecuacion es siempre par y el lado derecho es impar.
Ejemplo 3.3 La ecuacion 9x + 100y = 1 tiene soluciones x, y en los
enteros.
En efecto, como mcd (9, 100) = 1 , existen enteros x, y tales que 9x+
100y = 1 (representacion de B`ezout del maximo com un divisor de dos
enteros). Por ejemplo, 9(11) +100 1 = 1) o 9 89+100(8) = 1 . De
hecho, la ecuacion diafantina 9x + 100y = 1 tiene innitas soluciones
enteras x e y como muestra el siguiente resultado.
En el siguiente teorema formulamos un criterio de solubilidad para las
ecuaciones diofantinas del tipo (3.1).
Teorema 3.1 Sean a, b, c Z. Considere la ecuacion diofantina (3.1)
ax +by = c .
(a) Si mcd (a, b) | / c , entonces no esisten soluciones enteras x, y de
(3.1).
(b) Si mcd (a, b)|c , entonces existen innitas soluciones de (3.1). Ademas,
todas ellas son de la forma
x =
b
mcd (a, b)
k +x
0
, y =
a
mcd (a, b)
k +y
0
,
donde (x
0
, y
0
) es una solucion particular y k Z.
Sergio Plaza 91
Demostracion. Consideremos la ecuacion diofantina
ax +by = c .
Caso 1. Supongamos que mcd(a, b) | / c . Si x, y son soluciones de la
ecuacion diofantina, entonces
mcd (a, b)|(ax +by) = c,
y esto es una contradiccion. Por lo tanto no existen soluciones enteras
para la ecuacion (3.1).
Caso 2. Supongamos que mcd (a, b)|c . Entonces existe alg un k Z
tal que c = kmcd (a, b) . Por otra parte existen enteros m y n tales
que
am+bn = mcd (a, b) ,
luego, amk +bnk = mcd (a, b)k = c . Por lo tanto, x = km, y = kn es
una solucion de la ecuacion (3.1).
Ahora, sea (x
0
, y
0
) una solucion particular de la ecuacion diofantina
(3.1). Entonces, denotando por d = mcd(a, b) , tenemos
a
_
b
d
k +x
0
_
+b
_

a
d
k +y
0
_
= ax
0
+bx
0
= c .
Esto prueba que x =
b
d
k + x
0
, y =
a
d
k + y
0
es una solucion de la
ecuacion (3.1) para cada k Z.
Nos resta mostrar que cada solucion tiene la forma x =
b
d
k + x
0
,
y =
a
d
k+y
0
, con x
0
, y
0
una solucion particular y k Z. Supongamos
92 Teora de N umeros
para ello que x, y es una solucion. Entonces de ax+by = c y ax
0
+by
0
=
c tenemos que
a(x x
0
) +b(y y
0
) = c c = 0 .
Luego,
a
d
(x x
0
) +
b
d
(y y
0
) = 0 ,
esto es,
a
d
(x x
0
) =
b
d
(y y
0
) .
Ahora, como
a
d
divide el lado izquierdo de la ultima igualdad, debe
dividir tambien el lado derecho. Por otra parte, como mcd
_
a
d
,
b
d
_
= 1 ,
se debe tener que
a
d
| (y y
0
) , esto se traduce en que y y
0
= k
a
d
para
alg un k Z es decir, y = y
0
+ k
a
d
. Ahora, reemplazando y y
0
en
la igualdad
a
d
(x x
0
) =
b
d
(y y
0
) nos queda
a
d
(x x
0
) =
b
d
k
a
d
,
esto es, x =
b
d
k +x
0
. Lo que completa la prueba del teorema.
Otra forma de formular el mismo teorema es la siguiente.
Teorema 3.2 Denotemos por d = mcd (a, b) . Entonces la ecuacion
diofantica (3.1) posee una solucion si y solo si d|c . Ademas, si x
0
e
y
0
es una solucion particular de la ecuacion entonces todas las otras
soluciones x e y est an dadas por
x = x
0
+t, y = y
0
t,
para t un entero arbitrario, donde b = d y a = d .
Sergio Plaza 93
Observacion. Una consecuencia inmediata de este teorema es el caso
particular en que los coecientes a y b de la ecuacion son coprimos.
En tal caso se obtiene que si x
0
e y
0
es una solucion particular de la
ecuacion, entonces todas las soluciones x e y estan dadas por
x = x
0
+bt, y = y
0
at,
para todo t Z.
Ejemplo 3.4 La ecuacion 6x + 9y = 21 tiene innitas soluciones en-
teras.
En efecto, como mcd(6, 9) = 3 y 3|21 , por la primera parte del
teorema (3.1) existen innitas soluciones para la ecuacion propuesta.
Por otra parte, una solucion particular es x
0
= 8 , y
0
= 3 , pues
6 8 + 9(3) = 48 27 = 21 , y la solucion general es dada por x =
9
3
k + 8 = 3k + 8 e y =
6
3
k 3 = 2k 3 , con k Z. Por ejemplo,
para k = 1 obtenemos la solucion x = 11 , y = 5 y para k = 2
obtenemos la solucion x = 14 e y = 7 .
Ejemplo 3.5 Estudiemos la ecuacion diofantina 172x + 20y = 1000 .
Como mcd (172, 20) = 4 y como 4 divide a 1000, el teorema (3.1)
garantiza que la ecuacion posee solucion en los n umeros enteros.
Ademas se tiene que
1000 = 500 172 + (4250) 20 .
Por lo tanto, x
0
= 500 e y
0
= 4250 es una solucion de la ecuacion,
y todas las soluciones son de la forma x = 500 +5t e y = 4250 43t ,
con t Z.
94 Teora de N umeros
Ejemplo 3.6 Sean m y n enteros positivos, con m n. Entonces
mcd (m, n)
m
_
_
m
n
_
_
es un n umero entero.
Solucion. Usaremos la ecuacion de Bezout de un modo bastante ele-
gante. Sea S el conjunto de los enteros x tales que
x
m
_
_
m
n
_
_
sea un n umero entero. Notemos que m pertenece a S , pues los
n umeros binomiales son n umeros enteros. Tambien n pertenece a S ,
ya que
n
m
_
_
m
n
_
_
=
n
m
m!
(mn)!n!
=
_
_
m1
n 1
_
_
.
Ahora, si x e y pertenecen a S , entonces para cualquier par de
enteros u y v se tiene que ux +vy tambien pertenece a S . En efecto,
tenemos que
ux +vy
m
_
_
m
n
_
_
= u
x
m
_
_
m
n
_
_
+v
y
m
_
_
m
n
_
_
que es un entero por la condicion de que x e y pertenecen a S . Como el
maximo com un divisor de m y n puede ser escrito en la forma mu+nv
para alg un par de enteros u y v , se sigue por lo anterior que mcd(m, n)
pertenece a S .
Ejemplo 3.7 Cuando Juana sea un a no mas joven que Beatriz sera
cuando Juana tenga la mitad de la edad que tendra Beatriz cuando
Juana tenga el doble de la edad de Beatriz ahora, Beatriz tendra tres
veces la edad que Juana tena cuando Beatriz tena la edad que Juana
Sergio Plaza 95
tiene ahora. Una es adolescente y las edades son en a nos completos Que
edad tienen ellas?
Solucion. Sea x la edad de Juana ahora e y, la edad de Beatriz ahora.
Representemos las edades de Juana y Beatriz en tiempo, por x
i
e y
i
.
respectivamente. Reescribamos la informacion dada, incluyendo estas
elecciones de variables.
Cuando Juana sea x
1
un a no m as joven que Beatriz sera y
2
cuando
Juana tenga x
2
la mitad de la edad y
3
que tendra Beatriz cuando Juana
tenga x
3
el doble de la edad y de Beatriz ahora, Beatriz tendra y
1
tres
veces la edad x
4
que Juana tena cuando Beatriz tena y
4
la edad x
que Juana tiene ahora.
Se tienen as las siguientes ecuaciones x
1
= y
2
1 , x
2
=
1
3
y
3
, x
3
=
2y , y
1
= 3x
4
, y
4
= x .
Sean a, b, c, d tales que x
1
= x+a , x
2
= x+b , x
3
= x+c , x
4
= x+d ;
de modo que y
1
= y + a , y
2
= y + b , y
3
= y + c , y
4
= y + d . Las
ecuaciones de anteriores transforman entonces en x + a = y + b 1 ,
x +b =
1
2
(y +c) , x +c = 2y , y +a = 3(x +d) , y +d = x.
Reordenando estas ecuaciones (y multiplicando por un factor apro-
piado), obtenemos x + a b = y 1 , x + b
1
2
c =
1
2
y ,
1
2
x +
1
2
c = y ,
3xa +3d = y , 3x3d = 3y . Sumando estas ecuaciones encontramos
que se cancelan a , b , c y d , obteniendose
8
1
2
x = 6
1
2
y ,
esto es 13y 17x = 2 . Puesto que x e y pueden solamente ser enteros,
esta ecuacion es un ejemplo de ecuacion diofantina lineal.
Un metodo para resolver tal ecuacion consiste en aplicar primero el
96 Teora de N umeros
algoritmo euclidiano para encontrar el maximo com un divisor d de 13
y 17, que es por cierto 1. Siguiendo el algoritmo en reversa, podemos
expresar d en terminos de 13 y 17 como
1 = 13 12
= 13 3 4
= 13 3(17 3)
= 4 13 3 17 .
Luego 2 = 813617 = 813+1317t1317 = 13(8+17t)17(6+13t) .
As, obtenemos la solucion x = 6+13t , y = 8+17t para alg un entero
t . Ahora, dado que una de las chicas tiene menos de 20 a nos, debe ser
t = 1 , luego x = 19 e y = 25 . Por lo tanto, Juana tiene 19 y Beatriz
25.
Ejemplo 3.8 La ecuacion diofantina 6x + 9y = 5 no tiene solucion,
pues mcd (6, 9) = 3 y 3 no divide a 5.
Ejemplo 3.9 Resuelva 3x + 3y + 5z = 10 .
Solucion. Primero factorizando el factor mcd (3, 3) = 3 de las dos
primeros coecientes de la ecuacion, y obtenemos
3
_
3
3
x +
3
3
y
_
+ 5z = 0 .
Ahora hacemos w = x +y y la ecuacion se transforma en 3w +5z =
10 . Como mcd (3, 5) = 1 y 1|10 , la ecuacion 3w + 5z = 10 tiene
innitas soluciones. Por simple impeccion vemos que w
0
= 5 y z
0
= 1
es una solucion particular de esta ultima ecuacion. Luego su solucion
general es w = 5s + 5 y z = 3s 1 .
Sergio Plaza 97
Ahora debemos encontrar x e y . Tenemos x + y = w, es decir,
x + y = 5s + 5 . Como mcd (1, 1) = 1 y 1|(5s + 5) la ecuacion tiene
innitas soluciones. Como x = 5 e y = 5s es una solucion particular,
se sigue que la solucion general es dada por x = t + 5 e y = 5s t .
Juntando lo anterior, la solucion general de la ecuacion 3x+3y+5z = 10
es
x = t + 5 , y = 5s t , z = 3s 1 ,
con t , s Z. Por ejemplo, tomando t = 2 y s = 1 tenemos la
solucion x = 7 , y = 7 y z = 2 .
Ejemplo 3.10 Un comerciante gasto cincuenta y seis mil pesos en telas,
unas a cuatro mil ciento diez, otras a tres mil novecientos setenta Cuantos
de cada cual ha comprado?
Solucion. Sean x e y el n umero de telas a $ 4.110 y 4 3.970, respec-
tivamente. Entonces, tenemos la ecuacion diofantina
4110x + 3970y = 56000 ,
esto es, 411x + 397y = 5600 . Notese que 411 397 = 14 . Luego
411 400 397 400 = 5600 .
Por lo tanto, la ecuacion diofantina debe satisfacer z = 400 397t
y = 400 + 411t , para alg un entero t . Para tener x e y ambos
positivos, se necesita que t = 1 . As x = 3 e y = 11 . De modo que el
comerciante compro 3 telas a $ 4110 cada una y 11 telas a $ 3970 cada
una.
98 Teora de N umeros
Ejemplo 3.11 Se cuenta de una chica llamada Margot que al pregun-
tarle su edad as respondio Los dos tercios de su cuadrado como un
cubo pueden ser mirados A que n umero Margot se rerio?
Solucion. Sea x la edad, y
3
el cubo. Ambos x e y deben ser enteros.
Entonces
2
3
x
2
= y
3
,
o equivalentemente 2x
2
= 3y
3
. Puesto que 2 divide el lado izquierdo,
debe entonces dividir el lado derecho. Luego 2|y , y en consecuencia 8
divide el lado derecho. Por lo tanto 2|x .
Ahora, como 3 divide al lado derecho de la ultima ecuacion, tambien
divide el lado izquierdo, es decir, 3|x . Luego 9 divide el lado izquierdo,
y en consecuencia tambien el lado derecho. Se concluye que 3|y . Por lo
tanto 81 divide el lado derecho. Luego 9|x .
As, tenemos que mcm(2, 9) = 18|x y mcm(2, 3) = 6|y .
Pongamos x = 18 e y = 6 . Sustituyendo en la ecuacion 2x
2
=
3y
3
, obtenemos
2 18
2

2
= 3 6
3

3
.
Supongamos ahora que, dado un primo p , se tiene que p
i
es la mayor
potencia de p que divide el lado izquierdo. Entonces i debe ser un
m ultiplo de 2. Ademas, siendo p
i
la potencia de p que divide el lado
derecho, i debe ser un m ultiplo de 3. Por lo tanto i es un m ultiplo
de 6. Se tiene entonces que i = 0 para cualquier primo p o i es a lo
menos 6 para alg un primo p . Si i es a lo menos 6 para alg un primo p
entonces a lo menos 93|, en cuyo caso 2
3
= 8 y x 8 18 = 144 .
De acuerdo a standards de hoy en da, Margot no sera una chica, sino
Sergio Plaza 99
una candidata al libro de records Guiness. Entonces = 1 y x = 18 .
De modo que Margot tiene 18 a nos.
Ejemplo 3.12 Pili mira por sobre el hombro de Willy mientras este
hace unos calculos. No hay nada equivocado en eso, ella declara,
444888 es en verdad uno menos que el cuadrado de 667. Pregunta
Willy, tu piensas que es el unico n umero de 6 cifras que es uno menos
que un cuadrado, tal que su segunda mitad es justo el doble de la
primera? No, hay otro, responde Pili, pero veamos si tu puedes
encontrarlo Esta Pili en lo correcto?
Solucion. Representemos el n umero en la forma 1002(100x+10y +z) .
Entonces
1002(100x + 10y +z) = n
2
1
para cierto entero n. Luego 1002|(n 1)(n + 1) y 10
5
n
2
< 10
6
.
Luego 316 < n < 1000 . Como 1002 = 2 3 167 , cada uno de esos
n umeros 2, 3 y 167 divide a lo menos uno de entre n 1 y n + 1 .
Puesto que 2|((n + 1) (n 1) , se sigue que 2 debe dividir ambos
n1 y n+1. Luego 2 167 = 334 divide a uno de entre n1 y n+1 .
Por lo tanto n = 334k + 1 o n = 334k 1 para alg un entero k tal
que 316 < n < 1000 . Luego 3 debe dividir a 333, 667, 335 y 669 . Pero
3|(n 1) o 3|(n + 1) , de modo que 3|n, as es que n no es igual ni a
333 ni a 669. Si n = 667 , entonces n
2
2 1 = 444888 . Si n = 335 ,
entonces ( n 1)(n +1) = 334 336 = 1002 112 = 112224 . Por lo tanto
el otro n umero anunciado por Pili es 112224, que es uno menos que el
cuadrado de 335.
Ejemplo 3.13 Willy vacio el monedero de Pili sobre la mesa Tienes
aqu 56 monedas de $ 100, $ 50 y $ 10 Sabes cuanto tienes en total?
100 Teora de N umeros
S, me faltan $ 30 para tener mil pesos. Determine cuanto dinero tiene
Pili en su monedero.
Solucion. Sean x, y, z las cantidades de monedas de $ 100, $ 50 y $ 10
pesos, respectivamente. Entonces 10x + 5y +z = 97 y x +y +z = 56 .
Restando estas ecuaciones se elimina z , y obtenemos 9x + 4y = 41 .
Observe que 9 14 2 = 1 , luego 9 414 82 = 41 . Por lo tanto, para
resolver la ecuacion diofantina 9x+4y = 41 necesitamos que x = 414t
e y = 82 + 9t , para alg un entero t . Ahora, como x > 0 e y > 0 , se
debe tener que t = 10 . Por lo tanto x = 1 e y = 8 , de tal manera que
z = 56 8 1 = 47 . En consecuencia, el monedero de Pili contiene una
moneda de $ 100, ocho de $ 50 y cuarenta y siete de monedas de $ 10.
Ejemplo 3.14 T u y Bella fotograadas la semana pasada, eh? dijo
Jenny, mirando el retrato sobre el escritorio. Muy bonita Que edad
tiene ella ahora? Mim penso un momento. Si t u divides tu edad
por la ma y restas el resultado de tu edad dividida por la de ella, tu
obtendras un septimo de tu edad Que edad tena Mimi?
Solucion. Sea m la edad de Mimi, b la dedad de Bella y j la edad de
Jenny. Entonces
j
b

j
m
=
1
7
j .
Como j = 0 , se tiene que
1
b

1
m
=
1
7
, esto es, m b =
mb
7
. Luego
7|m o 7|b . Ademas, como m > 0 y b > 0 se sigue que
1
7
mb > 0 ,
luego m > b . Supongamos que b = 7k para alg un entero k . Entonces
b = mkm = (1 k)m.
Como b > 0 , se tiene que k = 0 , en cuyo caso b = m, lo que es
imposible, de donde conclumos que 7|b , luego m = 7k , para alg un
entero k . Por lo tanto, 7k = b +kb = (k +1)b . Ahora m > 0 , luego k
Sergio Plaza 101
y en consecuencia k + 1 son positivos. De modo que b =
7k
k+1
. Por lo
tanto (k + 1)|7k ; como mcd (k + 1, k) = 1 , se tiene que (k + 1)|7 , de
modo que k + 1 es igual a 1 o 7. Como k + 1 no puede ser 1, porque
entonces k = 0 y b = m, lo que es imposible. Luego k + 1 = 7 , esto
es, k = b = 6 y m = 7k = 42 . De tal manera que Mimi tiene 42 a nos
y Bella tiene 6 a nos. No hay suciente informacion para determinar la
edad de Jenny. Ella podra tener 42 a nos u 84 a nosun m ultiplo de 6 y
42.
Ejemplo 3.15 Dado un n umero de cuatro dgitos, intercambie el primero
con el segundo y el tercero con el cuarto. As usted obtendr a cuatro veces
el n umero original Cual es el n umero?
Solucion. Sea el n umero 1000w+100x+10y +z . Entonces 4(1000w+
100x + 10y + z) = 1000x + 100w + 10z + y . Reordenando, tenemos
3900w 200x = 2z 13y , esto es, 1300w 200x = 2z 13y .
Ahor, 0 y 9 y 0 z 9 , luego 20139 2z13y 29130 ,
esto es, 117 2z 3y 18 .
Ahora 100|(1300w 200x) . Luego 100|(2z 13y) , esto implica que
2z 13y = 0 o 2z 13y = 100 .
Supongamos que 2z 13y = 0 . Entonces 13w 2x = 0 , luego
w = 2t , x = 13t para alg un entero t tal que 0 < w 9 ( w es el dgito
de mas a la izquierda, luego debe ser necesariamente distinto de cero) y
0 x 9 , lo que es imposible. Por lo tanto, 2z 13y = 100 , luego se
tiene que 1300w 200x = 100 , simplicando nos queda 13w 2x =
1 .
Observemoa que 13 1 2 7 = 1 , luego w = 1 + 13t y x = 7 + 2t
para alg un entero t . La unica posibilidad es t = 0 , luego w = 1 y
x = 7 . Notemos que 13 8 2 2 = 100 , esto es, 2 2 13 8 = 100 .
102 Teora de N umeros
Por lo tanto, para satisfacer la ecuacion diofantina 2z 13y = 100 ,
debemos tener y = 8 + 2s y z = 2 + 13s , para alg un entero s , tal que
0 y 9 y 0 z 9 . La unica posibilidad es s = 0 .
En consecuencia, el n umero pedido es 1782.
Ejemplo 3.16 Cambia mi ultimo dgito por y , mi primero por cinco.
Entonces el cuadrado de un tercio de un noveno de mi t u encontraras.
En unos segundos tu veras, de eso estoy seguro. Claramente mis tres
dgitos Cuales crees tu que son?
Solucion. Sea n el n umero y sea y su segundo dgito. Entonces
_
1
3

1
9
n
_
2
= 509 + 10y .
El lado izquierdo debe ser en efecto el cuadrado de un entero, por
consiguiente el lado derecho debe ser 529, esto es, y = 2 . De manera
que n = 621 , cuyo segundo dgito de hecho es 2.
3.1 Problemas
Problema 3.1 Para las siguientes ecuaciones diofantinas, determine to-
das sus soluciones en enteros, y liste las soluciones en los enteros positivos
1. 18x + 5y = 48 ,
2. 54x + 21y = 906,
3. 123x + 360y = 99,
4. 158x 57y = 7 .
Sergio Plaza 103
Problema 3.2 Pruebe que si d = mcd (a, b) , entonces mcd
_
a
d
,
b
d
_
=
1 .
Problema 3.1 Para todo a, b, k, m Z se tiene
1. a y b son coprimos si y solo si existen u, v Z tales que au+bv =
1 ;
2. mcd (a, b) = mcd (b, a) = mcd (|a|, |b|) ;
3. mcd (ka, kb) = |k|mcd (a, b) ;
4. mcd (a, 0) = |a| y mcd(a, 1) = 1 ;
5. Si mcd(a, m) = mcd(b, m) = 1 , entonces mcd(ab, m) = 1 ;
6. Si mcd(a, b) = 1 , entonces mcd (a
k
, b

) = 1 para todo k, N.
Problema 3.2 Generalice el primer problema, es decir, si a
1
, a
2
, . . . , a
k
,
con k 2 , son enteros positivos tales que mcd (a
1
, a
2
, . . . , a
k
) = d . De-
muestre que
1. Existen enteros x
1
, x
2
, . . . , x
k
tales que x
1
a
1
+x
2
a
2
+ +x
k
a
k
=
d .
2. Si d = 1 , muestre que existe un entero positivo m
0
tal que todo
entero m m
0
puede ser escrito en la forma y
1
a
1
+y
2
a
2
+ +
y
n
a
n
, con y
1
, y
2
, . . . , y
n
0 .
Problema 3.3 Considere los n umeros que aparecen en la prueba de
Euclides sobre la innidad de n umeros primos, es decir,
104 Teora de N umeros
2 + 1 = 3
2 3 + 1 = 7
2 3 7 + 1 = 31
2 3 7 31 + 1 = 211
2 3 7 31 211 + 1 = 2311
.
.
.
son todos coprimos a pares son primos?
Problema 3.4 A la fecha son conocidos 3 primos de la forma n
n
+ 1 ,
ellos son 1
1
+ 1 = 2 , 2
2
+ 1 = 5 , 4
4
+ 1 = 257 . Mostrar que si existen
otros entonces ellos deben tener mas de 300.000 dgitos. No resuelto a
la fecha. Existen otros?
Problema 3.5 Hay una cantidad innita de primos de la forma n!+1 ?
No resuelto a la fecha. Por ejemplo, 2! +1 = 3 , 3! +1 = 7 , 4! +1 = 25 ,
no es primo, 5! +1 = 121 , no es primo. Encuentre algunos otros primos
de la forma indicada.
Problema 3.6 La siguiente sucesion de n umeros es llamada repunid
(repite unidad)
11, 111, 1111, 11111, . . . , 11 . . . 1, . . .
Para n jo, cuantos primos hay en esta sucesion entre 11 y 11 . . . 1 ?
Es posible probar que en la sucesion repunid hay innitos n umeros
primos?
Sergio Plaza 105
Problema 3.7 Encuentre todos los triangulos pitagoricos cuyo permetro
es igual a su area.
Problema 3.8 Pruebe que la longitud del radio del crculo inscrito en
un triangulo pitagorico es siempre un n umero entero.
Problema 3.9 Muestre que en cualquier triangulo pitagorico, uno de
los lados es divisible por 3, uno divisible por 4, y uno divisible por 5.
Problema 3.10 La ecuacion 3
n
+ 4
n
= 5
n
, n N tiene como unica
solucion n = 2.
Problema 3.11 Muestre que la unica terna pitagorica formada por en-
teros consecutivos es (3, 4, 5)
Problema 3.12 Es posible construir una caja cuyos lados y diagonales
son todos n umeros enteros? No resuelto a la fecha.
Problema 3.13 Examine los siguientes ejemplos y haga su propia con-
jetura.
1 8 + 1 = 9 = 3
2
3 8 + 1 = 25 = 5
2
6 8 + 1 = 49 = 7
2
10 8 + 1 = 81 = 9
2
15 8 + 1 = 121 = 11
2
21 8 + 1 = 169 = 13
2
Note que los n umeros 1, 3, 6, 10, 15, 21, . . . no son otros que los
n umeros triangulares.
106 Teora de N umeros
Problema 3.14 Pruebe geometricamente que la suma de dos n umeros
triangulares consecutivos es un cuadrado. Por ejemplo, 1 +3 = 4 = 2
2
,
3 + 6 = 9 = 3
2
, 6 + 10 = 16 = 4
2
, 10 + 15 = 25 = 5
2
, 15 + 21 = 36 =
6
2
, . . .
Problema 3.15 Considere lo siguiente
5 = 1
2
+ 2
2
13 = 2
2
+ 3
2
41 = 4
2
+ 5
2
Estos son n umeros primos que son suma de cuadrados de dos enteros
consecutivos.
Determine otros primos con esta propiedad. No se sabe a la fecha
si la cantidad de n umeros primos que satisfacen lo anterior es nita o
innita.
Problema 3.16 Encuentre todos los n umeros primos p, q tales que pq
divide a (5
p
2
p
)(5
q
2
q
) .
Problema 3.17 Sean p, q n umeros primos. Si q divide 2
p
+3
p
, pruebe
que q > p o q = 5 .
Problema 3.18 Demuestre que la fraccion
21n+4
14n+3
es irreducible para
todo n umero natural n.
Problema 3.19 Sean x, y, z enteros tales que x
3
+y
3
z
3
es m ultiplo
de 7. Pruebe que uno de esos n umeros es m ultiplo de 7.
Problema 3.20 Determine el n umero de cuadrados perfectos que hay
entre 40.000 y 640.000 que son m ultiplos simultaneamente de 3, 4 y 5.
Sergio Plaza 107
Problema 3.21 Pruebe que un entero de la forma 4
a
(8b + 7) , con a
y b enteros no negativos, no puede ser una suma de tres cuadrados.
Problema 3.22 Demostrar que un n umero natural es divisible po 2
n
,
para n = 1, 2, 3, . . . si y solo si el n umero formado por sus ultimos n
dgitos es divisible por 2
n
.
Problema 3.23 Cual es el tama no del subconjunto mayor S de {1, 2, . . . , 50}
tal que ning un para de elementos distintos de S tenga una suma divis-
ible por 7?
Problema 3.24 Demostrar que un entero que consiste de 3n dgitos
identicos es divisible por 3
n
.
Problema 3.25 Si 32ab427 es un m ultiplo de 99 , hallar los dgitos a
y b .
Problema 3.26 1. Encontrar todos los n umeros enteros positivos n
para los cuales 2
n
1 es divisible por 7.
2. Demostrar que no existen n umeros enteros positivos n para los
cuales 2
n
+ 1 es divisible por 7.
Problema 3.27 Sea k un entero positivo tal que
k(k+1)
3
es un cuadrado
perfecto. Demuestre que
k
3
y k + 1 . son cuadrados perfectos.
Problema 3.28 Un n umero triangular es un n umero entero positivo
de la forma 1 + 2 + + n, con n N. Por ejemplo, 3, 6, 10, 15,
21, 28, 36, y 45, son n umeros triangulares. Estos n umeros pueden ser
ordenados como se muestra a seguir, y es claro desde esa conguracion
a que se debe su curioso nombre
108 Teora de N umeros
1
2 3
1
2 3
4 5 6
1
2 3
4 5 6
7 8 9 10
Sergio Plaza 109
.
.
.
Tabla de n umeros triangulares
Pruebe que los dgitos 2, 4, 7, y 9 no pueden ser el ultimo dgito de
un n umero triangular.
3.2 Curiosidades Numericas
1. Veamos la siguiente curiosidad
1
3
=
1 + 3
5 + 7
=
1 + 3 + 5
7 + 9 + 11
=
1 + 3 + 5 + 7
9 + 11 + 13 + 15
=
Esta fue descubierta por Galileo (1615)
2. Considere la tabla
1 =1
3 5 =8
7 9 11 =27
13 15 17 19 =64
21 23 25 27 29 =125
.
.
.
Observe que sumando los elementos de cada la se obtiene el cubo
de un n umero natural Puede probar esto?
110 Teora de N umeros
3. Conjetura de los n umeros capic uas. Dado un n umero natu-
ral, sume a este sus dgitos escritos en forma reversa. Repita el
proceso con el resultado. Continue de esa forma hasta obtener
un n umero capic ua, es decir, uno que se lee igual de derecha a
izquierda y viceversa. Por ejemplo, 95+59=154, 154+451=605,
605+506=1111 que es capic ua, 72+27=99 que nos resulto capic ua
de inmediato. La armacion es que este procedimiento siempre nos
conduce a un n umero capic ua. Una demostracion de esta hecho
no es conocida a la fecha.
4. Primos Gemelos. Los primos gemelos son pares de primos de la
forma n, n + 2. Por ejemplo, 3,5, 5, 7, 11, 13, 17, 19, 29, 31,. . .
Conjetura. Existen innitos primos gemelos.
5. Considere la tabla
1+2 = 3
4+5+6 = 7+8
9+10+11+12 = 13+14+15
16+17+18+19+20 = 21+22+23+24
6. Considere la siguiente distribucion de los n umeros del 1 al 24,
Descubra la regla general de la formacion de esa distribucion y si
es posible haga una prueba.
7. N umeros de Fermat. El nesimo n umero de Fermat, F
n
, es
denido por F
n
= 2
2
n
+ 1 .
Conjetura. S olo un n umero nito de n umeros de Fermat son
primos. Una conjetura mas fuerte es que los unicos n umeros de
Sergio Plaza 111
Fermat primos son F
0
= 3 , F
1
= 5 , F
2
= 17 , F
3
= 257 , y
F
4
= 65537 .
8. Conjetura de Goldbach. Cada entero par mayor que 4 puede
ser escrito como la suma de dos primos impares.
Por ejemplo, 6 = 3 + 3 , 8 = 3 + 5 , 10 = 3 + 7 , 12 = 5 + 7 ,
14 = 3 + 11 = 7 + 7 , 16 = 3 + 13 , 18 = 5 + 13 , 20 = 3 + 17 ,
22 = 3+19 , 24 = 5+19 , 26 = 3+23 , 28 = 5+23 , 30 = 7+23 , . . .
9. Conjetura de Catalan. Las unicas torres de primos consecutivos
son 8 = 2
3
y 9 = 3
2
.
Langevin y Tijdeman mostraron que cualquier contraejemplo debe
ser menor que e
e
e
e
730
, luego en teora debera ser vericable en un
computador.
10. Problema de Callatz o problema 3x + 1 . Consideremos la
funcion f : N N dada por
f(x) =
_

_
x
2
si x es par
3x + 1 si x es impar
Conjetura. para cada entero positivo n existe un entero d tal
que f
d
(n) = 1 , donde f
d
= f f es la composicion de f
consigo misma d veces.
Por ejemplo,
112 Teora de N umeros
n = 2, 2
f
1, d = 1
n = 3, 3
f
10
f
5
f
16
f
8
f
4
f
2
f
1, d = 7
n = 4, 4
f
2
f
1d = 2
n = 5, 5
f
16
f
8
f
4
f
2
f
1, d = 5
Captulo 4
Congruencias
Calculo con congruencias ocurren a diario en nuestro vivir cotidiano.
Los principales ejemplos son la aritmetica del reloj sumamos las ho-
ras sobre un reloj tomando congruencias modulo 12 (o modulo 24, si
reemplazamos xpm por x + 12 ).
Denici on 4.1 Sean m N, y a, b Z. Decimos que a es con-
gruente con b modulo m si a b es m ultiplo de m. Si esto sucede lo
denotaremos por el smbolo a b (mod m) , es decir, a b (mod m)
si y solo si m|(a b) .
Ejemplo 4.1 11 1 (mod 5) , puesto que 11 1 = 10 es m ultiplo de
5 .
Ejemplo 4.2 23 2 (mod 7) , puesto que 23 2 = 21 es m ultiplo de
7, es decir, 7|(23 2) .
Es inmediato desde la denicion de congruencia que ella es reeja,
es decir, para todo n umero entero a se tiene que a a (mod m) .
113
114 Teora de N umeros
Ademas claro que es simetrica, puesto que a b (mod m) si y solo si
b a (mod m) .
A continuacion daremos algunas propiedades de las congruencias. Sus
demostraciones son directas de las propiedades de la divisibilidad y las
dejamos de ejercicio para el lector.
Teorema 4.1 (Propiedades basicas de la congruencia)
1. Si a 0 (mod m) , entonces m|a .
2. Si a b (mod m) , entonces a y b dejan el mismo resto en la
division por m.
3. Si a b (mod m) y b c (mod m) , entonces a c (mod m)
(transitividad).
4. Si a b (mod m) , entonces (a +c) (b +c) (mod m) y (a c)
(b c) (mod m).
5. Si a b (mod m) , entonces b
k
a
k
(mod m) para todo entero
positivo k .
6. Si p es primo y a b 0 (mod p) , entonces a 0 (mod p) o
b 0 (mod p) .
Es importante observar que para un n umero jo no nulo, m, y para un
entero z cualquiera, se debe satisfacer una y solo una de las siguientes
congruencias
z 0 (mod m), z 1 (mod m), z 2 (mod m), . . . , z (m1) (mod m).
Sergio Plaza 115
Este hecho es consecuencia directa del Algoritmo de la Division y es
otra forma de decir que cualquier n umero entero, al dividirlo por m,
deja resto 0 , 1 , 2 . . . , ( m1) . En particular, si elegimos m = 2 esto
nos asegura que todo n umero entero es par si z 0 (mod 2) o impar si
z 1 (mod 2) .
Ejemplo 4.3 Consideremos la congruencia modulo 3, es decir, m = 3 .
Entonces cualquier n umero entero a es de la forma a = 3k , de la forma
a = 3k + 1 o de la forma a = 3k + 2 . Notemos que a 2 (mod 3) es
lo mismo que decir a 1 (mod 3) .
Ejemplo 4.4 Pruebe que existen innitos primos de la forma 3n 1 .
Solucion. Por ejemplo, n = 1 , 3n 1 = 2 primo, n = 2 , 3n 1 = 5
primo, n = 3 , 3n 1 = 8 compuesto, n = 4 , 3n 1 = 11 primo,
n = 5 , 3n 1 = 14 compuesto, n = 6 , 3n 1 = 17 primo.
Vemos entonces que existen primos de la forma 3n 1 . Supongamos
que existe solo una cantidad nita de primos de la forma 3n 1 , y
sean estos p
1
, p
2
, . . . , p
r
. Denamos el n umero N = 3p
1
p
2
p
r
1 ,
entonces p
i
no divide a N para i = 1, 2, . . . r . En efecto, tenemos que
N +1 = 3 p
1
p
2
p
r
, luego p
i
|(N +1) y si p
i
|N , entonces p
i
divide a
la diferencia (N +1) N = 1 , es decir, p
i
|1 , esto es una contradiccion.
Por otra parte, tenemos que N > 1 , luego N tiene un factor primo.
Armamos que N tiene un factor primo p , con p 2 (mod 3) . En
efecto, como N no es divisible por 3, cada factor primo p de N debe
ser tal que p 1 (mod p) o p 2 (mod p) . Si todos los factores primos
de N fueran congruente a 1 (mod 3) , tendramos que N 1 (mod 3) ,
y es una contradiccion, por lo tanto N debe tener un factor primo p ,
con p 2 (mod 3) , y como p = p
i
, este es un primo que no esta en
nuestra lista.
116 Teora de N umeros
Luego, para cualquier lista de primos p 2 (mod 3) , podemos mostrar
que debe existir un primo congruente a 2 (mod 3) , el cual no esta en
nuestra lista, lo cual muestra que existen innitos primos de la forma
3n 1 .
Ejemplo 4.5 Sea p un primo impar tal que p = x
2
+2y
2
. Pruebe que
p 1 (mod 8) o p 3 (mod 8) .
Solucion. Si x es par, entonces p = x
2
+2y
2
es par, contradiciendo el
hecho que p es un impar.
Luego, x debe ser impar, luego x 1 (mod 8) , pues x tiene la forma
x = 2n 1 , as x
2
= 4n
2
2n + 1 . Ahora si n es par, n = 2a , luego
x
2
= 4(2a)
2
2(2a) + 1 = 16a
2
4a + 1 y si a es par a = 2 nos queda
x
2
= 16 4 t
2
4 2t + 1 1 (mod 8)
La prueba para las restantes alternativas es analoga.
Si y tambien es impar, entonces p = x
2
+2y
2
1+2 3 (mod 8) . Si
y es par entonces y 0, 2, 4, 6 (mod 8) , luego y
2
0, 4 (mod 8) . En
cada caso, tenemos que 2y
2
0 (mod 8) , por lo tanto p = x
2
+ 2y
2

1 + 0 = 1 (mod 8) .
Ejemplo 4.6 Determinar para cuales n umeros primos p se cumple que
2
p
+p
2
es primo.
Solucion. Notemos que p = 2 y p = 3 producen los n umeros 8 y 17,
compuesto en el primer caso y primo en el segundo. Basta considerar
entonces primos p > 2 .
Consideremos congruencia modulo 3. Sabemos que p debe satisfacer
una y solo una de las congruencias siguientes
Sergio Plaza 117
p 0 (mod 3), p 1 (mod 3), p 1 (mod 3) .
Claramente, el primer caso solo se puede dar si p = 3 , puesto que de
otra manera p sera un n umero compuesto.
Aplicando las propiedades de congruencia en cualquiera de los dos
casos restantes se obtiene que p
2
1 (mod 3) .
Por otro lado, como 2 1 (mod 3) se obtiene 2
p
(1)
p
(mod 3) .
Ademas p es impar, luego 2
p
1 (mod 3).
En resumen, 2
p
1 (mod 3) y p
2
1 (mod 3) . Aplicando las
propiedades de las congruencias con respecto a la suma se obtiene -
nalmente que 2
p
+ p
2
0 (mod 3) . Pero entonces 2
p
+ p
2
es siempre
divisible por 3 si p > 3 . Luego el unico primo p que resuelve el pro-
blema es p = 3 .
Ejemplo 4.7 (Reglas de divisibilidad). Veamos como se establecen
las reglas clasicas de divisibilidad usando el concepto de congruencia.
1. Divisibilidad por 2. Cada n umero natural n puede ser escrito
en la forma 10q +r , donde r es el resto de la division de n por
10, es decir, r es el ultimo dgito de n. Ahora, como 2|10 se
sigue 2|n si y solo si 2|r . En otras palabras, 2|n si y solo si n
termina en 0, 2, 4, 6 u 8.
2. Divisibilidad por 3. Supongamos que la representacion deci-
mal de n es dada por n = a
k
a
k1
a
0
, es decir, n = a
k
10
k
+
a
k1
10
k1
+ + a
1
10 + a
0
. Ahora, como 10 1 (mod 3) , se
sigue que 10
m
1 (mod 3) para todo n umero natural m. Por lo
tanto,
118 Teora de N umeros
n = a
k
10
k
+a
k1
10
k1
+ +a
1
10 +a
0
a
k
+a
k1
+ +a
1
+a
0
(mod 3).
Luego, 3|n si y solo si 3|(a
k
+ a
k1
+ + a
1
+ a
0
) . En otras
palabra 3 divide a un n umero natural n si y solo si divide a la
suma de sus dgitos.
3. Divisibilidad por 4. Cada n umero natural n puede ser escrito
en la forma n = 100q + r , donde r es el resto de la division de
n por 100. Ahora, como 4|100 se tiene que 4|n si y solo si 4|r ,
donde r consiste de al menos dos dgitos de n.
4. Divisibilidad por 5. Cada n umero natural n puede ser escrito
en la forma n = 10q + r , donde r es el resto de la division de n
por 10. Ahora, como 5|10 se tiene que 5|n si y solo si 5|r , donde
r es el ultimo dgito de n. En otras palabras 5|n si y solo si n
termina en 0 o en 5.
5. Divisibilidad por 6. Como el mnimo com un m ultiplo de 2 y 3
es 6, se sigue que para vericar si n es divisible por 6 vericamos
si es divisible por 2 y por 3.
6. Divisibilidad por 8. Cada n umero natural n puede ser escrito
en la forma n = 1000q +r , donde r es el resto de la division de n
por 1000. Ahora, como 8|1000 se tiene que 8|n si y solo si 8|r ,
donde r consiste de al menos tres dgitos de n.
7. Divisibilidad por 9. Supongamos que la representacion deci-
mal de n es dada por n = a
k
a
k1
a
0
, es decir, n = a
k
10
k
+
Sergio Plaza 119
a
k1
10
k1
+ +a
1
10+a
0
. Ahora como 10 1 (mod 9) , se sigue
que 10
m
1 (mod 9) para todo n umero natural m. Por lo tanto,
n = a
k
10
k
+a
k1
10
k1
+ +a
1
10 +a
0
a
k
+a
k1
+ +a
1
+a
0
(mod 9).
Luego, 9|n si y solo si 9|(a
k
+ a
k1
+ + a
1
+ a
0
) . En otras
palabra 9 divide a un n umero natural n si y solo si divide a la
suma de sus dgitos.
Ejemplo 4.8 Dado un n umero A de tres dgitos, se lo multiplica por
143. Ahora, multiplicando por 7 las tres ultimas cifras del n umero as
obtenido, se obtiene un n umero cuyas ultimas tres cifras coinciden con
A Cual es el n umero A?
Solucion. Sean A = abc = 100a + 10b + c y 143 = 100 + 4 10 + 3 ,
escritos en notacion decimal. Entonces
143A = 10000a + 1000 4a + 100 3a + 1000b + 100 4b + 10 3b +
100c + 10 4c + 3c
= a 10
4
+ (4a +b)10
3
+ (3a + 4b +c)10
2
+ (3b + 4c)10 + 3c .
Ahora multiplicamos por 7 y luego reducimos modulo 1000, obte-
niendo
7 143A (21a + 28b + 7c) 10
2
+ (21b + 28c) 10 + 21c (mod 1000)
= (2 10 + 1)a10
2
+ (2 10 + 8)b 10
2
+ 7c 10
2
+
(2 10 + 1)b 10 + (2 10 + 8)c 10 + 2 10c +c
a 10
2
+ 8b 10
2
+ 7c 10
2
+ 2b 10
2
+b 10 + 2c 10
2
+
120 Teora de N umeros
8c 10 + 2c 10 +c (mod 1000)
= (a + 8b 7c + 2b + 2c) 10
2
+ (b + 8 + 2c) 10 +c
= (a + 10b + 9c) 10
2
+ (b + 10c) 10 +c
= a 10
2
+b 10
3
+ 9c 10
2
+b 10 +c 10
2
+c
a 10
2
+b 10 +c (mod 1000)
= A.
Esto concluye el problema.
Ejemplo 4.9 Los enteros de dos dgitos desde el 19 hasta el 92 se
escriben consecutivamente para obteniendose el n umero entero N =
192021222324 . . . 89909192 Cual es la potencia mayor de 3 que divide
a este n umero?
Solucion. Por la regla de los nueves este n umero es divisible por 9 si y
solo si
19 + 20 + 21 + + 92 = 37
2
3
lo es. Por lo tanto, el n umero es divisible por 3 pero no por 9.
Ejemplo 4.10 Encuentre el resto de la division de 6
1987
por 37.
Solucion. Tenemos que 6
2
1 (mod 37) . Luego,
6
1987
= 6 6
1986
= 6 (6
2
)
993
6 (1)
993
(mod 37)
6 (mod 37)
31 (mod 37) .
Sergio Plaza 121
Por lo tanto, el resto de la divisi on de 61987 por 37 es 31.
Ejemplo 4.11 Encuentre el resto cuando 12233 455679 + 87653
3
es
dividido por 4.
Solucion. Tenemos que 12233 = 12200+32+1 1 (mod 4) , 455679 =
455600 + 76 + 3 3 (mod 4) , 87653 = 87600 + 52 + 1 1 (mod 4) .
Luego, 12233 455679 + 87653
3
1 3 + 1 0 (mod 4) . Por lo tanto
12233 45679 + 87653
3
es divisible por 4.
Ejemplo 4.12 Para cada n umero entero positivo n el n umero 3
2n+1
+
2
n+2
es divisible por 7.
Solucion. Observemos que 3
2n+1
3 9
n
3 2
n
(mod 7) y 2
n+2
=
4 2
n
, luego 3
2n+1
+ 2
n+2
= (3 + 4) 2
n
= 7 2
n
0 (mod 7) , lo que
termina la prueba.
Ejemplo 4.13 Pruebe el siguiente resultado debido a Euler. El n umero
de Fermat F
5
es divisible por 641 , es decir, 641|(2
32
+ 1) .
Solucion. Observemos que 641 = 2
7
5 + 1 = 2
4
+ 5
4
. Luego, 2
7
5
1 (mod 641) y 5
4
2
4
(mod 641) . Ahora, 2
7
5 1 (mod 641)
nos da que 5
4
2
28
= (5 2
7
)
4
(1)
4
1 (mod 641) . Esta ultima
congruencia y 5
4
2
4
(mod 641) nos da 2
4
2
28
1 (mod 641) , es
decir, 2
32
+ 1 0 (mod 641) , dicho de otra forma 641|(2
32
+ 1) = F
5
.
Ejemplo 4.14 Pruebe o encuentre un contraejemplo para las siguientes
armaciones.
1. Si a b (mod d) y d|n, entonces a b (mod n) .
2. Si a b (mod n) y d|n entonces a b (mod d) .
122 Teora de N umeros
Solucion. La primera armacion es falsa, por ejemplo, tomemos a =
0 , b = 1 , d = 1 y n = 2 . Entonces d|n y a b (mod d) , pero
a = b (mod n) .
La segunda es verdadera. En efecto, supongamos que a b (mod n)
y que d|n, entonces a b n m y n = cd , con c, m Z. Luego,
a b = n m = d(cm) , de donde a b (mod d) .
4.1 N umero de pasos en el algoritmo de la di-
vision
Si deseamos calcular mcd (3073531, 304313) por el algoritmo de division,
vemos que tendremos que hacer muchos pasos y nos gustara saber de
antemano mas o menos cuantos pasos deberamos hacer para obtener el
resultado.
Teorema 4.2 Sean a y b enteros positivos. El algoritmo de Euclides
para calcular mcd(a, b) toma a lo mas 2([log
2
(b)] + 1) pasos (es decir,
divisiones).
Demostracion. Sea s = [log
2
(b)] + 1 . Sea n el n umero de pasos que
debemos dar en el algoritmo de la division de Euclides para obtener
mcd (a, b) = r
n
.
Queremos probar que n 25 . Supongamos que n 25+1 . Primero
mostramos que r
j+2
< r
j
/2 para j {1, 2, . . . , n2} . Si r
j+1
r
j
/2 ,
entonces r
j+2
< r
j+1
r
j
/2 . Si r
j+1
> r
j
/2 , entonces r
j
= r
j+1
q
j+1
+
r
j+2
, donde q
j+1
= 1 . Luego, ,en este caso, r
j+2
= r
j
r
j+1
< r
j
/2 .
Por lo tanto, en cualquier caso, tenemos que r
j+2
< r
j
/2 . Deducimos
entonces que
Sergio Plaza 123
1 r
n
<
r
n2
2
<
r
n4
4
< <
r
n2
5
2
5

r
1
2
5
=
b
2
5
Por lo tanto, s < log
2
(b) . Esto contradice el hecho que s = [log
2
(b) +
1] > log
2
(b) .
4.2 Peque no Teorema de Fermat
Enunciamos a seguir dos teoremas acerca de los n umeros primos que
son utiles de recordar. Sus demostraciones son mas complicadas y las
omitiremos por ahora. En el captulo 3?? daremos una sencilla de-
mostracion del primero de ellos.
Teorema 4.3 (Peque no Teorema de Fermat) Dados dos entero a y
p , con a > 0 y p primo. Entonces
a
p
a (mod p) .
Otra version de este teorema es la siguiente
Teorema 4.4 (Peque no Teorema de Fermat, otra version ). Si n es
un n umero entero y p es un n umero primo entonces p|(n
p
n) .
Corolario 4.1 Dados n umeros naturales k y n, y dado un primo p ,
si k 1 (mod p 1) . Entonces p|(n
k
n) .
El peque no teorema de Fermat puede ser usado para determinar si un
n umero entero dado N es compuesto como sigue:
124 Teora de N umeros
Paso 1. Chequear N para factores primos pequeos (este paso no es
necesario, pero es razonable de hacer).
Paso 2. Escriba N en base 2, digamos N =
k

j=0

j
2
j
, con
j
{0, 1}
para cada j y k = [log N/ log 2] + 1 .
Paso 3. Calcule 2
2
j
(mod N ).
Paso 4. Calcule m {0, 1, . . . , N 1}
tal que
m =
k
j=0
2

j
2
j
2
N
(mod N)
Paso 5. Si m = 2 , entonces N es compuesto. Si no el algoritmo no
concluye nada.
Comentario. El algoritmo funciona bien para establecer que la mayorade
los n umeros son compuestos, es decir, para la mayora de los n umeros
compuestos, m = 2 . Si m = 2 , entones podemos chequear si 3
N
3
(mod N ). Note que el algoritmo toma sobre el orden de log(N) pa-
sos, luego el algoritmo es un algoritmo de tiempo que es un polinomial
(este corre en tiempo que es un polinomio en la longitud de la entrada
(input)). No existe algoritmo de tiempo polinomial que determina con-
clusivamente cuando un entero arbitrario es compuesto.
4.3 Test de Primalidad Probabilstico
De acuerdo al peque no teorema de Fermat, si p es primo y 1 a p1 ,
entonces
a
p1
1 (mod p)
Sergio Plaza 125
La recproca tambien vale en el siguiente sentido.
Teorema 4.5 Si m 2 y para todo entero a tal que 1 a m 1
se tiene a
m1
1 (mod m) . Entonces m debe ser primo.
Demostracion. Si la hipotesis vale, entonces para todo a con 1 a
m 1 , se tiene que a posee un inverso modulo m, es decir, a
m2
es
un inverso para a modulo m. Ademas, tenemos que 1 a m 1
y mcd(a, m) = 1 . Ahora si m no es primo, entonces m = a b , con
1 < a < m y 1 < b < m, y tenemos que mcd(a, m) = a > 1 , lo cual es
una contradiccion. Luego m debe ser primo.
Observacion. Sea m > 0 . Si ab 1 (mod m) , entonces mcd (a, m) =
1 y mcd (a, m) = 1 y mcd (b, m) = 1 . En efecto, si ab 1 (mod m) ,
entonces m(ab 1) . Luego, ab 1 = m t para alg un t . De esto, se
sigue que
ab +m(t) = 1
de donde obtenemos que mcd (a, m) = 1 y mcd (b, m) = 1 .
Si usamos el teorema anterior para chequear si p es primo, debemos
chequear que a
p1
1 (mod p) para a = 1, 2, , p 1 . Esto es un
trabajo no trivial (por su extension). Supongamos que tenemos 2
m1

1 (mod m) para alg un m > 2 Debe ser m primo? El menor entero


compuesto m que satisface 2
m1
1 (mod m) es m = 341 = 11 31 .
En otras palabras, 2
3411
(mod 341) , pero 341 no es primo. La
moral del asunto, es que a un cuando 2
m1
1 (mod m) , el n umero m
no es necesariamente primo. Por otra parte, tomando m = 63(= 21 3) ,
tenemos 2
6
= 64 1 (mod 63) .
126 Teora de N umeros
Luego, elevando a la decima potencia la congruencia 2
6
1 (mod 63) ,
obtenemos
2
60
1 (mod 63)
Multiplicando por 2
2
esta ultima congruencia, obtenemos
2
62
4 (mod 63)
y como 4 = 1 (mod 63) se sigue que
2
62
= 1 (mod 63) .
Por lo tanto 63 no es primo. Note que para saber esto no necesitamos
descomponer 63.
Reiteremos lo que acabamos de concluir. Si 2
m1
no es congruente a
1 (mod m) , entonces m no es primo.
Aceptemos los siguientes hechos. Existen 455052511 primos impares
p 10
10
, todos los cuales satisfacen 2
p1
1 (mod p) . Existen solo
14884 n umeros compuestos 2 < m 10
10
que satisfacen 2
m1

1 (mod m) .
Luego, si 2 < m 10
10
y m satisface 2
m1
1 (mod m) , entonces
la probabilidad de m ser primo es
455052511
455052511 + 14884
0.999672928
Sergio Plaza 127
En otras palabras, si encontramos que 2
m1
1 (mod m) , entonces
es seguro que m es primo, al menos cuando m 10
10
.
Este tipo de algoritmo es mucho mas eciente si la cantidad de primos
que estamos considerando es menor, por ejemplo se sabe que el n umero
de primos menor o igual que 10
6
es 78498, y que la cantidad de n umeros
compuestos m 10
6
tales que 2
m1
1 (mod m) es 245.
La cantidad de n umeros compuestos m 10
6
tales que 2
m1

1 (mod m) y 3
m1
1 (mod m) es 66.
La cantidad de n umeros compuestos m 10
6
tales que a
m1

1 (mod m) para a {2, 3, 5, 7, 11, 13, 17, 19, 31, 37, 41} es 0.
De lo anterior, tenemos el siguiente resultado.
Si m 10
6
y a
m1
1 (mod m) para a {2, 3, 5, 7, 11, 13, 17, 31, 37, 41}
entonces m es primo.
Si m > 10
6
y a
m1
1 (mod m) para a {2, 3, 5, 7, 11, 13, 17, 19, 31, 37, 41}
es altamente probable pero no es seguro, que m es primo.
Ejercicios.
Problema 4.1 Verique que 3
90
1 (mod 91) , pero 91 = 7 13 no es
primo.
Problema 4.2 Para m = 1105 = 221 5 (no primo) se verica que
2
m1
1 (mod m) y m
m1
1 (mod m) . Compruebe esta armacion.
4.4 Ejemplos
Ejemplo 4.15 Usaremos este teorema para probar que 3 divide a (a+
b)
3
a
3
b
3
. Tenemos que (a+b)
3
(a+b) (mod 3) , a
3
a (mod 3) ,
y b
3
b (mod 3) . Luego,
(a +b)
3
a
3
b
3
a +b a b (mod 3)
128 Teora de N umeros
0 (mod 3) ,
es decir, 3|((a +b)
3
a
3
b
3
) .
Ejemplo 4.16 Determinar todos los enteros positivos n para los cuales
la expresion 2
n
+ 1 es divisible por 3.
Solucion. Consideremos congruencia modulo 3.
Tenemos 2 1 (mod 3) , luego 2
n
(1)
n
(mod 3) y por lo tanto
2
n
+1 [(1)
n
+ 1] (mod 3) . Luego, si n es impar, 2
n
+1 0 (mod 3) ,
es decir, 2
n
+ 1 es divisible por 3 para todo n impar. Ademas, si n
es par se obtiene que 2
n
+ 1 2 (mod 3) . Luego 2
n
+ 1 no es nunca
divisible por 3 si n es par.
Ejemplo 4.17 Calcule 50
250
(mod 83) .
Solucion. Usando un programa de calculo numerico obtenemos que
50
250
= 55271478752604445602472651921922557255142402332392200864151
70220907898754023953317101764802222264464998750268125535
78470207686332597244588393792241731716785579919815063
4765625000000000000000000000000000000000000000000000
0000000000000000000000000000000000000000000000000000
0000000000000000000000000000000000000000000000000000
0000000000000000000000000000000000000000000000000000
0000000000000000000000000000000000000000000000000
Ahora usaremos el peque no teorema de Fermat, es decir, si p es un
n umero primo y p no divide a a entonces a
p1
1 (mod p) . Tenemos
que 83 no divide a 50, luego tomando p = 17 , por el peque no teorema
de Fermat, 50
82
1 (mod 17) . Ahora, como 3 82 = 246 tenemos
50
250
= 50
246
50
4
=
_
50
82
_
3
2500
2
1
3
10
2
= 100 = 17 (mod 83).
Sergio Plaza 129
Ejemplo 4.18 Resolvamos la ecuacion en congruencia 16x 25 (mod 41) .
Solucion. La idea sera multiplicar ambos lados de la ecuacion 16x
25 (mod 41) por el recproco de 16 (mod 41) . Una manera simple de
encontrar tal n umero es usar el teorema de Fermat, para ello notemos
que 16
40
1 (mod 41) . Luego,
16
39
16x 16
39
25 (mod 41)
nos da que
x 16
39
25 (mod 41)
que es una solucion buscada.
Ejemplo 4.19 No existe un entero n > 1 que divide a 3
n
2
n
.
Solucion. Supongamos que existe alg un entero n > 1 que divide a
3
n
2
n
, es decir, 3
n
2
n
0 (mod n) . Es facil vericar que 2 ni 3
dividen a n. Consideremos el menor factor primo p de n, el cual existe
pues n > 1 . Usando ahora el peque no teorema de Fermat, tenemos
??. Como 3
n
2
n
(mod n) se sigue que 3
mp
2
mp
(mod p) , de donde
3
m
2
m
(mod p) . Sea d = mcd (m, p 1) , tenemos que en particular
d divide a n. Por lo tanto, como p es el menor factor primo de n
deducimos que d = 1 . Luego existen entero positivos x e y tales que
mx = (p1)y +1 . Usando nuevamente el peque no teorema de Fermat,
junto con el hecho que 3
m
2
m
(mod p) , obtenemos 3 3
(p1)y+1
=
3
mx
2
mx
= 2
(p1)y+1
2 (mod p) , lo cual es imposible.
Ejemplo 4.20 Muestre que 7|(2222
5555
+ 5555
2222
) .
Solucion. Por el peque no Teorema de Fermat, para cualquier n umero
natural n se tiene que n
7
n (mod 7) . Luego, para n umeros naturales
q y r se tiene que
130 Teora de N umeros
n
7q+r
(n
7
)
q
n
r
(mod 7)
n
q
n
r
(mod 7)
= n
q+r
(mod 7) .
A continuacion usaremos este resultado varias veces. Pero antes, ob-
serve que
2222 3 (mod 7) y 5555 3 (mod 7)
Luego
2222
5555
+ 5555
2222
3
5555
+ (3)
2222
(mod 7)
3
7937+4
+ (3)
3177+3
(mod 7)
3
113+6
+ (3)
45+5
(mod 7)
3
17+0
+ (3)
7+1
(mod 7)
3
2+3
+ (3)
1+1
(mod 7)
3
2
(3
3
+ 1) (mod 7)
28 3
2
(mod 7)
0 (mod 7) .
De donde concluimos que 7 divide a 2222
5555
+ 5555
2222
.
Solucion alternativa. Una manera mas astuta de usar el peque no
teorema de Fermat es la siguiente. Puesto que n
7
n = n(n
6
1) ,
del peque no teorema de Fermat sigue que si n es un n umero natural y
Sergio Plaza 131
n 0 (mod 7) , entonces n
6
1 (mod 7) . Por lo tanto, para n umeros
naturales n, q y r , si n 0 (mod 7) , entonces
n
6q+r
(n
6
)
q
n
r
(mod 7)
1
q
n
r
(mod 7)
n
r
(mod 7 .
En otras palabras, si n 0 (mod 7) , entonces podemos reducir la
potencia de n modulo 6. Luego
2222
5555
+ 5555
2222
3
55555
+ (3)
2222
(mod 7)
3
5
+ (3)
2
(mod 7)
3
2
(3
3
+ 1) (mod 7)
28 3
2
(mod 7)
0 (mod 7)
Luego 7|(2222
5555
+ 5555
2222
) .
Ejemplo 4.21 Muestre que existen innitos enteros positivos n tales
que 2
n
+ 27 es divisible por 7.
Solucion. Para n = 3 , se tiene que 2
3
+ 27 = 35 es divisible por 7.
Observemos que 2
1
2 , 2
2
4 , 2
3
1 , 2
4
2 , 2
5
4 , y 2
6
1
(mod 7) . Luego 2
3k
1 (mod 7) para todo entero positivo k . Por lo
tanto 2
3k
+ 27 = 1 + 27 0 (mod 7) para todo entero positivo k .
Ejemplo 4.22 Para todo k = 0, 1, 2, . . . al dividir 2
k
5 por 7 nunca
se obtiene resto igual a 1.
132 Teora de N umeros
Solucion. Tenemos 2
1
2 , 2
2
4 , 2
3
1 (mod 7) y este ciclo se
repite. Luego 2
k
5 puede dejar resto 3, 4 o 6 en la division por 7.
Ejemplo 4.23 Muestre que n
31
n es divisible por 56786730, para
todos n umero natural n.
Solucion. La descomposicion del n umero dado como producto de pri-
mos es 56786730 = 2 3 5 7 11 13 31 61 .
Aplicaremos el corolario del peque no teorema de Fermat.
Usaremos este resultado varias veces para k = 61 y p igual a cada
uno de los primos 2, 3, 5, 7, 11, 13, 31 y 61. Tenemos
61 1 (mod 1) . Luego 2|(n
61
n) .
61 1 (mod 2) . Luego 3|(n
61
n) .
61 1 (mod 4) . Luego 5|(n
61
n) .
61 1 (mod 6) . Luego 7|(n
61
n) .
61 1 (mod 10) . Luego 11|(n
61
n) .
61 1 (mod 12) . Luego 13|(n
61
n) .
61 1 (mod 30) . Luego 31|(n
61
n) .
61 1 (mod 61) . Luego 61|(n
61
n) .
Por lo tanto mcm(2, 3, 5, 7, 11, 13, 31, 61) = 2 3 5 7 11 13 31 61
divide n
61
n para todos los n umeros naturales n.
Ejemplo 4.24 Cual es el dgito nal de ((((((((((7
7
)
7
)
7
)
7
)
7
)
7
)
7
)
7
)
7
)
7
?
(7 ocurre 10 veces como potencia)
Solucion. El dgito nal de un n umero escrito en forma decimal es
su resto modulo 10 ( mod 10) . Ahora, tenemos 7
2
1 (mod 10) .
Luego 7
7
= (7
2
)
3
7 7 (mod 10) , y (7
7
)
7
(7)
7
(7)
7

(7) = 7 (mod 10) . Procediendo de esta forma, vemos que ((7


7
)
7
)
7

7 (mod 10) , en general, ( (((7


7
)
7
)
7
) )
7
7 (mod 10) , donde el
Sergio Plaza 133
signo es + si tenemos una cantidad par de 7 que aparecen como poten-
cias, y el signo es si una cantidad impar de 7 aparece como potencias
en la formula. Como en nuestro caso 7 aparece 10 veces como potencia
concluimos que el dgito nal en la expresion es 7.
Ejemplo 4.25 Consideremos la sucesion de n umeros 3, 15, 24, 48, . . . ,
que consiste de los m ultiplos de 3 que son iguales a un cuadrado perfecto
menos 1 Cual es el resto cuando el termino 1994 de esta sucesion se
divide por 1000?
Solucion. Deseamos 3|(n
2
1) , es decir, 3|(n 1)(n + 1) . Como 3 es
primo, obtenemos que n = 3k + 1 o n = 3k 1 , con k = 1, 2, 3, . . .
La sucesion 3k + 1 , con k = 1, 2, 3, . . . produce los terminos n
2
1 =
(3k + 1)
2
1 que son los terminos en los lugares pares de la sucesion
3, 15, 24, 48, . . . La sucesion 3k 1 , con k = 1, 2, . . . produce los
terminos n
2
1 = (3k1)
2
1 que son los terminos en los lugares impares
de la sucesion 3, 15, 24, 48, . . . Debemos encontrar el termino 997 de la
sucesion 3k + 1 , con k = 1, 2, . . . . Este termino es (3 997 + 1)
2
1
(3 (3) + 1)
2
1 64 1 63 (mod 1000) . Por lo tanto, el resto
buscado es 63.
Ejemplo 4.26 Cual es el ultimo dgito de 7
7
7
7
7
7
7
?
Solucion. Llamamos a una expresion como la anterior una torre de
sietes. En nuestro caso la torre de sietes tiene 7 sietes. Ahora como
7
4
= (7
2
)
2
(1)
2
= 1 (mod 10) , de donde (mod 10) tenemos que
7
k
=
_

_
1k 0 (mod 4)
7k 1 (mod 4)
1k 2 (mod 4)
7k 3 (mod 4)
134 Teora de N umeros
donde k es un n umero natural. Por lo tanto, para determinar el ultimo
dgito de la torre de 7 sietes, necesitamos determinar que una torre de
6 sietes es congruente mod 4 . Ahora, como 7 1 (mod 4) , tenemos
(mod 4)
7
m

_
_
_
1 si m es par
1 si m es impar,
donde m es un n umero natural. Una torre de 5 sietes es ciertamente
impar. Luego, una torre de 6 sietes es congruente a 1 (mod 4) ( 1
3 (mod 4)) . Por lo tanto, una torre de 7 sietes es congruente a 7
modulo 10 (y 7 3 (mod 10)) . As que una torre de 7 sietes debe
terminar en un 3.
Ejemplo 4.27 Calcule los dos ultimos dgitos de 7
73
.
Solucion. Tenemos
7
2
= 49
7
4
= 2401 1 (mod 100)
Luego 7
72
= (7
4
)
18
1 (mod 100) , por lo tanto 7
73
7 (mod 100 ),
y los dos ultimos dgitos de 7
73
son 01.
El mismo problema para 3
73
es un poco mas interesante;
3
2
= 9
3
4
= 81
3
8
= 6561 61 (mod 100)
3
16
61
2
= 3721 21 (mod 100)
3
3
2 21
2
= 441 41 (mod 100)
3
64
41
2
= 1681 81 (mod 100)
Sergio Plaza 135
luego 3
73
= 3
64
3
8
3 81 61 3 23 (mod 100) y los dos ultimos
dgitos de 3
73
son 23.
Ejemplo 4.28 Cuando 4444
4444
es escrito en notacion decimal, la suma
de sus dgitos es A. Sea B la suma de los dgitos de A. Encuentre la
suma de los dgitos de B.
Solucion. Primero notemos que como 4444 < 10000 = 10
4
se tiene
que 4444
4444
< 10
44444
= 10
17776
, por lo tanto la cantidad de dgitos de
4444
4444
no puede ser mayor que 17776, y en consecuencia la suma de los
dgitos de 4444
4444
, que es A, no puede ser mayor que 177769 = 159984
(pues cada dgito es a lo mas un 9). De entre los n umeros menores que
159984, el n umero con la mayor suma de sus dgitos es 99999. Luego
B, no puede ser mayor que 45. De entre los n umero naturales menores
o iguales que 45, el n umero con mayor suma de sus dgitos es 39. Por lo
tanto la suma de los dgitos de B no puede ser mayor que 12.
Ahora usaremos el siguiente hecho ya demostrado sobre la disibilidad
por 9
para cualquier n umero natural N , se tiene que
N suma de los dgitos de N (mod 9) .
Usando esto vemos que 4444
4444
es congruente a la suma de los dgitos
de A (mod 9) . Usando otra vez ese resultado, vemos que A es congru-
ente a la suma de los dgitos de B (mod 9) . Usando el resultado una
vez mas tenemos que B es congruente a la suma de sus dgitos modulo
9, esto es
4444
4444
A (mod 9)
B (mod 9)
(suma de los dgitos de B) (mod 9) .
136 Teora de N umeros
Ahora determinaremos a que n umero es congruente 4444
4444
(mod 9) .
4444
4444
(4 + 4 + 4 + 4)
4444
(mod 9)
16
4444
(mod 9)
(2)
4444
(mod 9)
(2)
31481+1
(mod 9)
((2)
3
)
1481
(2) (mod 9)
(8)
1481
(2) (mod 9)
1
1481
(2) (mod 9)
1 (2) (mod 9)
7 (mod 9) .
Por lo tanto, juntando los hechos anteriores vemos que
(suma de los dgitos de B) 7 (mod 9)
y la suma de los dgitos de B es un n umero natural menor o igual que
12. Luego la suma de los dgitos de B es igual a 7.
Ejemplo 4.29 Calcule (m 1)! (mod m) para 2 m 9 . Formule
una conjetura.
Solucion. Tenemos 2! 2 (mod 3) , 3! 2 (mod 4) , 4! 4 (mod 5) , . . . ,
8! 0 (mod 9) .
Conjetura. (m1)! 1 (mod m) si y solo si m es primo. Ademas,
(m1)! 0 (mod m) para todo m compuesto, excepto m = 4 .
Sergio Plaza 137
Ejemplo 4.30 Hallar los cuadrados de los n umeros enteros positivos
(mod 13) .
Solucion. Observemos primero que solo necesitamos calcular los cuadra-
dos de los enteros hasta 6, porque r
2
(13 r)
2
(mod 13) . Calcu-
lando los cuadrados de los enteros no negativos hasta el 6, obtenemos
0
2
1
2
1 , 2
2
4 , 3
2
9 , 4
2
3 , 5
2
12 , 6
2
10 (mod 13) .
Por lo tanto los cuadrados (mod 13) son 0, 1, 4, 9, 3, 12 y 10.
Ejemplo 4.31 La ecuacion x
2
+ 5y
2
= 2 no tiene soluciones enteras.
Solucion. Si x
2
= 2 + 5y
2
, entonces x
2
2 (mod 5) . Pero 2 no es un
cuadrado (mod 5) .
Ejemplo 4.32 Calcular el resto de la division de 3
100
por 101.
Solucion. Por ejemplo, usando un computador, obtenemos
3
100
= 515377520732011331036461129765621272702107522001
y
3
100
101
= 0.5102747730 10
46
no es muy revelador para solucionar nuestro problema. Por otra parte,
si usamos aritmetica modular, es decir, congruencias, os calculos los
podemos hacer facilmente.
Escribiendo 100 como suma de potencias de 2, esto es 100 = 64 +
32 + 4 = 2
6
+ 2
5
+ 2
2
, tenemos 3
100
= 3
64
3
32
3
4
. Ahora, 3
4
= 81
20 (mod 101) , es decir, 3
4
20 (mod 101) . Elevando al cuadrado
ambos lados de la congruencia, tenemos 3
8
20
2
4 (mod 101) .
Repitiendo el proceso nos queda 3
16
4
2
= 16 (mod 101) , 332 162
47 (mod 101) . Por lo tanto 3
100
36433234 (13)(47)(20)
(13)(31) 1 (mod 101) . Simple y lo pudimos hacer con calculos
sencillos.
138 Teora de N umeros
Ejemplo 4.33 Existen enteros positivos x, y tal que x
3
= 2
y
+ 15 ?
Solucion. No. Los cubos (mod 7) son 0, 1, and 6. Ahora bien, cada
potencia de 2 es congruente a 1, 2, o 4 (mod 7) . As pues, 2
y
+ 15
2, 3, 5 (mod 7) . Esto es imposible.
El siguiente teorema caracteriza los n umeros primos.
Teorema 4.6 (Wilson) Sea a un n umeros entero mayor que 1 . En-
tonces a es un primo si y solo si (a 1)! 1 (mod a) .
Demostracion. Ponerla.
4.4.1 Ejercicios
Problema 4.3 Pruebe que entre cualesquiera diez enteros positivos y
consecutivos al menos uno es coprimo con el producto de los otros.
Problema 4.4 Pruebe que para cada r 1 , existen innitos primos
p con p 1 (mod 2
r
) .
Problema 4.5 Calcule 2
100000
(mod 77) de dos formas: primero, us-
ando el teorema de Euler, y segundo, usando el teorema de Fermat y el
teorema chino de los restos.
Problema 4.6 Encuentre los dos ultimos dgitos de 9
9
9
9
(= 9
(9
9
9
)
) .
Problema 4.7 Sea y un n umero de 3 dgitos, el cual cuando es elevado
a la potencia 89 da un n umero cuyos 3 ultimos dgitos es 247. Encuentre
y .
Problema 4.8 El n umero [(44 +

1996
100
)] es par o impar?
Sergio Plaza 139
Problema 4.9 Un n umero de 1996 dgitos comienza con 6. Cualquier
n umero formado por dos dgitos consecutivos de tal n umero es divisible
por 17 o 23 Cual es el ultimo dgito del n umero?
Problema 4.10 En cuantos ceros termina la expansion decimal de
2003!?
Problema 4.11
Problema 4.12
Problema 4.13
Problema 4.14
Problema 4.15
Problema 4.16
Problema 4.17
Problema 4.18
4.5 Factorizacion
Teorema 4.7 Sea n un entero impar. Entonces existe una correspon-
dencia unoauno
{factorizacion} {expresion de n como la diferencia de dos cuadrados} .
140 Teora de N umeros
Demostracion. Si escribimos n = ab , entonces como n es impar se
debe tener que a y b son impares. Luego, a +b y a b son n umeros
pares, por lo tanto
a+b
2
y
ab
2
son n umeros enteros. Ahora,
n =
_
a +b
2
_
2

_
a b
2
_
2
expresa a n como diferencia de dos cuadrados de n umeros enteros.
Recprocamente, supongamos que n es escrito como la diferencia de
dos cuadrados de n umeros enteros, es decir, n = s
2
t
2
, donde s y t
son n umeros enteros. Tenemos entonces que
n = (s t)(s +t)
es una factorizacion de n.
Ejemplo 4.34 Factoricemos 4819. La idea es tratar de escribir 4819 de
la forma 4819 = s
2
t
2
, de donde t
2
= s
2
4819 . Hacemos crecer el
valor de s de modo a obtener un cuadrado perfecto. Como

4819
69.4 y como 4819 = s
2
t
2
, se debe tener que s es mayor o igual que
70 (entero inmediatamente mayor o igual que 69.4). Por otra parte la
factorizacion con el mayor factor posible es 4819 = 1 4819 , usando
la prueba del teorema anterior, procedemos con s de la forma s =
4819+1
2
= 2410 . Luego, solo necesitamos tratar con 70 s 2410 .
Primero tratamos con, 70
2
4819 = 4900 4819 = 81 = 9
2
. Por lo
tanto, tomando s = 70 y t = 9 obtenemos s + t = 79 , s t = 61 y
79 61 = 4819 .
Sergio Plaza 141
4.6 Teorema de Fermat de los dos cuadrados
Un teorema bien conocido primero establecido por Girard, y proba-
blemente primero probado por Fermat (la primera prueba conocida es
dedebida a Euler) concerniente a primos que son suma de dos cuadros,
tales como 5 = 1
2
+ 2
2
y 29 = 2
2
+ 5
2
. La siguiente caracterizacion de
tales primos es una consecuencia simple de la nocion de congruencia, la
recproca es tambien verdadera, pero mucho mas dcil de probar.
Teorema 4.8 Si un n umero primo p es la suma de dos cuadrados de
enteros, entonces p 1 (mod 4) o p = 2 .
Demostracion. Dado un n umero entero n, se tiene n 0 (mod 4) ,
n 1 (mod 4) , n 2 (mod 4) y n 3 (mod 4) . Luego para n
2
se
tiene n
2
0 (mod 4) o n
2
1 (mod 4) .
Supongamos ahora que p = a
2
+ b
2
. Como a
2
, b
2
0, 1 (mod 4) ,
vemos que a
2
+ b
2
debe ser congruente modulo 4 a 0 = 0 + 0 , 1 =
1 + 0 = 0 + 1 o 2 = 1 + 1 , esto es p 0, 1, 2 (mod 4) . Como ning un
primo es congruente a 0 (mod 4) y como 2 es el unico primo congruente
a 2 (mod 4) , se sigue que p = 2 o p 1 (mod 4) . Lo que completa la
prueba
Para la recproca, necesitamos conocer cuando 1 es un cuadrado
(mod p) para p primo. Por ejemplo 1 no es un cuadrado mod 3, 7,
u 11 , y 2
2
1 (mod 5) , 5
2
1 (mod 13) . Tenemos el siguiente
resultado general.
Teorema 4.9 Sea p un primo impar; entonces la congruencia a
2

1 (modp) tiene una solucion si y solo si p 1 (mod 4) .


Para la prueba necesitamos de algunos resultados auxiliares.
142 Teora de N umeros
Teorema 4.10 (Teorema de Wilson). Para p > 1 , se tiene (p1)!
1 (mod p) si y solo si p es primo.
Note que el Teorema de Wilson nos da un test de primalidad; desafor-
tunadamente, la unica manera conocida para calcular (n 1)! es va
n 2 multiplicaciones.
Teorema 4.11 Sea p un primo impar y sea a =
_
p 1
2
_
! , entonces
a
2
(1)
(p+1)/2
(mod p) . En particular, a 1 (mod p) si p
3 (mod 4) , y a
2
1 (mod p) si p 1 (mod 4) .
Demostracion. Del Teorema de Wilson (p 1)! 1 (mod p) ; si en
el producto (p1)! reemplazamos los elementos
p + 1
2
,
p + 3
2
, . . . , p1
por sus negativos
p + 1
2

p 1
2
,
p + 3
2

p 3
2
, . . . , (p 1)
1 (mod p) , entonces hemos introducido exactamente
p 1
2
factores
1 ; luego (p 1)! (1)
(p1)/2
a
2
(mod p) con a =
_
p 1
2
_
! .
Prueba del teorema 4.9. Si p 1 (mod 4) , entonces hemos cons-
truido una solucion a la congruencia a
2
= 1 (mod p) . Supongamos
recprocamente que esta congruencia es soluble. Elevando ambos lados a
la potencia
p 1
2
nos da que 1 a
p1
(1)
(p1)/2
(mod p) , y como
1 = 1 (mod p) para p primo impar, se debe tener que (1)
p1/2
= 1 ,
luego p 1 (mod 4) .
El siguiente resultado es debido a Birkho, redescubierto por Aubry,
y posteriormente conocido como Teorema de Thue.
Teorema 4.12 (BirkhoAubry-Thue). Dado un entero a no divis-
ible por p , entonces existen x, y Z con 0 < |x|, |y| <

p tales que
ay x (mod p) .
Sergio Plaza 143
Demostracion. Sea f el menor entero mayor que

p , consideremos
u + av (mod p) , con 0 u, v < f . Hay f
2
> p de tales expresiones,
pero solo p posibles resultados, luego deden existir u, u

, v y v

tales
que u + av u

+ av

(mod p) . Sea x = u u

, y v

v , entonces
x = ay (mod p) y ademas, f < x, y < f .
Ahora podemos probar el siguiente resultado.
Teorema 4.13 (GirardFermatEuler) Cada primo p 1 (mod 4) es
una suma de dos cuadrados enteros.
Demostracion. Como p 1 (mod 4) , existe a Z tal que a
2

1 (mod 4) . Por el Teorema 4.12, existen enteros x e y tales que


ay x (mod p) y 0 < x, y <

p . Elevando al cuadrado esta ultima
congruencia obtenemos y
2
x
2
(mod p) , esto es, x
2
+y
2
0 (mod p) .
Como 0 < x
2
, y
2
< p , se sigue que 0 < x
2
+y
2
< 2p , y como x
2
+y
2
es
divisible por p , se debe tener que x
2
+y
2
= p , como queramos probar.
Corolario 4.2 Si p es un primo, entonces

p es irracional.
Demostracion. Para p = 2 ya hicimos la prueba.
Supongamos que p > 2 es primo y que

p es racional, entonces
p = r
2
/s
2
, con r, s N. Podemos asumir, sin perdida de generalidad,
que r y s son coprimos (si no lo son simplique primero). Luego
ps
2
= r
2
. Luego p|r
2
, y como p es primo se sigue que p|r , digamos
r = pt . De esto se sigue que ps
2
= p
2
t
2
, y entonces s
2
= pt
2
, lo cual
signica que p|s
2
, y por lo tanto p|s , pues p es primo. Esto es una
contradiccion pues mcd (r, s) = 1
144 Teora de N umeros
Teorema 4.14 (Lucas). Sean m, n enteros no negativos y sea p un
primo. Sean
m = m
k
p
k
+m
k1
p
k1
+ +m
1
p +m
0
y
n = n
k
p
k
+n
k1
p
k1
+ +n
1
p +n
0
las expansiones en base p de m y n, respectivamente. Entonces
_
m
n
_
=
_
m
k
n
k
__
m
k1
n
k1
_

_
m
1
n
1
__
m
0
n
0
_
(mod p) .
Demostracion. Tenemos la siguiente identidad
(1 +x)
p
k
= 1 +x
p
k
(mod p)
cuya prueba es inmediata. De esto se sigue que
(1 +x)
m
= (1 +x)
m
k
p
k
+m
k1
p
k1
++m
1
p+m
0
= (1 +x)
m
k
p
k
(1 +x)
m
k1
p
k1
(1 +x)
m
1
p
(1 +x)
m
0
(1 +x
p
k
)
m
k
(1 +x
p
k1
)
m
k1
(1 +x
p
)
m
1
(1 +x)
m
0
(mod p) .
Por expansion en base p , los coecientes de x
n
en ambos lados es
_
m
n
_
=
_
m
k
n
k
__
m
k1
n
k1
_

_
m
1
n
1
__
m
0
n
0
_
(mod p) .
Corolario 4.3 Dado un entero no negativo n, sea A(n) el n umero
de factores de 2 en n! , y sea B(n) el n umero de 1 en la expansion
binaria de n. Entonces el n umero de entradas impares en la nesima
la del triangulo de Pascal, o equivalentemente el n umero de coecientes
impares en la expansion de (1+x)
n
es 2B(n) . Ademas, A(n)+B(n) =
n para todo n N.
Sergio Plaza 145
4.6.1 Orden de un Elemento
Sii mcd (a, m) = 1 , entonces existe un entero positivo n tal que a
n

1 (mod m) . Denotamos por ord


m
(a) , al menor entero positivo n para
el cual se tiene a
n
1 (mod m) .
Teorema 4.15 Si mcd (a, m) = 1 , entonces a
n
1 (mod m) si y solo
si ord
m
(a)|n . Ademas, a
n
0
a
n
1
(mod m) si y solo si ord
m
(a)|(n
0

n
1
) .
Demostracion. Sea d = ord
m
(a) . Es claro que si d|n entonces a
n

1 (mod m) . Por el algoritmo de la division, existen enteros q y r tales


que n = qd+r , con 0 r < d . Luego a
n
(a
d
)
q
a
r
a
r
1 (mod m) .
Pero r < d , luego r = 0 y por lo tanto d|n.
La prueba de la parte restante del teorema es inmediata.
Observacion. En particular, por el teorema de Euler, se tiene que
ord
m
(a)|(m) .
Ejemplo 4.35 El orden de 2 (mod 101) es 100.
En efecto, sea d = ord
101
(2) . Entonces d|(101) , esto es, d|100 .
Ahora, si d < 100 entonces d divide a 100/2 o a 100/5 , esto es, d
pierde al menos un factor primo. Sin embargo,
2
50
1024
5
14
5
196 196 14
(6)(6) 14
1 (mod 101)
y
2
20
1024
2
14
2
6 (mod 101)
146 Teora de N umeros
Por lo tanto d = 100 .
Ejemplo 4.36 Si p es un primo, entonces cada divisor primo de 2
p
1
es mayor que p .
En efecto, sea q un primo tal que q|(2
p
1) . Entonces 2
p
1 (mod q) ,
luego ord
q
(2)|p . Pero ord
q
(2) = 1 , por lo tanto ord
q
(2) = p . Ahora,
por el peque no teorema de Fermat, ord
q
(2)|(q 1) , de donde p q 1 ,
en consecuencia q > p . De hecho, para p > 2 , q debe ser de la forma
2kp + 1 . De lo anterior, ord
q
(2)|(q 1) , es decir, p|(q 1) , lo cual
implica que q = mp + 1 . Como q debe ser impar, m debe ser par.
Ejemplo 4.37 Sea p un primo que es coprimo con 10, y sea n un
entero con 0 < n < p . Sea d = ord
p
(10) . Entonces
a) La longitud del periodo de la expansion decimal de n/p es d .
b) Si d es par, entonces el perodo de la expansion decimal de n/p
cuya suma es 10
d/2
1 ?? Falta algo. Por ejemplo, 1/7 =
0.142857 , luego d = 6 y 142 + 857 = 999 = 10
3
1 .
En efecto,
a) Sea m la longitud del perodo de la expansion decimal de n/p , y
sea n/p = 0.a
1
a
2
a
m
. Entonces 10
m
np = a
1
a
2
a
m
a
1
a
2
a
m
,
de donde
(10
m
1)n
p
= a
1
a
2
a
m
es un n umero entero. Como mcd (n, p) = 1 se tiene que p debe
dividir a 10
m
1 , luego d|m. Recprocamente, p|(10
d
1) , luego
(10
d
1)n/p es un entero, con a lo mas d dgitos. Si dividimos
Sergio Plaza 147
este entero por 10
d
1 , entonces obtenemos un n umero racional,
cuya expansion decimal tiene a lo mas d dgitos. Por lo tanto,
m = d .
a) Sea d = 2k . Si n/p = 0.a
1
a
2
a
k
a
k+1
.
Ahora, p divide a 10
d
1 = 10
2k
1 = (10
k
1)(10
k
+ 1) . Sin
embargo, p no puede dividir a 10
k
1 , pues el orden de 10 es 2k ,
luego p|(10
k
+ 1) . De esto,
10
k
n
p
= a
1
a
2
a
k
a
k+1
a
2k
,
de donde
(10
k
+ 1)n
p
= a
1
a
2
a
k
+ 0.a
1
a
k
+ 0.a
k+1
a
2k
es un entero. Esto puede ocurrir si y solo si a
1
a
2
a
k
+a
k+1
a
2k
es un n umero que consiste solo de nueves, y luego igual a 10
k
1 .
4.7 Races Primitivas
Denici on 4.2 Si el orden de un elemento a modulo m es (m) , de-
cimos que a es una raz primitiva modulo m.
Demostremos antes que nada el siguiente resultado.
Lema 4.1 2
23
n1
1 + 3
n
(mod 3
n
+ 1) , para todo n 1 .
Demostracion. Claramente el resultado es verdadero para n = 1 .
Asumamos que es verdadero para alg un n = k . Entonces 2
23
k1
=
1 + 3
k
+ 3
k+1
m para alg un entero m, luego 2
23
k
= 1 + 3
k+1
+ 3
k+2
M
para alg un entero M (obtenido elevando al cubo).
Por lo tanto, 2
23
k
1 + 3
k+1
(mod 3
k+2
) . Por induccion la prueba
del lema esta completa.
148 Teora de N umeros
Corolario 4.4 Si 2
n
1 (mod 3
k
) , entonces 3
k1
|n.
Demostracion. La congruencia 2
n
1 (mod 3
k
) implica que 2
2n

1 (mod 3
k
) , luego (3
k
)|2n , de donde 3
k1
|n.
Ejemplo 4.38 2 es una raz primitiva mod 3
n
para todo n 1 .
En efecto, es claro que la armacion es verdadera para n = 1 . Su-
pongamos, por induccion, que el resultado vale para n = k , esto es,
2
(3
k
)
2
23
k1
1 (mod 3
k
) .
Sea d = ord
3
k+1 (2). Entonces 2
d
1 (mod 3
k+1
) , luego 2
d
1 o
d|2 3
k
. De esto deducimos que d es 2 3
k1
o bien 2 3
k
.
Tenemos 2
23
k1
1+3
k
= 1 (mod 3
k+1
) , luego el orden de 2 modulo
3
k+1
es 2 3
k
, y otra vez por induccion el resultado se sigue.
Teorema 4.16 Si m tiene una raz primitiva, entonces tiene ((m))
races primitivas distintas (mod m) .
Teorema 4.17 Un entero positivo m tiene una raz primitiva si y solo
si m es uno de los n umeros siguientes 2, 4, p
k
o 2p
k
, donde p es un
primo impar.
Teorema 4.18 Si g es una raz primitiva de m, entonces g
m

1 (mod m) si y solo si (m)|n . Ademas, g


n
0
g
n
1
(mod m) si y
solo si (m)|(n
0
n
1
) .
Teorema 4.19 Si g es una raz primitiva de m, entonces las poten-
cias 1, g, g
2
, . . . , g
(m)1
representan cada entero coprimo a m uni-
camente modulo m. En particular, si m > 2 , entoncecs g
(m)/2

1 (mod m) .
Sergio Plaza 149
Demostracion. Es claro que cada potencia g
i
es coprimo con m, y
existen (m) enteros coprimos con m. Ahora, si g
i
g
j
(mod m) ,
entonces g
ij
1 (mod m) y tenemos en este caso que (m)|(i j) .
Luego cada una de las potencias son distintas modulo m. Luego, cada
entero coprimo con m es alguna potencia g
i
modulo m.
Ademas, existe un unico i , con 0 i (m) 1 , tal que g
i

1 (mod m) , esto implica que g


2i
1 (mod m) , de donde 2i = (m)
o equivalentemente, i = (m)/2 .
Teorema 4.20 Sea m un entero positivo. Entonces las unicas solu-
ciones de la congruencia x
2
1 (mod m) son x 1 (mod m) si y
solo si m tiene una raz primitiva.
4.8 N umeros de Fermat
Estos son n umeros de la forma F
n
= 2
2
n
+ 1 . Por ejemplo, F
1
= 5 ,
F
2
= 17 , F
3
= 257 , F
4
= 65537 , los cuales son todos primos. El propio
Fermat armo que los n umeros F
n
son todos primos. Sin embargo,
Euler probo que F
5
= 2
2
5
+ 1 = 2
32
+ 1 es divisible por 641, pues
F
5
= 2
32
+ 1 = 641 6700417 . Para ver que F
5
es divisible por 641,
notemos que 641 = 2
4
+5
4
y 641 = 2
7
5+1 . Por lo tanto, 2
7
5 = 6411
y de aqu se tiene que 2
28
5
4
= (641 1)
4
= 641 N + 1 con N un
entero. Por otra parte, 5
4
= 641 2
4
, de donde obtenemos que
2
28
(641 2
4
) = 641 N + 1
2
28
641 2
28
2
4
= 641 N + 1
2
32
+ 1 = 641 (2
28
N) ,
es decir, 641|(2
32
+ 1) como deseabamos probar
150 Teora de N umeros
Despues se han descompuesto, por ejemplo, F
6
= 27417767280421310721 ,
y F
7
= 59649589127497217 5704689200685129054721 , y no se han en-
contrado otros primos entre los n umeros de Fermat.
Ejemplo 4.39 Sean F
0
, F
1
, . . . , F
k
, , . . . los n umeros de Fermat, es de-
cir, F
n
= 2
2
n
+ 1 . Tenemos que
F
0
F
1
F
n1
= F
n
2
para todo n > 0 .
Solucion. Tenemos que F
0
= 3 y F
1
= 5 , luego F
0
= F
1
2 , y el
resultado vale para n = 1 . Ahora, supongamos por Induccion que
F
0
F
1
F
n1
= F
n
2 .
Luego
F
0
F
1
F
n1
F
n
= (F
n
2)F
n
.
Como (F
n
2)F
n
= (2
22
n
1)(2
2
n
+ 1) = 2
22
n
1 = 2
n+1
1 =
2
n+1
+ 1 2 = F
n+1
2 el resultado se sigue por induccion.
Ejemplo 4.40 Si m = n, entonces mcd (F
m
, F
n
) = 1 .
Solucion. Sin perdida de generalidad, supongamos que m < n.
Supongamos que p es primo y p|F
m
y p|F
n
. Ahora
F
n
2 = F
0
F
1
F
n1
.
Sergio Plaza 151
Y p|F
0
F
1
F
n1
, pues m < n implica que F
m
ocurre como un
factor del producto F
0
F
1
F
m1
. Luego, p|2 , y por lo tanto p = 2 ,
lo cual es imposible pues todos los F
j
son impares. Por lo tanto no existe
ning un primo que divide a F
m
y a F
n
a la vez, as mcd (F
m
, F
n
) = 1 .
4.9 Teorema chino de los restos
Tanto en teora y practica se presentan problemas de encontrar un
n umero que tiene resto prescrito cuando es dividido por dos o mas
modulos. Tales problemas aparecen en adivinanzas chinas antiguas y
su solucion es conocida como teorema chino de los restos. Una de esas
adivinanzas tpicas es la de la canasta con huevos que puede ser enun-
ciada as. Hay una cierta cantidad de huevos en una canasta. Esa
cantidad es tal que si extraemos los huevos desde la canasta en grupos
de 2, 3, 4, 5 y 6 siempre sobra uno, pero si los extraemos en grupos de
7 huevos al nal no queda ninguno en la canasta. Nos estamos pregun-
tando por un entero positivo n tal que n 1 (mod 2) , n 1 (mod 3) ,
n 1 (mod 4) , n 1 (mod 5) ,, n 1 (mod 6) y n 0 (mod 7) .
Veamos primero como se soluciona una congruencia lineal, para des-
pues estudiar la solucion de un sistema de congruencias lineales. Una
congruencia lineal la podemos expresar en la forma
ax b (mod m) .
Por ejemplo, 2x 1 (mod 7) . En este caso vemos que x = 4 es
una solucion. Por otra parte 2x 1 (mod 6) no tiene solucion, pues
la ecuacion 2x 1 (mod 6) signica que buscamos un n umero entero
152 Teora de N umeros
positivo x tal que al dividir 2x por 6 obtenemos resto 1, por otra parte,
al dividir un n umero entero positivo n por 6 obtenemos como restos 0,
1, 2, 3, 4 y 5. Luego los posibles restos para la division de 2n por 6 son
0, 2 y 4. Consecuentemente no hay enteros positivos (ni negativos) que
satisfacen la congruencia 2x 1 (mod 6) .
Una diferencia inmediata entre la congruencia 2x 1 (mod 7) y
2x 1 (mod 6) es que mcd (2, 7) = 1 y mcd (2, 6) = 2 , es decir, 2 y 7
son coprimos, pero 2 y 6 no lo son.
Ejemplo 4.41 Existe un entero x tal que 7x 1 (mod 17) ? lo
mismo para 6x 1 (mod 15) ?
Solucion. Tenemos 7 5 = 35 1 (mod 17) . Armamos que no existe
solucion para 6x 1 (mod 15) . En efecto, si existe un tal x , entonces
6x 1 , y debe ser divisible por 15; en particular debe ser divisible por
3, y entonces 1 debe ser divisible por 3, esto es una contradiccion, lo
cual prueba nuestra armacion.
Antes de enunciar el resultado que establece bajo que condiciones una
congruencia lineal ax b (mod m) tiene solucion, veamos un principio
logico, que parece demasiado elemental para ser enunciado, pero que es
de gran utilidad en algunas demostraciones
si m objetos son colocados en m cajas, de modo que no haya dos
objetos en una misma caja, entonces existe al menos un objeto en cada
caja.
Volvamos a la congruencia ax b (mod m) . Tomamos los m objetos
como los distintos n umeros
0, a , 2a , 3a , . . . , (m1)a
Sergio Plaza 153
que corresponden a los posibles restos de dividir ax por m.
El problema que podemos tener aqu es que un par de esos n umeros
sean congruentes (mod m) , esto es
ax ay (mod m)
tiene solucion en {0, a, 2a, . . . , (m1)a} .
Si mcd (a, m) = 1 , es decir, a y m son coprimos, podemos usar
la ley de cancelacion en la congruencia ax ay (mod m) obteniendo
que x y (mod m) . Por lo tanto, por el principio logico enunciado, los
n umeros 0 , a , 2a , 3a , . . . , (m1) a deben ser congruente en alg un or-
den a los n umeros 0, 1, 2, . . . , m1 . En particular, existe exactamente
un x {0, 1, . . . , m 1} para el cual ax b (mod m) . Formalizando
la discusion anterior, tenemos el siguiente.
Teorema 4.21 La congruencia lineal ax b (mod m) tiene solucion
si a y m son coprimos. Si este es el caso, entonces existe una unica
solucion (mod m) .
Recordemos que en un conjunto numerico, el inverso multiplicativo de
un n umero no cero a , es un n umero x tal que ax = 1 . En el conjunto de
los enteros tenemos que la ecuacion ax = 1 tiene solucion para a = 1
y a = 1 , solamente, con x = 1 y x = 1 , respectivamente. Nos
podemos preguntar que ocurre en aritmetica modular, es decir, cuando
la congruencia lineal ax 1 (mod m) tiene solucion.
Por lo que vimos antes, esa congruencia tiene solucion si y solo si a
y m son coprimos. Esto es, si a y m son coprimos, entonces en el
conjunto {0, 1, . . . , m 1} con el producto, (mod m) , cada elemento
distinto de cero tiene un inverso multiplicativo.
154 Teora de N umeros
Retornemos al problema de sistemas de congruencias lineales. Ten-
emos el siguiente.
Teorema 4.22 (teorema chino de los restos). Un sistema de congruen-
cias lineales x c
1
(mod m
1
) , . . . , x c
k
(mod m
k
) , en el cual los
modulos m
1
, . . . , m
k
son coprimos a pares (es decir, mcd (m
i
, m
j
) = 1
cuando i = j ) tiene solucion. Ademas, la solucion es unica (mod m
1

m
2
m
k
.
Demostracion. Probemos primero la unicidad de la solucion, bajo
el supuesto que exista. Si tenemos dos soluciones x e y , entonces
x y (mod m
i
) para cada i = 1, . . . , k , y por lo tanto, dado que los
m
i
son coprimos dos a dos, se tiene que x y (mod m
1
m
2
m
k
) , y
por lo visto anteriormente, en este caso existe solo una solucion.
Veamos ahora el problema de la existencia de una tal solucion.
Para cada i = 1, 2, . . . , k sea M
i
el producto de todos los modulos,
excepto m
i
, es decir, M
i
= m
1
m
2
m
i1
m
i+1
m
k
.
Es claro que M
i
y m
i
son coprimos, luego la congruencia lineal
a
i
M
i
1 (mod m
i
)
tiene solucion.
Consideremos el n umero x dado por
x = a
1
M
1
c
1
+a
2
M
2
c
2
+ +a
k
M
k
c
k
.
Como M
j
0 (mod m
i
) cuando i = j , vemos que
x a
i
M
i
c
i
= c
i
(mod m
i
)
Sergio Plaza 155
para cada i = 1, 2, . . . , k . Por lo tanto x es la solucion buscada. Esto
completa la prueba del teorema.
Busquemos la solucion al problema de los huevos en la canasta, tene-
mos que resolver el sistema de congruencias lineales
x 0 (mod 7)
x 1 (mod 2)
x 1 (mod 3)
x 1 (mod 4)
x 1 (mod 5)
x 1 (mod 6)
es decir, x es divisible por 7 y deja resto 1 cuando se lo divide por 2, 3,
4, 5 y 6. De esto ultimo vemos que x excede en 1 a un m ultiplo de 60.
Luego el problema se reduce a resolver
x 1 (mod 60)
x 0 (mod 7) .
Como 7 y 60 son coprimos, c
1
= 1 y c
2
= 0 ?? desde la con-
struccion de la prueba del tenemos que denir
x = a
1
M
1
c
1
+a
2
M
2
c
2
lo cual se reduce a x a
1
M
1
c
1
(mod 420) y c
1
= 1 , M
1
= 7 , es
decir, x = 7a
1
1 (mod 1)(mod 60) . Realizando los calculos (va
calculadora) obtenemos que a
1
= 43 , luego x = 301 (mod 420) . Por
156 Teora de N umeros
lo tanto el menor n umero de huevos que debe haber en la canasta para
cumplir las condiciones del problema es de 301 huevos.
4.9.1 Ejercicios
Problema 4.19 Pruebe que los enteros n 7 (mod 8) no pueden ser
escritos como suma de tre cuadrados.
Indicacion. Escribe n = x
2
+y
2
+z
2
y considere todas las posibilidades
mod 8 .
Problema 4.20 Cuales son los posibles valores de a s
3
(mod 9) ?
Cuales son los posibles valores de 5
3
+t
3
(mod 9) ? Pruebe que existen
innitos enteros que no pueden ser escritos como suma de dos cubos
Existen enteros que no pueden ser escritos como suma de tres cubos?
Problema 4.21 Sea N = d
n
d
n1
d
1
d
0
un entero escrito en notacion
decimal. Pruebe que N d
0
(mod 2) , N d
1
d
0
(mod 4) y N
d
2
d
1
d
0
(mod 8) .
Problema 4.22 Suponga que p es un primo impar. Pruebe que
p

j=0
_
p
j
__
p +j
j
_
2
p
+ 1 (mod p
2
) .
Problema 4.23 Sea n un entero positivo. Pruebe que n es primo si
y solo si
_
n 1
k
_
(1)
k
(mod n)
para todo k {0, 1, . . . , n 1} .
Sergio Plaza 157
Problema 4.24 Pruebe que para cada n 2 ,
n terminos n 1 terminos
..
2
2

2
=
..
2
2

2
( mod n)
Problema 4.25 Para cada n 1 , determine los posibles valores de la
sucesion
2, 2
2
, 2
2
2
, 2
2
2
2
, . . . , (mod n) .
Problema 4.26
Problema 4.27
Problema 4.28
Problema 4.29
Problema 4.30
Problema 4.31
Problema 4.32
4.10 Ecuaciones de Fermat
Sea n un entero positivo mayor que 2 . La ecuacion de Fermat de grado
n es
x
n
+y
n
= z
n
. (4.1)
158 Teora de N umeros
Fermat armo que la ecuacion (4.1) no tiene soluciones enteras posi-
tivas para todo n > 2 .
Para n = 2 , la ecuacion es x
2
+ y
2
= z
2
la cual tiene soluciones
enteras positivas, de hecho innitas, dadas por enteros de las ternas
pitagoricas.
El ultimo teorema de Fermat establece que no existen enteros positivos
x, y y z con
x
n
+y
n
= z
n
si n es un entero mayor que dos.
Lista de personas mas importantes relacionadas con este resul-
tado
Euclides de Alejandra 300
Diofanto de Alejandra 250
Pierre Fermat 16011665
Leonhard Euler 17071783
Joseph Louis Lagrange 17361813
Sophie Germain 17761831
Carl Friedrich Gauss 17771855
Agustin Louis Cauchy 17891857
Gabriel Lame 17951870
Peter Gustav Lejeune Dirichlet 18051859
Joseph Liouville 18091882
Ernst Eduard Kummer 18101893
Harry Schultz Vandiver 18821973
Gerhard Frey
Kenneth A. Ribet
Sergio Plaza 159
Andrew Wiles 1953
Para n = 2 existen innitas soluciones
3
2
+ 4
2
= 5
2
, 5
2
+ 12
2
= 13
2
, 8
2
+ 15
2
= 17
2
, . . .
Los triples pitagoricos. En el margen de su copia de la Aritmetica
de Diofantos en la edicion de Bachet, el jurista frances P. Fermat es-
cribio, alrededor de 1637, que para n grande no pueden existir tales
triples, agrega ademas que el tiene una prueba maravillosa para esa
armacion, la cual, sin embargo, siendo el margen del libro tan peque no
no la puede escribir ah. La armaci on exacta de Fermat fue la siguiente.
Cubum autem in duos cubos, ant quadrato-quadratum in duos quadrato-
quadratos, et generaliter nullam in innitum ultra quadratum potes-
tatem in duous ejusdem nominis fas et dividere cujus rei demostrationem
mirabilem sane detexi. Hame marginis exiguitas non caparet.
Todos los otros resultados que Fermat enuncio en esta forma fueron
probados hace mucho tiempo, solo este, el ultimo permanecio casi 360
a nos sin resolverse, fue en 1994 que A. Wiles obtuvo una prueba para el
ultimo teorema de Fermat.
En el problema 8 el Libro II de la traduccion de Claude Bachet de
la Aritmetica de Diofanto se pregunta sobre una regla para escribir un
cuadrado como la suma de dos cuadrados. La ecuacion resultante z
2
=
x
2
+ y
2
es la del Teorema de Pitagoras, el cual establece que en cada
triangulo rectangulo el cuadrado sobre la hipotenusa es la suma de los
cuadrados sobre los otros dos lados (en general, llamados catetos).
Como vimos en el caso n = 2 , las soluciones de la ecuacion de Fermat
son conocidas, y son llamadas ternas pitagoricas, las que estudiamos
160 Teora de N umeros
ahora.
4.11 Ternas Pitagoricas
Una terna ( x, y, z) de n umeros enteros positivos es llamada pitagorica si
x
2
+y
2
= z
2
. Por ejemplo, (3, 4, 5) es una terna pitagorica, pues 3
2
+
4
2
= 5
2
. Una terna pitagorica puede ser interpretada geometricamente
como los lados de un triangulo rectangulo como en la gura.
Figura
Un problema interesante es saber si existen otras ternas pitagoricas
aparte de la ya mencionada.
Si (x, y, z) es una terna pitagorica y d = mcd (x, y) entonces d
2
|(x
2
+
y
2
) de donde d|z , por lo tanto existen enteros no nulos a , b , c con
mcd (a, b) = 1 tales que (x, y, z) = (da, db, dc) . Ahora como x
2
+
y
2
= z
2
si y solo si a
2
+ b
2
= c
2
, basta encontrar ternas pitagoricas
(a, b, c) tales que mcd (a, b) = 1 , esto implica que mcd (a, c) = 1 y
mcd (b, c) = 1 . La ternas pitagoricas (x, y, z) , tales que mcd(x, z) = 1
y mcd(y, z) = 1 son llamadas ternas pitagoricas primitivas.
Tenemos que si k Z entonces k
2
0 (mod 4) o k
2
1 (mod 4) ,
pues si k es par entonces k = 2 , luego k
2
4
2
, es decir, k
2
es
Sergio Plaza 161
divisible por 4, as k
2
0 (mod 4) , por otra parte si k es impar entonces
k = 2n + 1 , luego k
2
= 4n
2
+ 4n + 1 = 4(n
2
+n) + 1 1 (mod 4) .
Aplicando esto a nuestro problema vemos que si a y b son impares
entonces a
2
1 (mod 4) y b
2
1 (mod 4) luego a
2
+b
2
2 (mod 4) lo
cual es imposible, pues a
2
+b
2
= c
2
y tendramos que c
2
2 (mod 4) .
Ahora como mcd (a, b) = 1 , es decir a y b son coprimos y no pueden
ser ambos impares, concluimos que hay dos casos a considerar: a impar
y b par, a par y b impar. Por la simetra de la ecuacion y del argumento
a seguir, solo basta analizar uno de los casos. Elegimos el primero, es
decir, a impar y b par. Como a es impar se sigue que a
2
tambien es
impar, luego a
2
+b
2
es impar, por lo tanto c
2
es impar y en consecuencia
c debe ser impar.
Desde a
2
+ b
2
= c
2
se sigue que b
2
= (c a)(c + a) y como a y
c son impares, tenemos que c a y c + a son pares. Ademas, como
mcd (c a, c + a) = 2 y como b es par se tiene que b
2
es divisible por
4, y podemos escribir
_
b
4
_
2
=
c a
2

c +a
2
y como mcd (ca, c+a) = 2 se tiene que mcd
_
ca
2
,
c+a
2
_
= 1 , es decir,
ca
2
y
c+a
2
son coprimos.
Como
ca
2

c+a
2
= cuadrado perfecto, y mcd
_
ca
2
,
c+a
2
_
= 1 debemos
tener que
ca
2
y
c+a
2
son cuadrados perfectos, es decir,
c+a
2
= u
2
y
ca
2
= w
2
, de donde c + a = 2u
2
y c a = 2v
2
. Sumando estas dos
ecuaciones nos queda 2c = 2(u
2
+ v
2
) , y de ah c = u
2
+ v
2
de aqu
u
2
+v
2
+ a = 2u
2
luego a = u
2
v
2
. Ahora, como
_
b
4
_
2
=
(ca)
2
2
(c+a)
2
obtenemos
_
b
2
_
2
=
2v
2
2

2u
2
2
= v
2
u
2
, de donde b
2
= 4u
2
v
2
, por lo tanto
b = 2uv (pues son positivos).
162 Teora de N umeros
Concluimos entonces que la terna pitagorica viene dada por (a, b, c) =
(u
2
v
2
, 2uv, u
2
+v
2
) , con u, v enteros positivos y u > v .
Por ejemplo, tomando u = 2 y v = 1 obtenemos la terna (u
2

v
2
, 2uv, u
2
+ v
2
) = (3, 4, 5) . Ahora, si elegimos u = 3 y v = 2 , obten-
emos (u
2
v
2
, 2uv, u
2
+ v
2
) = (5, 12, 13) . Con la eleccion u = 7 y
v = 5 , obtenemos (u
2
v
2
, 2uv, u
2
+v
2
) = (24, 70, 74) .
En particular, si elegimos u = 2n y v = 1 , obtenemos la terna
pitagorica (4n
2
1, 4n, 4n
2
+ 1) , y dando valores a n N obtenemos
innitas ternas pitagoricas. Por ejemplo, para n = 1 obtenemos la terna
pitagorica clasica (3, 4, 5) , para n = 2 , obtenemos la terna (15, 16, 17) ,
para n = 3 , tenemos la terna (35, 12, 37) y para n = 4 tenemos la
terna (63, 16, 65) . E lector puede calcular, usando la formula arriba
otras ternas pitagoricas.
Apliquemos la idea anterior para estudiar la ecuacion x
2
+y
2
= 2 z
2
con x, y enteros, x = y .
Observemos primero que como 2 z
2
es par, x e y deben tener la
misma paridad (ambos pares o ambas impares). Luego, existen enteros
a y b tales que x = a +b , y = a b . Para ello basta tomar a =
x+y
2
y b =
xy
2
, los cuales son enteros pues x + y y x y son pares.
Reemplazando en la ecuacion original nos queda ( a+b)
2
+(ab)
2
= 2z
2
,
y desarrollando obtenemos 2(a
2
+ b
2
) = 2z
2
, es decir, nos queda la
ecuacion a
2
+ b
2
= z
2
, y por lo tanto debemos encontrar solucion a
esta ultima ecuacion, y como vimos estas vienen dadas por las ternas
pitagoricas (a, b, z) = ((u
2
v
2
)d, 2duv, (u
2
+ v
2
)d) , donde u, v, d son
enteros, con u = v , y mcd (u, v) = 1 , y u y v con paridad distinta.
Sergio Plaza 163
Ahora como x = a +b e y = a b nos queda
_

_
x = (du
2
dv
2
) + 2duv = d(u
2
+ 2uv v
2
)
y = (du
2
dv
2
) 2duv = d(u
2
2uv v
2
)
z = d(u
2
+v
2
)
4.11.1 Ecuacion de Fermat para n = 4
Veamos ahora el caso n = 4 .
Supongamos primero que n es un m ultiplo de 4, es decir, n = 4k
para alg un entero positivo k . Si existiesen enteros positivos no nulos,
x, y , z tales
x
n
+y
n
= z
n
podemos escribir esto en la forma (x
k
)
4
+ (y
k
)
4
= (z
2k
)
2
, es decir,
x
k
+y
k
= z
2k
es entonces una solucion de la ecuacion
u
4
+v
4
= w
2
. (4.2)
En conclusion para mostrar que x
n
+ y
n
= z
n
. Con n = 4k , no tiene
soluciones enteras positivas no nulas, basta demostrar que la ecuacion
u
4
+v
4
= w
2
no tiene soluciones enteras positivas no nulas.
Supongamos que (4.2) tiene una solucion entera positiva (a, b, c) .
Podemos elegir (a, b, c) de modo que no existe otra solucion ( a,

b, c)
de (4.2) con c < c . Si a y b son coprimos, entonces existen enteros
positivos u y v tales que a
2
= u
2
v
2
, b
2
= 2uv y c = u
2
+ u
2
.
Como a
2
+ v
2
= u
2
, existen enteros positivos p y q , coprimos, tales
que a = p
2
q
2
, v = 2pq y u = p
2
+q
2
.
De esto, se sigue que b
2
= 2uv = 4pq(p
2
+ q
2
) . Como p y q son
coprimos, se sigue que p , q son coprimos con p
2
+q
2
. Como 4pq(q
2
+q
2
)
164 Teora de N umeros
es un cuadrado, se sigue que p , q y p
2
+ q
2
deben ser cuadrados, as
p = r
2
y p
2
+ q
2
= t
2
, para algunos enteros r, s, t enteros positivos no
nulos. De lo anterior tenemos que r
4
+s
4
= t
2
, con c = u
2
+v
2
> u =
p
2
+ q
2
= t
2
> t . Esto contradice el hecho que c fue elegido de modo
que cualquier otra solucion ( a,

b, c) de (4.2) debe satisfacer c < c .


Usando la caracterizacion completa de los triangulos pitagoricos, pode-
mos dar una prueba del ultimo Teorema de Fermat en el caso especial
n = 4 .
Sin embargo, preferimos dar unas pequeas vueltas por otra ruta la
cual tambien nos lleva al mismo objetivo, y tambien a una propiedad
notable acerca de triangulos rectagulos.
Teorema 4.23 El area de un triangulo pitagorico no puede ser un
cuadrado.
Demostracion. La prueba consiste en mostrar que, si el area de un
tal triangulo es un cuadrado, entonces existe un triangulo menor con la
misma propiedad, y as sucesivamente ad innitum, lo cual es imposible.
Supongamos que existen enteros positivos a , b , c tales que
a
2
+b
2
= c
2
y para el cual el area del triangulo, ab/2 es un cuadrado. No existe
perdida de generalidad en suponer que el triangulo dado es primitivo,
luego, a , b y c tienen la forma
a = m
2
n
2
, b = 2mn, c = m
2
+n
2
,
donde m y n son coprimos de paridad opuesta. Intercambiando a
y b , si es necesario, podemos suponer que a es impar. Ahora, el area
del triangulo es mn(m n)(m + n) . Pero la unica manera que un
Sergio Plaza 165
producto de n umeros coprimos sea un cuadrado es que cada una de ellos
sea un cuadrado. (Esto se sigue directamente del T.F.A.). Por lo tanto
(mn)(m +n) es un cuadrado, el cual llamamos p
2
, luego
p
2
+n
2
= m
2
,
donde p y m son impares y n es par (recuerde que a = p
2
es impar).
El nuevo triangulo con lados p , m y m es tambien primitivo, y podemos
escribir
p = m
2
1
n
2
1
, n = 2m
1
n
1
, m = m
2
1
+n
2
1
,
donde m
1
y n
1
son coprimos y de paridad opuesta. Pero n es un
cuadrado, y por lo tanto m
1
o bien n
1
debe ser un cuadrado impar
mientras que el otro debe ser el doble de un cuadrado. Finalmente, m
es tambien un cuadrado, digamos m = u
2
y entonces la tercera ecuacion
arriba es
m
2
1
+n
2
1
= u
2
.
Luego, m
1
, n
1
, y u son los lados de un triangulo pitagorico cuya
area
1
2
m
1
n
1
es un cuadrado perfecto cuya hipotenusa es menor que la
hipotenusa del triangulo original, pues
u =

m < m < m
2
< m
2
+n
2
= c .
Esto completa la prueba por descenso innito.
Usando la caracterizacion completa de los triangulos pitagoricos, pode-
mos dar una prueba del ultimo Teorema de Fermat en el caso especial
n = 4 .
166 Teora de N umeros
Sin embargo, preferimos dar unas peque nas vueltas por otra ruta la
cual tambien nos lleva al mismo objetivo, y tambien a una propiedad
notable acerca de triangulos rectagulos.
Teorema 4.24 El area de un triangulo pitagorico no puede ser un
cuadrado.
Demostracion. La prueba consiste en mostrar que, si el area de un
tal triangulo es un cuadrado, entonces existe un triangulo menor con la
misma propiedad, y as sucesivamente ad innitum, lo cual es imposible.
Supongamos que existen enteros positivos a, b, c tales que
a
2
+b
2
= c
2
y para el cual el area del triangulo, ab/2 es un cuadrado. No existe
perdida de generalidad en suponer que el triangulo dado es primitivo,
luego, a, b, c tienen la forma
a = m
2
n
2
, b = 2mn, c = m
2
+n
2
donde m y n son coprimos de paridad opuesta. Intercambiando a y
b , si es necesario, podemos suponer que a es impar. Ahora, el area
del triangulo es mn(m n)(m + n) , y es facil ver otros tres. Pero la
unica manera que un producto de n umeros coprimos sea un cuadrado es
que cada una de ellos sea un cuadrado. (Esto se sigue directamente del
T.F.A). Por lo tanto (mn)(m+n) es un cuadrado, el cual llamamos
p
2
, luego
p
2
+n
2
= m
2
Sergio Plaza 167
donde p y m son impares y n es par (recuerde que a = p
2
es impar).
El nuevo triangulo con lados p, m, y m es tambien primitivo, y podemos
escribir
p = m
2
1
n
2
1
, n = 2m
1
n
1
, m = m
2
1
+n
2
1
donde m
1
y n
1
son coprimos y de paridad opuesta. Pero n es un
cuadrado, y por lo tanto m
1
o bien n
1
debe ser un cuadrado impar
mientras que el otro debe ser el doble de un cuadrado. Finalmente, m
es tambien un cuadrado, digamos m = u
2
y entonces la tercera ecuacion
arriba es
m
2
1
+n
2
1
= u
2
.
Luego, m
1
, n
1
, y u son los lados de un triangulo pitagoricocuya
area m
1
n
1
/2 es un cuadrado perfecto cuya hipotenusa es menor que la
hipotenusa del triangulo original, pues
u =

m < m < m
2
< m
2
+n
2
= c .
Esto completa la prueba por descenso innito.
Corolario 4.5 La ecuacion
a
4
b
4
= c
2
no tiene soluciones enteras positivas a, b, c .
Demostracion. Si la ecuacion tiene soluciones enteras positivas a , b ,
c entonces a
4
b
4
, 2a
2
b
2
, a
4
+ b
4
forman los lados de un triangulo
rectangulo, pues
168 Teora de N umeros
(a
4
b
4
)
2
+ (2a
2
b
2
)
2
= (a
4
+b
4
)
2
y el area del triangulo debe ser
1
2
2a
2
b
2
(a
4
b
4
) = a
2
b
2
(a
4
b
4
) = a
2
b
2
c
2
que es un cuadrado. Ahora, como el area de ning un triangulo Pitagorico
puede ser un cuadrado, tenemos una contradiccion, y el corolario esta
demostrado.
La misma tecnica usada para resolver x
2
+ y
2
= x
2
puede ser usada
para resolver ecuaciones del tipo x
2
+ ay
2
= x
2
, para ello escriba la
ecuacion en la forma ay
2
= (z x)(z +x) .
Ecuaciones como la siguiente x
2
+ y
2
= 2z
2
no pueden reducirse al
tipo anterior, pues no podemos escribirla como diferencia de cuadrados.
Logrange, observo que en este caso multiplicacion por 2 permite escribir
(2z)
2
= 2x
2
+ 2y
2
= (x +y)
2
+ (x y)
2
luego nos queda (2z x y)(2z +x +y) = (x y)
2
, y ahora procede-
mos a solucionar la ecuacion exactamente como lo hicimos para triples
pitagoricos.
Veamos como podemos tratar ecuaciones del tipo AX
2
+BY
2
= CZ
2
,
teniendo al menos una solucion. Multiplicando por A
1
vemos que es
suciente considerar ecuaciones del tipo x
2
+ ay
2
= bz
2
. Supongamos
que (x, y, z) es una solucion de esta ecuacion. Entonces
Sergio Plaza 169
(bzZ)
2
= bz
2
Z
2
+abz
2
Z
2
= (x
2
+ay
2
)X
2
+ (ax
2
+a
2
y
2
)Y
2
= (xX +yY )
2
+a(yX xY )
2
Luego, a(yX xY )
2
= (bzZ)
2
(xX + ayY )
2
es una diferencia de
cuadrados, y procedemos como lo hicimos para triangulos pitagoricos.
4.12 Ultimo Teorema de Fermat
Teorema 4.25 (

Ultimo teorema de Fermat) La ecuacion x


n
+y
n
= z
n
no tiene soluciones enteras no ceros si n 3 .
Se cree que Fermat tena una prueba de este resultado para n = 4 y
que erroneamente creyo que su argumento poda ser generalizado para
el caso general. Por mas de tres siglos y medio un gran n umero de
matematicos trataron, infructuosamente de probar este teorema, y du-
rante estas investigaciones para tener que dar una prueba muchos con-
ceptos nuevos y teoras fueon creadas. En los inicios de los 90 (siglo
pasado), la conjetura de Fermat, como paso a llamarse este resultado,
fue vericado para todo n continendo un factor primo impar menor que
10
6
, usando computadoras. En junio de 1993, Andrew Wiles anuncio
que tena una prueba del teorema de Fermat, pero su prueba original
contena algunas lagunas; esas fueron corregidas un ao despues por An-
drew Wiles y Richard Taylor. La conjetura de Fermat fue nalmente
probada y paso a ser un teorema. La prueba es muy larga y usa resul-
tados profundos de geometra algebraica.
Daremos la prueba del teorema de Fermat para n = 4 . De hecho
probaremos un resultado un poco mas fuerte.
170 Teora de N umeros
Teorema 4.26 La ecuacion x
4
+y
4
= x
2
no tiene soluciones enteras
no ceros.
Demostracion. Supongamos lo contrario, entonces existen soluciones
enteras x, y, x . Como cualquier cambio de signo de esas soluciones
no altera el resultado, podemos suponer que x, y, z son enteros posi-
tivos. Podemos suponer tambien que z es el menor posible. Vamos
a obtener una contradiccion probando que existe otra solucion entera
positiva (x
1
, y
1
z
1
) con z
1
< z .
Supongamos que mcd(x, y) > 1 , entonces existe un primo p dividi-
endo a ambos x e y . Se sigue entonces que p
4
|(x
4
+y
4
) , esto es, p
4
|z
2
luego p
2
|z . De esto, se tiene que
_
x
p
_
4
+
_
y
p
_
4
=
_
z
p
2
_
2
, y hemos encon-
trado una solucion con z
1
=
z
p
2
< z . Esto contradice lo que asumimos
sobre la eleccion original de (x, y, z) , por lo tanto se debe tener que
mcd (x, y) = 1 . De esto se sigue que mcd (x
2
, y
2
) = 1 , luego (x
2
, y
2
, z)
es un triple pitagorico primitivo. Podemos asumir que x
2
es impar e y
2
es par, luego existen enteros positivos coprimos u y v tales que
x
2
= u
2
v
2
, y
2
= 2uv, x = u
2
+v
2
.
En particular, (x, v, u) es un triple pitagorico primitivo con x impar.
Por lo tanto, existen enteros positivos coprimos s y t tales que
x = s
2
t
2
, v = 2st, us
2
+t
2
Como mcd (s, t) = 1 se sigue de la ultima igualdad que u, s , y t son
coprimos a pares. Pero
_
y
2
_
2
=
uv
2
= ust , luego el producto ust es
un cuadrado perfecto, y esto implica que u, s , y t son todos cuadrados
Sergio Plaza 171
perfectos. Luego existen enteros positivos a, b y c tales que s = a
2
, t =
b
2
y u = c
2
. Como u = s
2
+ t
2
, se sigue que a
4
+ b
4
= c
2
, es decir,
(a, b, c) es una solucion positiva de la ecuacion original. Esto contradice
lo asumido sobre la minimalidad de z , pues c =

y u
2
< u
2
+v
2
= z .
Esto completa la prueba del teorema.
Corolario 4.6 La ecuacion x
4
+ y
4
= z
4
no tiene soluciones enteras
no ceros.
Demostracion. Si (x, y, z) es una tal solucion, entonces (x, y, z
2
) es
una solucion de la ecuacion del teorema anterior. Esto es una con-
tradiccion.
172 Teora de N umeros

Ultimo teorema de Fermat para n = 4 . La solucion de x


2
+y
2
= z
2
es la clave para demostar que la ecuacion diofantina
x
4
+y
4
= z
4
,
tiene solo solicones triviales, es decir, aquellas con x = 0 o y = 0 .
Para ello basta probar que la ecuacion
x
4
+y
4
= z
2
tiene solo soluciones triviales, esto es, si x
4
+ y
4
no puede ser un
cuadrado, entonces tampoco puede ser una cuarta potencia.
Teorema 4.27 La ecuacion de Fermat
x
4
+y
4
= z
2
no tiene soluciones enteras con x y z = 0
La prueba que daremos es debida a Euler, sin embargo Fermat propuso
algo similar.
Demostracion. Primero que nada, para el caso n par basta conside-
rar el caso x, y , z enteros positivos. Podemos suponer, y de hecho
lo hacemos, que los enteros x, y y Z son coprimos a pares, si no es
as, cancelamos los divisores comunes. Seguimos un poco la solucion
de la ecuacion pitagorica, tenemos que z debe ser impar, pues si z es
par, entonces x e y debe ser impares y tenemos una contradiccion (ver
demostracion de ternas pitagoricas).
Sergio Plaza 173
Supongamos que x es impar e y es par (esto basta por la simetra
de la ecuacion). Escribamos x
4
+y
4
= z
2
como y
4
= (z x
2
)(z +x
2
) ,
como cualquier divisor com un d de z x
2
y z +x
2
divide su suma y su
diferencia, se obtiene que d = 2 . Luego R =
1
2
(z x
2
) y S =
1
2
(z +x
2
)
son coprimos, y RS =
1
4
y
2
. Como R y S no son ambos pares, bien
R es impar (y entonces R y 4S son coprimos) o bien S es impar (y
entonces 4R y S son coprimos). En el primer caso, R 4S = y
4
es una
cuarta potencia, luego 2R = z x
2
= 2a
4
y 4S = 2(z + x
2
) = (2b)
4
,
esto es, z + x
2
= 8b
4
para enteros positivos a , b , en el segundo caso,
4R S = y
4
, y entonces z x
2
= 8a
4
y z + x
2
= 2b
4
. La primera
posibilidad nos lleva a 4b
4
a
4
= x
2
, la cual es imposible mod 4 ,
pues la ecuacion nos da a
4
x
2
(mod 4) con a y x impar, pero
los cuadrados de n umeros impares son congruentes mod 4 a 1, luego la
congruencia es 1 1 mod 4 que es imposible. Por lo tanto estamos
en el segundo caso y obtenemos b
4
4a
4
= x
2
.
Ahora escribiendo 4a
4
= (b
2
x)(b
2
+ x) , como x y b son impares,
tenemos que mcd (b
2
x, b
2
+ x) = 2 y b
2
x = 2r
4
, b
2
+ x = 2s
4
.
Sumando estas ecuaciones obtenemos b
2
= r
4
+ s
4
, esto es, hemos en-
contrado una nueva solucion (b, r, s) a nuestra ecuacion z
2
= x
4
+ y
4
,
como 0 < b < x < z , esto signica que para cada solucion (x, y, z) en
los n umeros enteros positivos existe otra solucion con un z mas peque no.
Esto es imposible y por y por descenso innito la prueba esta completa.
4.13 Teorema de Wilson y Teorema de Fermat
Lema 4.2 Sea p un primo y sea 0 < k < p . Entonces k
2
1 (mod p)
si y solo si k = 1 o k = p 1 .
174 Teora de N umeros
Demostracion. Si k = 1 , entonces k
2
1 (mod p ) . Si k = p 1 ,
entonces k
2
= p
2
2p + 1 1 (mod p) .
Recprocamente, supongamos que k
2
1 (mod p) . Entonces p|(k
2

1) = (k1)(k+1) , y como p es primo, , se tiene que p|(k1) o p|(k+1) .


El unico n umero en {1, 2, . . . , p 1} que satisface p|(k 1) es 1, y el
unico n umero en {1, 2, . . . , p 1} que satisface p|(k + 1) es p 1 .
Teorema 4.28 (Wilson) Sea p > 1 . Entonces p es primo si y solo si
(p 1)! 1 (mod p) .
Demostracion. Supongamos que p es primo. Si k {1, . . . , p 1} ,
entonces k es coprimo con p . Luego existen enteros a y b tales que
ak+bp = 1 , esto es, ak 1 (mod p) . Reduciendo a (mod p) , podemos
asumir que a {1, . . . , p 1} .
Luego, cada elemento de {1, . . . , p 1} tiene un recproco mod p en
este conjunto. El lema anterior muestra que solo 1 y p 1 son sus
propios recprocos. Tenemos as que los elementos 2, . . . , p 2 deben
ser pareados en pares {x, x
1
} . Se sigue que su producto es 1. Luego,
(p 1)! = 1 2 (p 2)(p 1)
= 1 1 (p 1)
= p 1
1 (mod p)
Ahora, suponga que (p 1)! 1 (mod p) .
Tenemos que mostrar que p es primo. Comenzamos reescribiendo la
ecuacion como (p 1)! + 1 = kp .
Sergio Plaza 175
Supongamos que p = ab , con 1 a, b p . Si a = p , la factorizacion
es trivial, luego a < p . Entonces a|(p 1)! , pues a {1, . . . , p 1} y
a|p , luego (p1)! +1 = kp muestra que a|1 . Por lo tanto, a = 1 . Esto
prueba que la unica factorizacion de p es la trivial, consecuentemente
p es primo.
Ejemplo 4.42 El teorema ded Wilson implica que el producto de cua-
lesquiera 10 enteros consecutivos, ninguno divisible por 11, son iguales
a 1 (mod 11) , pues cualesquiera 10 enteros consecutivos se reducen
mod 11 a {1, 2, . . . , 10} .
Por ejemplo,
12 13 20 21 1 (mod 11) .
Teorema 4.29 (Fermat) Sea p un primo. Suponga que p no divide
a a , entonces
a
p1
1 (mod p) .
Demostracion. La idea es mostrar que los enteros
a, 2a, . . . , (p 1)a
se reducen mod p a 1, . . . , p 1 , y entonces aplicar el teorema de
Wilson.
Hay p 1 n umeros en el conjunto {a, 2a, . . . , (p 1)a} . Luego, lo
que necesitamos probar que ellos son distintos (mod p) . Supongamos
que 1 j, k p 1 , y
a
j
= ak (mod p).
176 Teora de N umeros
Esto signica que p|(aj ak) = a(j k) , luego p|a o p|(j k) . El
primer caso es descartado por hipotesis, se tiene que p|(j k) . Pero
como 1 j, k p 1 , se tiene p|(j k) solo si j = k .
Luego, {a, 2a, . . . , (p 1)a} son p 1 n umeros distintos (mod p) .
Si reducimos (mod p) , obtenemos los n umeros {1, . . . , p 1} .
Luego,
a 2a (p 1)a = 1 2 (p 1) = (p 1)! 1 (mod p) .
Por otra parte, aplicando el teorema de Wilson otra vez, tenemos que
a 2a (p 1)a = a
p1
(p 1)! a
p1
(mod p) ,
esto es, a
p1
1 (mod p) , es decir, a
p1
1 (mod p) .
Corolario 4.7 Si p es primo, entonces a
p
a (mod p) para todo a .
Demostracion. Si p|a , entonces a
p
a (mod p) y a 0 (mod p) ,
luego a
p
a (mod p) . Si p no divide a a , entonces a
p1
1 (mod p) .
Multiplicando por a esta congruencia, obtenemos a
p
a (mod p) .
Ejemplo 4.43 Calcule 50
250
(mod 83) .
Reducimos modulo 17. Como 83 no divide a 80, por el teorema de
Fermat
50
82
1 (mod 17) .
Ahora, 3 82 = 246 , luego
50
250
= 50
246
50
4
Sergio Plaza 177
= (50
82
)
3
2500
2
= 1
3
10
2
= 100
= 17 (mod 83) .
En otras palabras, si esta tratando de reducir a
k
(mod p) , donde p
no divide a a , saque tantos factores a
p1
como sea posible, y entonces
reduzca el resto a mano.
4.14 Matematicas e Historia
En Chile se declaro la Independencia el 18 de Septiembre de 1810 Que
da de la semana fue?
1
Similarmente podemos preguntarnos por otras
fechas memorables, por ejemplo, el 12 de Octubre de 1492.
Un a no normal tiene 365 das y un a no bisiesto tiene 366 das. Los
a nos bisiestos son aquellos no seculares divisibles por cuatro, por ejem-
plo, 1812, 1816, 1820,... Los a nos seculares son bisiestos si son divisibles
por cuatrocientos, por ejemplo, 1600, 2000,... El a no 1.500 si fue bisiesto
pues es anterior a la reforma que se hizo del calendario en 1582, durante
el papado de Gregorio XIII, donde se introdujo un cambio en el calen-
dario que desplaza todo el calendario 10 das.
Para calcular el da es necesario saber algo de congruencias en teora
de n umeros.
Examinemos la fecha 12 de Octubre de 1492. La observacion impor-
tante es que si a una fecha le agregamos un n umero de das m ultiplo de
7 el da de la semana no cambia.
1
El da 18 de Septiembre fue un Martes
178 Teora de N umeros
Del 12 de Octubre de 1492 al 12 de Octubre de 2005 hay entonces 513
a nos. La cantidad de a nos bisiestos hasta ahora desde 1492 (el cual fue
bisiesto) es
2004 1492
4
= 128
Notese que el a no 1492 no se cuenta pues el 12 de Octubre es posterior
al 29 de Febrero. Ademas, debemos descontar los tres a nos seculares que
no son bisiestos, es decir, 1700, 1800, 1900.
Por lo tanto el n umero de das es N = 513 365 + 128 = 187373 .
Ahora, para identicar el da de la semana debemos calcular N modulo
7. Aplicando el algoritmo de la division se obtiene N = 187373 =
26767 7 + 4 , es decir, 187373 4 (mod 7) .
De acuerdo al calendario de 2005, el 12 de Octubre es un Mierc oles.
Por lo tanto del 12 de Octubre de 1492 al 12 de Octubre de 2005 han
transcurrido 7 k + 4 das. El da buscado corresponde al da 7 k .
Por lo tanto, de America sera un Martes. Tomando en cuenta que
1492 es anterior al cambio ya mencionado, podemos asegurar que Colon
descubrio America un da Viernes??.
4.15 Funcion
Denotemos por (n) la suma de los divisores positivos de n.
Teorema 4.30 Si n = p

1
1
p

2
2
p

k
k
es la descomposicion primaria de
n. Entonces
(n) =
p

1
+1
1
1
p
1
1

p

2
+1
2
1
p
2
1

p

k
+1
k
1
p
k
1
.
Sergio Plaza 179
Demostracion. Es claro que cada divisor positivo de n tiene la forma
p

1
1
p

k
k
, con 0
i

i
para i = 1, 2, . . . , k , y cada uno de esos
n umeros es un divisor de n. Por lo tanto
(n) =

1
=0

k
=0
p

1
1
p

k
k
=

1
=0
p

1
1

2
=0
p

2
2

k
=0
p

k
k
=
p

1
+1
1
1
p
1
1

p

2
+1
2
1
p
2
1

p

k
+1
k
1
p
k
1
.
Teorema 4.31 Si mcd (m, n) = 1 , entonces
(m n) = (m) (n) ,
es decir, es una funcion multiplicativa.
Demostracion: Inmediata del teorema anterior, pues mcd (m, n) = 1
signica que m y n no tienen factores comunes.
Ejemplo 4.44 Denamos g : N N por g(n) = 1 si n no es di-
visible por el cubo de un primo y g(n) = 0 en otro caso Cuales son
los valores de g(1) , g(3) , g(4) , g(27) y g(28) ? Pruebe que g es
multiplicativa.
Solucion. Tenemos que
g(n) =
_
_
_
1 si para todo primo p, p
3
| / n
0 otro caso.
Ahora, es claro que g(1) = g(2) = g(4) = 1 , g(27) = g(3
3
) = 0 , y
g(28) = g(2
2
7
1
) = 1 .
180 Teora de N umeros
Debemos probar g(m n) = g(m) g(n) si mcd(mn) = 1 .
Si m o n es divisible por el cubo de un primo, entonces tambien lo
es m n. Ahora, si m n es divisible por p
3
, entonces solo uno de
esos n umeros m y n puede ser divisible por p , pues ellos son coprimos.
Este hecho prueba que bien m o n debe ser divisible por p
3
.
Esto muestra que m n es divisible por el cubo de un primo si y solo
si uno de ellos m o n es divisible por el cubo de un primo.
Caso m n no es divisible por el cubo de un primo, tenemos que
g(m n) = 1 y g(n) = 1 , g(m) = 1 . Caso m n es divisible por el cubo
de un primo, entonces g(m n) = 0 y uno de ellos m o n es divisible
por el cubo de un primo, luego g(m) = 0 o g(n) = 0 (posiblemente
ambos) y tenemos g(m n) = g(n) g(m) = 0 .
4.16 La Funcion de Mobius
Denicion 4.3 La funcion de Mobius es la funcion aritmetica :
N N denida por (1) = 1 y para n > 1
(n) =
_
_
_
(1)
k
si n = p
1
p
k
, con p
i
primos distintos
0 otro caso.
Ejemplo 4.45 (6) = 1 , pues 6 = 2 3 (k = 2) , (30) = 1 , pues
30 = 2 3 5 (k = 3) , (12) = 0 ya que 12 = 2
2
3 , (250) = 0 , pues
250 = 5
3
2 .
Denicion 4.4 Si f es una funcion aritmetica, la suma de divisores
de f es
[D(f)](n) =

d|n
f(d) .
Sergio Plaza 181
Ejemplo 4.46 Sea f : N N denida por f(n) = n
2
. Entonces
[D(f)](n) =

d|n
d
2
.
As, por ejemplo,
[D(f)](12) =

d|12
d
2
= 1
2
+ 2
2
+ 3
2
+ 4
2
+ 6
2
+ 12
2
= 210 .
Lema 4.3
[D()](n) =

d|n
(d) =
_
_
_
1 si n = 1
0 otro caso.
Demostracion. La formula para n = 1 es trivial.
Supongamos que n > 1 y sea
n = p
r
1
1
p
r
2
2
p
r
k
k
la factorizacion de n en factores primos distintos. Veamos cuales son los
terminos no cero en la suma

d|n
(d) . Desde la denicion de , vemos
que esos terminos corresponden a los productos de potencias simples de
p
1
, . . . , p
2
y d = 1 . (Por ejemplo, (p
1
p
3
p
7
) y (p
2
p
8
) daran origen a
182 Teora de N umeros
terminos no cero en la suma

d|n
(d) , pero (p
2
3
p
8
) = 0) . Luego

d|n
(d) = 1 + ((p
1
) + +(p
k
)) + ((p
1
p
3
) +
+(p
k1
p
k
)) + +(p
1
p
2
p
k
)
= 1 +
_
k
1
_
(1) +
_
k
2
_
(1)
2
+ +
_
k
k
_
(1)
k
= (1 1)
k
= 0 .
Ejemplo 4.47 Supongamos que n = 24 . Los divisores de 24 son
1, 2, 3, 4, 6, 8, 12, 24 . Luego,

d|24
(d) = (1) +(2) +(3) +(4) +(6) +(12) +(24)
= 1 + (1) + (1) + 0 + 1 + 0 + 0
= 0 .
Denicion 4.5 Si f y g son funciones aritmeticas, su producto de
Dirichlet es
(f g)(n) =

d|n
f(d) g
_
n
d
_
.
Ejemplo 4.48 (fg)(12) = f(1)g(12)+f(2)g(6)+f(3)g(4)+f(4)g(3)+
f(6)g(2) +f(12)g(1) .
Denamos las funciones aritmeticas I, e : N N por I(n) = 1 para
todo n N, y
Sergio Plaza 183
e(n) =
_
_
_
1 si n = 1
0 otro caso.
Teorema 4.32 (Propiedades del producto de Dirichlet.) Sean f, g y
h funciones aritmeticas. Entonces
1. f g = g f
2. (f g) h = f (g h)
3. f e = f = e f
4. f I = Df = I f
5. I = e .
Demostracion. Para la propiedad 1 notemos que los divisores aparecen
a pares, d y
n
d
. Luego si
_
d,
n
d
_
es un par de divisores de n, entonces
tambien lo es
_
n
d
, d
_
. Esto signica que los mismos terminos ocurren
en las sumas
(f g)(n) =

d|n
f(d) g
_
d
n
_
y
(g f)(n) =

d|n
f
_
d
n
_
g(n) .
Luego esas suman son iguales.
La propiedad 2, es facil pero fastidiosa de vericar y se deja a cargo
del lector.
184 Teora de N umeros
Para ver la propiedad 3, notemos que
(f e)(n) =

d|n
f(d) e
_
n
d
_
= f(n)e(1) = f(n)
pues e
_
n
d
_
= 0 , excepto cuando
n
d
= 1 , es decir, d = n.
Para la propiedad 4, tenemos
(f I)(n) =

d|n
f(d) I
_
n
d
_
=

d|n
f(d) = [D(f)](n) .
Para la propiedad 5, comenzamos con n = 1 . Tenemos
( I)(1) = (1) I(1) = 1 1 = 1 = e(1) .
Supongamos que n > 1 . Entonces
( I)(n) = [D()](n) = 0 = e(n) .
Por lo tanto la formula vale para todo n.
Teorema 4.33 (formula de inversion de Mobius) Si f es una funcion
aritmetica, entonces f = [D(f)] .
Demostracion. Tenemos
[D(f)] = I [D(f)] = e f = f .
Lema 4.4 [D()](n) =

d|n
(n) = n.
Sergio Plaza 185
Demostracion. Sea n un entero positivo construyamos las fracciones
1
n
,
2
n
, . . . ,
n 1
n
,
n
n
.
Reducimos todas ellas. Consideremos una fraccion tpica reducida
a
b
.
Tenemos que d|n, pues provino de una fraccion cuyo denominador era
n, a < d , pues la fraccion original es menor que 1, y mcd (a, d) = 1,
pues la fraccion esta reducida.
Note que (yendo en la otra direccion) si
a
b
es un fraccion con numera-
dor y denominador positivo, la cual satisface d|n, a < d y mcd (a, d) =
1 , entonces es una de las fracciones reducidas. Pues si dk = n para
alg un k , y entonces
a
b
=
ka
kd
=
ka
n
, y la fraccion es una de las fracciones
originales.
Nos podemos preguntar cuantas de las fracciones reducidas tienen a
d como denominador? Como el numerador a es un n umero positivo
coprimo con d , vemos que existen (d) de tales fracciones. Sumando
sobre todo los d que dividen a n, obtenemos

d|n
(d) . Pero como
cada fraccion reducida tiene alguno de tales d como denominador, esta
suma toma en cuenta todas tales fracciones, y existen n de ellas. Por
lo tanto

d|n
(d) = n.
Ejemplo 4.49 Supongamos que n = 6 . Entonces

d|6
(d) = (1) +(2) +(3) +(6) = 1 + 1 + 2 + 2 = 6 .
Lema 4.5 Sea n 1 , entonces
(n) =

d|n
(d)
n
d
.
186 Teora de N umeros
Demostracion Por la formula de inversion de Mobius
(n) = ( [D()](n) =

d|n
(d)[D()]
_
n
d
_
=

d|n
(d)
n
d
.
Ejemplo 4.50 Para n = 6 , (6) = 2 . Ahora

d|6
(d)
6
d
= (1)
6
1
+(2)
6
2
+(3)
6
3
+(6)
6
6
= 1 6 + (1) 3 + (1) 2 + 1 1 = 2
Teorema 4.34 Para n 1 ,
(n) = n

p|n
p primo
_
1
1
p
_
.
(Por convencion, el producto vaco el producto sin terminos, es igual
a 1).
Demostracion. Para n = 1 , el resultado es inmediato por convencion.
Si n > 1 , sean p
1
, p
2
, . . . , p
k
los factores primos distintos de n.
Entonces,

p|n
p primo
_
1
1
p
_
=
_
1
1
p
1
__
1
1
p
2
_

_
1
1
p
k
_
= 1

j
1
p
j
+

i=j
1
p
i
p
j
+
(1)
k
p
1
p
2
p
k
(1)
Cada termino es 1/d , donde d es 1 (primer termino) o un producto
de primos distintos. Los (1)
i
cada uno de esos terminos alternan de
Sergio Plaza 187
signo de acuerdo al n umero de p lo cual exactamente lo que hace la
funcion de Mobius. Luego, la expresion (1) es

d|n
(d)
d
(2)
Esta suma puede ser tomada sobre todos los divisores, pues (d) = 0
si d tiene factores primos repetidos. Ahora, multiplicando (2) por n,
obtenemos
n

p|n
p primo
_
1
1
p
_
=

d|n
(d)
n
d
= (n) .
Ejemplo 4.51 Para 40 = 2
3
5 se tiene que (40) = 40
_
1
1
2
__
1
1
5
_
=
16 . Para 81 = 3
4
, tenemos (81) = 81
_
1
1
3
_
= 54 . Mas general,
si p es primo y k 1 , entonces
(p
k
) = p
k
p
k1
pues
(p
k
) = p
k
_
1
1
p
_
= p
k
p
k1
.
Denici on 4.6 Una funcion aritmetica f es multiplicativa si
f(m n) = f(m) f(n)
cuando mcd (m, n) = 1 .
188 Teora de N umeros
Lema 4.6 es multiplicativa.
Demostracion. Supongamos que mcd(m, n) = 1 . Ahora,
(m) = m

p|m
p primo
_
1
1
p
_
y
(n) = n

q|n
q primo
_
1
1
q
_
.
(m)(n)
m n
=
_
_
_
_
_

p|m
p primo
_
1
1
p
_
_
_
_
_
_
_
_
_
_
_

q|n
q primo
_
1
1
q
_
_
_
_
_
_
.
Como mcd (m, n) = 1 , esos dos productos no tienen factores primos
comunes.
Ademas, los primos que aparececen en esos productos son exactamente
los factores primos de m n. Luego
(m)(n)
m n
=

r|m n
r primo
_
1
1
r
_
.
Por lo tanto,
(m)(n) = m n

r|m n
r primo
_
1
1
r
_
.
Sergio Plaza 189
Teorema 4.35 Si n 3 , entonces (n) es par.
Demostracion. Si n tiene k factores primos impares, entonces 2
k
|(n) .
Para ver esto, observemos primero que
(2
k
) = 2
k
2
k1
,
es par si 2
k
4 .
Supongamos que n tiene k factores primos impares. Entonces
(n) = n

p|n
p primo
_
1
1
p
_
= n

p|n
p primo
_
p 1
p
_
=
n

p|n
p primo
p

p|n
p primo
(p 1) .
El denominador de la fraccion es el producto de los primos que dividen
a n, luego la fraccion es de hecho un entero. El segundo termino tiene
al menos un factor par para cada primo impar que divide a n. Luego,
el segundo termino y por lo tanto (n) es divisible por 2
k
.
Ejemplo 4.52 La descomposcion primaria de 7623 es 7623 = 3
2
7
11
2
, la cual tiene 3 factores primos impares, luego (7623) debe ser
divisible por 8. De hecho, (7623) = 3960 = 8495 . ??
190 Teora de N umeros
4.17 N umeros Perfectos.
Un n umero entero positivo es perfecto si es igual a la suma de sus divi-
sores propios. Mas precisamente, tenemos la siguiente denicion.
Denicion 4.7 Un n umero entero n > 0 es perfecto si (n) = 2n .
Es decir, n es perfecto si es igual a la suma de sus divisores propios.
Ejemplo 4.53 6 es perfecto pues 6= 1 + 2 + 3; 28 es perfecto, pues 28
= 1 + 2 + 4 + 7 + 14; tambien son perfectos los n umeros 496 y 8128.
Problema 4.1 (no resuelto a la fecha) No se sabe si existen n umeros
perfectos impares, o si existen innitos n umeros perfectos pares.
El nesimo n umero de Mersenne, M
n
, es denido por M
n
= 2
n
1 .
La existencia de innitos n umeros perfectos pares esta relacionada
a la existencia de innitos primos de Mersenne a traves del siguiente
resultado
Teorema 4.36 (Euclides) Un n umero entero positivo n es perfecto si
y solo si n = 2
k1
(2
k
1) , donde 2
k
1 es un primo de Mersenne.
Demostracion. Supongamos que 2
k
1 es primo de Mersenne, en-
tonces n = 2
k1
(2
k
1) es par. Como 2
k
1 , es un primo impar, es
Sergio Plaza 191
coprimo con 2
k1
. Luego
(n) = (2
k1
(2
k
1))
= (2
k1
)(2
k
1)
=
2
k
1
2 1

(2
k
1)
2
1
(2
k
1) 1
= (2
k
1)((2
k
1) + 1)
= (2
k
1)2
k
= 2 2
k1
(2
k
1)
= 2n.
Por lo tanto n es perfecto.
Recprocamente, supongamos que n es un n umero perfecto par. Que-
remos mostrar que n = 2
k1
(2
k
1) , donde 2
k
1 es un primo de
Mersenne.
Como n es par, podemos escribir n = 2
i
m, donde i 1 y m es
impar. Entonces
2
i+1
m = 2n
= (n)
= (2
i
m)
= (2
i
)(m)
= (2
i+1
1)(m) .
192 Teora de N umeros
Como 2
i+1
divide al lado izquierdo de estas igualdadeds, debe di-
vidir el lado derecho. Pero 2
i+1
1 es impar, luego debemos tener que
2
i+1
|(m) .
Armamos que 2
i+1
es la mayor potencia de 2 que divide a (m) .
Pues si 2
i+2
|(m) , entonces
2
i+1
m = (2
i+1
1)(m) = (2
i+1
1)2
i+2
k .
Luego, m = (2
i+1
1) 2

K , lo cual contradice el hecho que m es
impar.
Como 2
i+1
es la mayor potencia de 2 que divide a m, podemos
escribir (m) = 2
i+1
s , donde s es impar. Luego
2
i+1
m(2
i+1
1)(m) = (2
i+1
1)2
i+1
s ,
luego m = (2
i+1
1)s .
Tenemos que mostrar que s = 1 . Para hacer esto, comenzamos con
m = (2
i+1
1)s . Sumando s a ambos lados de esta igualdad nos queda
m+s = 2
i+1
s = (m) .
Luego m es divisible por 1, por si mismo, y por s (pues m = (2
i+1

1)s) . Si s = m, entonces
n = 2
i
m = 2
i
(2
i+1
1)s = 2
i
(2
i+1
1)m,
luego, 1 = 2
i+1
1 , esto implica que i = 0 , lo que es una contradiccion.
Ademas, si s > 1 , entonces 1 , s y m son tres divisores distintos de
m, luego
(m) m+s + 1 .
Sergio Plaza 193
Esto contradice el hecho que (m) = m+s . Por lo tanto, s = 1 .
Tenemos as que n = 2
i
(2
i+1
1) . Ahora debemos mostrar que
2
i+1
1 es primo. Como 1 y 2
i+1
1 son factores distintos de 2
i+1
1 ,
tenemos
2
i+1
= (m) = (2
i+1
1) 1 + (2
i+1
1) = 2
i+1
.
Por lo tanto, (2
i+1
1) = 2
i+1
, esto signica que 1 y 2
i+1
1 son
los unicos divisores de 2
i+1
, es decir, 2
i+1
1 es primo.
Ejemplo 4.54 Para i = 1 , 2
2
1 = 3 es primo, por lo tanto 2
1
(2
2

1) = 2 3 = 6 es perfecto. Para i = 2 , el primer primo, se tiene que


2
3
1 = 7 es primo, luego 2
2
(2
3
1) = 28 es perfecto. Para i = 3 ,
el segundo primo se tiene que 2
4
1 = 15 no es primo. Para n = 4 ,
se tiene que 2
5
1 = 31 es primo, luego 2
4
(2
5
1) = 496 es perfecto.
Para n = 6 , tenemos 2
7
1 = 127 es primo, luego, 2
6
(2
7
1) = 8128
es perfecto. El lector, puede hacer otros ejemplos.
Por el teorema 4.36 tenemos que encontrar n umeros perfectos pares es
equivalente a encontrar primos de Mersenne, es decir, primos de la forma
2
n
1 . Por otra parte, se tiene que si 2
n
1 es primo entonces n es
primo. Luego para buscar primos de Mersenne, necesitamos ver si para
n primo, el n umero 2
n
1 es primo. Vamos a mostrar un resultado
que simplica el problema de vericar si 2
n
1 es primo cuando n es
primro. Primero, mostramos el siguiente resultado.
Lema 4.7 Sean a y b enteros positivos. Entonces
mcd (2
a
1, 2
b
1) = 2
mcd (a,b)
1 .
194 Teora de N umeros
Demostracion. Sin perdida de generalidad podemos suponer que a
b . Como el maximo com un divisor de dos n umeros no cambia si restamos
el menor al mayor, tenemos
mcd (2
a
1, 2
b
1) = mcd ((2
a
1) (2
b
1), 2
b
1)
= mcd (2
a
2
b
, 2
b
1)
= mcd (2
b
(2
ab
1), 2
b
1) .
Como 2
b
1 es impar no tiene factores comunes con 2
b
en el primer
factor.
Luego,
mcd (2
b
(2
ab
1), 2
b
1) = mcd (2
ab
1, 2
b
1) .
Siguiendo este proceso, vemos que los exponentes convergen a mcd (a, b) .
Cuando el algoritmo termina, obtenemos
mcd(2
mcd (a,b)
1, 0) = 2
mcd (a,b)
1 .
Ejemplo 4.55 Como mcd (42, 54) = 6 , se tiene que
mcd (2
42
1, 2
54
1) = 2
6
1 = 63 .
Esto es ciertamente no obvio, en especial si calculamos 2
42
1 y 2
54
1
cuyos valores son 2
42
1 = 4398046511103 , 2
54
1 = 18014398509481983 .
Sergio Plaza 195
Teorema 4.37 Sea p un primo impar. Entonces cada factor de 2
p
1
tiene la forma 2kp + 1 para alg un k 0 .
Demostracion. Basta probar que el resultado es valido para factores
primos de 2
p
1 . En efecto,
(2ap + 1)(2bp + 1) = (2abp +a +b)p + 1 ,
es decir, el producto de n umeros de la forma 2kp + 1 tiene la misma
forma.
Supongamos entonces que q es un factor primo de 2
p
1 . El peque no
teorema de Fermat nos da que q|(2
p
1) . Por el lema 4.7, tenemos que
mcd (2
p
1, 2
q1
1) = 2
mcd (p,q1)
1 .
Ahora, q|(2
p
1) y q|(2
q1
1) implican que q|(2
mcd (p,q1)
1) . En
particular, 2
mcd (p,q1)
1 > 1 , pues es divisible por el primo q . Esto
implica que mcd (p, q1) > 1 . Ahora, p es primo, luego mcd (p, q1) >
1 es posible solo si mcd(p, q 1) = p . En particular p|(q 1) .
Escribamos q 1 = pt , esto es, q = pt +1, y tenemos que q es impar
y por lo tanto q 1 es par y pt es par.
Como p es impar, t debe ser par, es decir, t = 2k para alg un k .
Entonces q = 2kp + 1 , lo que termina la prueba.
Ejemplo 4.56 Es 2
17
1 = 131071 primo?
Como

131071 362 . Si 2
17
1 tiene un factor propio primo este
debe ser menor que 362, y los factores primos deben tener la forma
2k 17 + 1 = 34k + 1 .
196 Teora de N umeros
Luego debemos chequear los primos menores que 362 para ver si ellos
dividen a 131071.
k 34k + 1
1 35 no es primo
2 69 no es primo
3 103 103 es primo, pero no divide a 131071
4 137 137 es primo, pero no divide a 131071
5 171 no es primo
6 205 no es primo
7 239 239 es primo, pero no divide 131071
8 273 no es primo
9 307 307 es primo, pero no divide a 131071
10 341 no es primo
Luego 2
17
1 es primo.
Una conjetura es un resultado del que se tiene completo convencimiento
de que es verdadero, por m ultiples razones, pero no se tiene una prueba.
Por ejemplo se tiene las siguientes respecto de los n umeros perfectos.
Conjetura Existe una cantidad innita de primos de Mersenne (equiv-
alentemente, n umeros perfectos) y una cantidad innita de n umeros de
Mersenne no primos.
Conjetura No existen n umeros perfectos impares.
cambiar algunos problemas a la correspondi-
ente seccion o captulo
Sergio Plaza 197
4.18 Problemas
Problema 4.33 Escriba un programa (simple) que ponga en evidencia
la veracidad de las conjeturas anteriores, es decir, que encuentre una can-
tidad tan grande cuanto se desee de n umeros primos de Mersenne. En la
lista que obtendra vera tambien la evidencia para la segunda conjetura.
Nota. La b usqueda de primos de Mersenne es un hobby en la cual
cualquiera puede participar. Ayuda, codigos computacionales y resul-
tados pueden ser encontrados en GIMP (The Great Internet Mersenne
Primes Search)
http://www.mersenne.org/prime.htm
Encuentre otras paginas web similares a la indicada, existen muchas
y con bastante informacion.
Problema 4.34 Usando la formula de Euclides encuentre mas n umeros
perfectos.
Problema 4.35 Pruebe que no existe entero n > 1 que divide a 2
n
1 .
Problema 4.36 Encuentre todos los n umeros primos p, q tales que pq
divide a (5
p
2
p
)(5
q
2
q
) .
Problema 4.37 Sean p, q n umeros primos. Si q divide 2
p
+3
p
, pruebe
que q > p o q = 5 .
Problema 4.38 Escribamos los n umeros en el triangulo de Pascal en
la forma siguiente
Note que al sumar en diagonal, en cada diagonal, se obtienen los
n umeros 1, 1, 2, 3, 5, 8, 13, ... que no son otros que los primeros n umeros
de Fibonacci. Es posible probar este resultado en forma general?
198 Teora de N umeros
Hasta hoy no se sabe si existen n umeros perfecto impares. Este es un
problema famoso y difcil de la teora de n umeros. Por ejemplo, Brent,
Cohen, y Riele probaron que la cota inferior para un n umero perfecto
impar es 10
300
si existe. Brandstein mostro que el mayor factor primo
es mayor que 500000, y Sayer mostro que un n umero perfecto impar
tiene al menos 29 factores primos, no necesariamente distintos.
Problema 4.39 Recuerde que la sucesion de Fibonacci es dada por
(f
n
)
nN
donde f
n+1
= f
n
+f
n1
y f
0
= f
1
= 1 . Los primeros n umeros
de Fibonacci son 1, 1, 2, 3, 5, 8, 13, 21, 34, 55, 89, 144,... de entre ellos
vemos los siguientes primos 2, 3, 5, 13, 89,...
Encuentre otros primos en la sucesion de Fibonacci. Se conjetura que
existen innitos primos en la sucesion de Fibonacci (No resuelto a la
fecha).
Problema 4.40 Una manera de generar los n umeros de Fibonacci es
denir f
0
= 0 , f
1
= 1 y f
n+1
= f
n
+f
n1
, con n 1 . As
f
n
: 0, 1, 1, 2, 3, 5, 8, 13, 21, 34, 55, 89, 144, 239, 377, ...,
Note el siguiente hecho: 3|6 y f
3
|f
6
, 4|8 y f
4
|f
8
, 3|9 y f
3
|f
9
, 5|10
y f
5
|f
10
y 6|12 y f
6
|f
12
.
Podemos conjeturar entonces que k|n implica que f
k
|f
n
.
Para probarlo, demuestre primero la identidad
f
s+t
= f
s1
f
t
+f
s
f
t+1
.
Despues, tomando s = ny t = kn reemplazando nos queda
Sergio Plaza 199
f
n+kn
= f
(k+1)n
= f
n1
f
k
n +f
n
f
kn+1
De aqu, si f
n
|f
k
n entonces f
n
|f
(k+1)n
. Como es inmediato que
f
n
|f
n1
el resultado se sigue por induccion.
Problema 4.41 Sea f(n) = (n) n,donde (n) denota la suma de
los divisores de n. Por ejemplo, f(1) = 0 , f(2) = 1 + 2 2 = 1 ,
f(3) = 1 + 3 3 = 1 , f(4) = 1 + 2 + 4 4 = 3 , f(5) = 1 + 5 5 = 1 ,
f(6) = 1+2+3+66 = 6 , f(7) = 1 , f(8) = 1+2+4+88 = 7 , . . .
Note que si n es primo entonces f(n) = 1 , pues los divisores de n
son 1 y n.
Para n = 8 tenemos f(8) = 7 , f(7) = 1 y f(1) = 1 , si n = 9 ,
f(9) = 1 + 3 + 9 9 = 4 , f(4) = 3 , f(3) = 1 , y f(1) = 1 . Para
n = 10 , f(10) = 1 + 2 + 5 5 = 3 , f(3) = 1 y f(1) = 1 . Caso
n = 12 , f(12) = 1+3+4+6+1212 = 14 , f(14) = 1+7+1414 =
8 , f(8) = 7 , f(7) = 1 y f(1) = 1 . Haga mas ejemplos y formule
una conjetura respecto al calculo , n, f(n), f(f(n)), f(f(f(n))), . . . (Si
su conjetura es la siguiente, n, f(n)), f(f(n)), f(f(f(n))), . . . se vuelve
periodica. Entonces a ella no se le conoce si es verdadera o falsa. Por
ejemplo, f(95) = 25 , f(25) = 6 , f(6) = 6 , f(6) = 6 , . . . .
Problema 4.42 Sea a
n
= 6
n
+ 8
n
. Calcule el resto de la division de
a
83
por 49.
Problema 4.43 Si 30x0y03 es divisible por 13 encuentre x e y .
Problema 4.44 Pruebe que su 9|(a
3
+b
3
+c
3
) entonces 3|(abc) , para
enteros positivos a, b, c .
200 Teora de N umeros
Problema 4.45 Encuentre el ultimo dgito de 3
100
.
Problema 4.46 Pruebe que si 7|(a
2
+b
2
) entonces 7|a y 7|b .
Problema 4.47 Pruebe que para todo n, se tiene que n
9
6n
7
+9n
5

4n
3
Es divisible por 8640.
Problema 4.48 Pruebe que para cada entero positivo n se tiene que
(n + 1) (n + 2) (2n) es divisible por 2
n
.
Problema 4.49 Determine los ultimos dgitos de los n umeros en la
sucesion 23 , 23
23
, 23
(23
23
)
, . . .
Problema 4.50 Pruebe que si 3 d 2
n+1
, entoncecs d no divide a
(a
2n
+ 1) para todo entero positivo a .
Problema 4.51 Pruebe que si p es primo, entonces p
p
1 tiene un
factor primo que es congruente a 1 modulo p .
Un ejemplo menos trivial es el ISBN (international standar book num-
bers) codigo, por ejemplo el libro The Queen on Mathematics del autor
J. Goldman tiene el ISBN 1-56881-006-7; el primer dgito codica en pas
en el cual la editorial se encuentra: 0 para USA, 1 para el Reino Unido, y
3 para alemania. El siguiente grupo de dgitos da informacion acerca de
la compa na editorial, por ejemplo, 0-387 es para Springer Verlag New
York , 3-540 para Springer Verlag Heidelberg. El tercer grupo de dgitos
distingue los diferentes libros publicados por cada compa na editorial.
Luego, podemos explicar cada dgito en un ISBN, excepto el ultimo
dgito. El ultimo dgito no lleva consigo ninguna informacion, si no que
tiene por objetivo ser un dgito vericador cuando cualquier error ha sido
cometido en copiar el ISBN. Veamos como opera esto, supongamos que
Sergio Plaza 201
los dgitos de un ISBN son n
1
n
2
n
9
, si el codigo tiene solo 8 dgitos,
ponga n
9
= 0 . Calcule la suma N = n
1
+ 2n
2
+ + 9n
9
=
9

j=1
jn
j
,
enseguida reduzca el resultado mod 11, este es el ultimo dgito del ISBN.
202 Teora de N umeros
Captulo 5
Funciones aritmeticas y
sucesiones
Usualmente, cuando el dominio de la funcion es el conjunto Z o un
subconjunto de este, por ejemplo N, decimos que la funcion es una
funcion aritmetica.
Examinemos algunos ejemplos de tales fucniones aritmeticas.
5.1 Funcion
Denamos la siguiente funcion. A cada n umero natural n le asociamos
la suma de sus divisores positivos, es decir,
(n) = suma de los divisores de n.
Por ejemplo, (1) = 1 , (2) = 3 , (3) = 4 , (4) = 7 , (5) = 6 ,
(6) = 12 .
203
204 Teora de N umeros
Puesto que todo n umero natural posee una unica cantidad de divi-
sores, esta regla dene una funcion. Claramente, por denicion de primo,
se tiene que (n) = n + 1 si solo si n es primo.
Teorema 5.1 Si n = p
e
1
1
p
e
2
2
p
e
k
k
es la descomposicion primaria de
n, entonces
(n) =
p
e
1
1
1
p
1
1

p
e
1
2
1
p
2
1

p
e
k
k
1
p
k
1
.
Demostracion. Es claro que cada divisor positivo de n es de la forma
p
f
1
1
p
f
2
2
p
f
k
k
, donde 0 f
i
e
1
para i = 1, . . . , k , y cada tal n umero
es divisor de n. Por lo tanto,
(n) =
e
1

f
1
=0

e
k

f
k
=0
p
f
1
1
p
f
k
k
=
e
1

f
1
=0
p
f
1
1

e
k

f
k
=0
p
f
k
k
=
p
e
1
1
1
p
1
1

p
e
1
2
1
p
2
1

p
e
k
k
1
p
k
1
.
Usando este resultado podemos demostrar que la funcion es multi-
plicativa, es decir, tenemos el siguiente resultado.
Teorema 5.2 La funcion : N N es multiplicativa, en otras pala-
bras, (n m) = (n) (m) , cuando mcd(m, n) = 1 .
Demostracion. De lo anterior se sigue inmediatamente que si mcd(m, n) =
1 , entonces (m n) = (m)(n) , es decir, es una funcion aritmetica
multiplicativa.
Sergio Plaza 205
Teorema 5.3 (Euler) Si p = 2
n
1 es primo entonces
t
n
=
1
2
p(p + 1) = 2
n1
(2
n
1)
es un n umero perfecto. Ademas, cada n umero perfecto par es de esta
forma.
Demostracion. Se sigue de lo anterior que
(t
n
) =
2
n
1
2 1

p
2
1
p 1
= (2
n
1) (p + 1) = p(p + 1).
Ahora sea a un n umero perfecto par. Supongamos que a = 2
n1
u ,
con u > 1 impar. Tenemos entonces que 2
n
u = 2a = (a) =
2
n
1
21
(u) ,
lo que implica que (n) =
2
n
u
2
n
1
= u +
u
2
n
1
. Notando que u y
u
2
n
1
son divisores de u , obtenemos que u es primo y
u
2
n
1
= 1 , es decir,
u = 2
n
1 .
Hasta hoy no se sabe si existen n umeros perfecto impares. Este es un
problema famoso y difcil de la teora de n umeros. Por ejemplo, Brent,
Cohen, y te Riele probaron que la cota inferior para un n umero perfecto
impar es 10
300
si existe. Brandstein mostro que el mayor factor primo
es mayor que 500000, y Sayer mostro que un n umero perfecto impar
tiene al menos 29 factores primos, no necesariamente distintos.
Ejemplo 5.1 Denamos la funcion : N N por (n) =n umero de
primos que dividen a n. Probar que n 2
(n)
.
Solucion. Aplicando el T.F.A. se tiene que n = p

1
1
p

k
k
, donde k
es el n umero de primos que divide a n, luego (n) = k . Como p
i
2
para todo i = 1, . . . , k se tiene que
n = p

1
1
p

2
2
p

k
k
2

1
2

2
2

k
= 2

1
++
k
206 Teora de N umeros
Luego
n 2

1
+
2
++
k
.
Como cada
i
1, se tiene que 2

i
2
1
, para todo i = 1, . . . , k .
Por lo tanto,
n 2

1
++
k
2
1++1
= 2
k
= 2
(n)
.
5.2 Funcion de Euler
A todo n umero n N le asociamos la cantidad de n umeros positivos
coprimos con n que no sean mayores que n. Denotamos por a tal
funcion, en otras palabras : N N es dada por
(n) =

d|n
mcd (d, n) = 1
d .
Claramente, ella dene una funcion aritmetica, puesto que a cada
n umero natural se le asocia un unico n umero que, en este caso, es
tambien un elemento de N.
Por ejemplo
(1) = 1, (2) = 1, (3) = 2, (4) = 2, (5) = 4, (6) = 2, . . .
Teorema 5.4 Si n N es primo. Entonces (n) = n 1
Sergio Plaza 207
Demostracion. Sea n N un n umero primo. Entonces todo n umero
natural menor que n es coprimo con n. De donde obtenemos que,
(n) = n 1 si y solo si n es primo.
A continuacion veremos una manera de calcular (n) . Empecemos
con (21) . La cantidad de enteros positivos menores o iguales a 21
es 21. Por otro lado, sabemos que 21 posee como divisores a 1, 3,
7 y 21. Luego los m ultiplos de ellos que no sobrepasan a 21 quedan
descartados. Ademas la cantidad de estos m ultiplos puede ser calculada
de las siguiente manera. Hay tantos m ultiplos de 3 como 21/3 = 7 , hay
tantos m ultiplos de 7 como 21/7 = 3 , y as sucesivamente, es decir,
(21) = 21
21
3

21
7
+
21
21
,
donde el ultimo factor (que es 1) debe agregarse, puesto que 21 fue
sacado dos veces.
Ahora tratemos de aplicar el mismo argumento a un n umero natural
n = p

, con p y q n umeros primos. Contemos primero los m ultiplos


de p y q . Tenemos, los m ultiplos de p son
p

p
y los m ultiplos de
q son
p

q
. Pero los m ultiplos de np son m ultiplos de p y de q si-
multaneamente, de modo que estan contados dos veces, por lo tanto
(p

) = p

p

p

q
+
p

pq
= p

_
1
1
p

1
q
+
1
pq
_
= p

_
1
1
p
__
1
1
q
_
.
La formula para un n umero n arbitrario se obtiene a partir de la
descomposicion primaria de n.
208 Teora de N umeros
Teorema 5.5 Si n = p

1
1
p

2
2
p
r
r
es la descomposicion primaria
de n, entonces
(n) = n
_
1
1
p
1
__
1
1
p
2
_

_
1
1
p
r
_
.
Ejemplo 5.2 Para n = 570 , su descomposicion primaria es dada por
n = 2 5 57 . Aplicando el teorema 5.5, obtenemos
(570) = 570
_
1
1
2
__
1
1
5
__
1
1
57
_
= 224 ,
es decir, existen 224 n umeros enteros positivos menores o iguales que
570 y que son coprimos con 570.
Ejemplo 5.3 Para n = 660 , su descomposicion primaria es dada por
n = 2
2
3 5 11 . Aplicando el teorema 5.5, obtenemos que
(660) = 660
_
1
1
2
__
1
1
3
__
1
1
5
__
1
1
11
_
,
y calculando cada termino se obtiene
(660) = 660
_
1
2
__
2
3
__
4
5
__
10
11
_
= 160.
Para nalizar esta seccion daremos la formula para un caso especial.
El metodo usado en la demostracion es interesante de recordar.
Teorema 5.6 Si p es un primo y k es un entero positivo, entonces
(p
k
) = p
k
p
k1
.
Sergio Plaza 209
Demostracion. Es evidente que p no divide a n si y solo si mcd (n, p
k
) =
1. Ademas, hay p
k1
enteros entre 1 y p
k
que son divisibles por p, , es-
tos son p , 2p , 3p, . . . , p
k1
p . Luego el conjunto {1, 2, . . . , p
k
} contiene
exactamente p
k
p
k1
enteros que son coprimos con p
k
.
Una operacion natural entre funciones es llamada composicion. Esta
operacion se dene de la siguiente manera. Dadas dos funciones, f y
g , de tal manera que el recorrido de f este contenido en el dominio de
g , entonces podemos construir una nueva funcion, que se denota por
g f, , cuyo dominio es el de f , denida por la regla de que a cada
imagen mediante f se le aplica la regla de g . Mas precisamente, si D
es el conjunto dominio de f y a es cualquier elemento de D, entonces
su imagen f(a) debe estar en el dominio de g , luego le aplicamos g a
f(a) , es decir, calculamos g(f(a)) .
Claramente esta denicion asegura que el recorrido de g f es un
subconjunto del recorrido de g .
En general, g f es diferente de f g . Mas a un, es posible que
la primera exista, mientras que la segunda no tenga sentido alguno.
Examinemos los siguientes ejemplos.
Ejemplo 5.4 Composicion de y .
Las funciones y denidas arriba poseen como dominio todos
los n umeros enteros no negativos y sus recorridos son subconjuntos de
los n umeros enteros. Luego podemos formar y tambien .
Calculemos algunos valores de ellas. Por ejemplo,
( )(1) = ((1)) = (1) = 1
( )(2) = ((2)) = (1) = 1
( )(3) = ((3)) = (2) = 3
210 Teora de N umeros
( )(4) = ((4)) = (2) = 3
( )(6) = ((6)) = (3) = 4
por otra parte,
( )(1) = ((1)) = (1) = 1
( )(2) = ((1)) = (3) = 2
( )(3) = ((3)) = (4) = 2
( )(4) = ((4)) = (7) = 6
( )(6) = ((6)) = (4) = 6
Desde los calculos anteriores, claramente vemos que es diferente
de .
5.3 Sucesiones
En el captulo XXXX formalizaremos el concepto de n umero racional.
Por ahora entenderemos por n umero racional una fraccion con nume-
rador un n umero entero y denominador un n umero entero diferente de
cero.
Una sucesion de n umeros racionales es una funcion cuyo dominio es
el conjunto de los n umeros naturales y su recorrido un subconjunto de
los n umeros racionales. Es costumbre denotar una sucesion describiendo
el recorrido de ella de la manera siguiente (x
n
)
nN
o simplemente por
(x
n
)
n
, es decir, una sucesion es una funcion
x : N Q,
donde x
n
denota la imagen del n umero natural n mediante la funcion
x, esto es, x
n
= x(n) .
Sergio Plaza 211
Por ejemplo, (x
n
)
nN
, donde x
n
= 1/n , representa la sucesion de
n umeros racionales {1, 1/2, 1/3, 1/4, . . .} .
Ejemplo 5.5 Progresiones aritmeticas
Consideremos una sucesion (x
n
)
nN
, donde los elementos x
n
se for-
man de la manera siguiente
x
0
= a, x
1
= a +d, x
2
= a + 2d, . . . , x
n
= a +nd, . . .
es decir, x(n) = a +nd .
Decimos, en este caso, que los n umeros x
1
, x
2
, . . . , x
n
, . . . se encuen-
tran en progresion aritmetica. Otra manera de decir esto es que la
diferencia de dos terminos consecutivos cualesquiera de la sucesion es
constante.
Ejemplo 5.6 Los n umeros 3, 5, 7, 9, . . . estan en progresion aritmetica,
puesto que la diferencia de dos terminos consecutivos es constante e
igual a 2. En este caso se tiene que a = 3 y d = 2 . En otras palabras,
x(n) = x
n
= 3 + 2n, para n = 0, 1, . . .
Ejemplo 5.7 Progresiones geometricas
En el caso en que los n umeros dados por la sucesion (x
n
)
nN
se rigen
por la ley de formacion
x
0
= a, x
1
= ar, x
2
= ar
2
, . . . , x
n
= ar
n
, . . .
es decir, x(n) = ar
n
.
Decimos que los n umeros x
1
, x
2
, . . . , x
n
, . . . se encuentran en pro-
gresion geometrica, es decir, si las razones x
n+1
/x
n
de dos terminos
consecutivos de la sucesion son iguales para todo n. A r = x
n+1
/x
n
se
le llama la razon de la progresion.
212 Teora de N umeros
Ejemplo 5.8 Los n umeros 3, 9, 27, 81, . . . estan en progresion geometrica,
puesto que x
n+1
/x
n
= 3 . En este ejemplo a = 1 y r = 3 . En otras
palabras, x(n) = x
n
= 3
n
, para n = 0, 1, 2, . . . .
Ejemplo 5.9 Sucesiones de Fibonacci.
Consideremos la sucesion de n umeros enteros (f
n
)
nN
denida como
sigue
f
0
= a, f
1
= b, f
2
= x
1
+x
0
, . . . , f
n+2
= f
n+1
+f
n
, para n 0 ,
es decir, f
n
, el termino nesimo, es la sucesion de Fibonacci. Cuando
a = b = 1 , tenemos la sucsion de Fibonacci clasica {1, 1, 2, 3, 5, 8, 13, . . .} .
Una sucesion (x
n
)
nN
puede tener un recorrido nito, como en el caso
x
n
= (1)
n
. Claramente esta sucesion asocia a todo n umero natural
par el 1 y a todo impar el 1 .
Las funciones aritmeticas y tambien se pueden interpretar como
sucesiones de n umeros enteros, donde x
n
= (n) y x
n
= (n) .
5.4 Problemas
Problema 5.1 Pruebe que para todo entero positivo n y todo entero
a > 1 , se tiene que n|(a
n
1) .
Problema 5.1 Existen innitos enteros n para los cuales (n) =
n
2
?
Lo mismo para (n) =
n
3
y (n) =
n
4
.
Problema 5.2
Problema 5.3
Problema 5.4
Sergio Plaza 213
Problema 5.5
Problema 5.6
Problema 5.7
Problema 5.8
Problema 5.9
Problema 5.10
Problema 5.11
214 Teora de N umeros
Captulo 6
Representaciones
Numericas y
Aproximaciones
6.1 Representacion decimal
En esta seccion construiremos un algoritmo para lograr aproximar n umeros
reales por n umeros racionales. Primero estudiaremos la representacion
decimal de un n umero real.
Consideremos el conjunto D = {0, 1, 2, 3, 4, 5, 6, 7, 8, 9} , el cual lla-
maremos conjunto de dgitos.
Comencemos con un n umero natural n. Se quiere expresar n como
una suma de potencias de 10 y coecientes (dgitos) en D, esto es,
queremos escribir
n = d
0
+d
1
10 +d
2
+ +d
n
10
N
=
N

i=0
d
i
10
i
,
215
216 Teora de N umeros
donde los coecientes d
i
, con i = 0, 1, . . . , N, son elementos en D. En
el argumento que a continuacion explicamos aplicaremos reiteradas veces
el algoritmo de la division y no lo mencionaremos en forma explcita cada
vez que lo utilicemos.
Primero, si 0 n 9 , se tiene que n = d
0
10
0
; basta, pues,
con tomar d
0
= n. Ahora bien, si 10 n < 10
2
podemos escribir
n = d
1
10
1
+r
1
, donde d
1
D y 0 r
1
< 10 . Por lo tanto, r
1
puede
ser escrito como r
1
= d
0
10
0
, con d
0
= r
1
; luego
n = d
1
10
1
+d
0
10
0
, con d
0
, d
1
D.
Si 10
2
n < 10
3
, tenemos que n = d
2
10
2
+ r
2
, con d
2
D y
0 r
2
< 10
2
. Si 0 r
2
< 10 , entonces r
2
= d
0
10
0
, donde d
0
= r
2
;
tomando d
1
= 0 podemos escribir
n = d
0
10
0
+d
1
10
1
+d
2
10
2
, con d
0
, d
1
, d
2
D.
Por otra parte, si 10 r
2
< 10
2
, con r
2
= d
1
10
1
+ r
1
, donde
d
1
D y 0 r
1
< 10 , luego, tomando d
0
= r
1
, obtenemos tambien
que

3
i=0
d
i
10
i
, con d
1
, d
2
, d
3
D.
Puesto que, para todo n umero natural n, existe tal que 10

n < 10
+1
aplicando en metodo descrito n se puede escribir en la forma
dada. En resumen, hemos probado que cada n umero natural n se puede
expresar como una suma de potencias de 10 y coecientes (dgitos) en
D. Esta representaci on es llamada representacion decimal (o en base
10) de n.
Sergio Plaza 217
El mismo tipo de representacion, mediante una suma nita, para un
n umero real x, con 0 x < 1 , ya no es posible, como se muestra en el
siguiente ejemplo.
Ejemplo 6.1 Consideremos el n umero racional 2/3 . Tenemos
2
3
= 0.6 + 0.06 + 0.006 +
=

j=1
6 10
j
= 6

j=1
10
j
.
Para ver si la suma con innitos terminos de la ultima igualdad repre-
senta un n umero real, debemos detrminar su convergencia. Mas general,
consideremos una suma de la forma
S = q +q
2
+q
3
+ +q
j
+ =

j=1
q
j
con innitos terminos, donde q R es distinto de cero. Para determi-
nar su convergencia, consideremos la sucesion asociada a ella, es decir, la
sucesion de sumas parciales S
n
=

n
j=1
q
j
, con n = 1, 2, . . . . Multipli-
cando S
n
por q , obtenemos la sucesion qS
n
= q
2
+q
3
+ +q
n
+q
n+1
.
Restando qS
n
de S
n
, obtenemos,
S
n
(1 q) = q q
n+1
de donde
S
n
=
q(1 q
n
)
1 q
que tiene como lmite
q
1q
cuando n siempre y cuando |q| < 1 .
En conclusion
si |q| < 1 entonces

j=1
q
j
=
q
1 q
.
218 Teora de N umeros
Ahora, la convergencia de la serie innita

j=1
6 10
j
es inmediata,
puesto que es una serie geometrica de razon 1/10 y su suma es 2/3 .
A continuacion construiremos una representacion decimal para los
n umeros reales x, con 0 x < 1 . Denotaremos a este conjunto por el
smbolo [0, 1[ , y geometricamente lo representaremos por el segmento
de recta
1
10
1
10
5
10
1
10
4
10
1
10
3
10
1
10
2
10
1
10
7
10
1
10
9
10
1
10
8
10
1
10
6
10
1
10
En otras palabras, a cada punto de la recta corresponde un elemento
de [0, 1[ . Dado x [0, 1[ , queremos representarlo como
x =

i=1
k
i
10
i
,
donde k
i
D para cada i 1 . Para obtener esta suma dividamos
el intervalo [0, 1[ en 10 partes iguales, como se muestra en la gura
anterior.
Sea k
1
el mayor elemento en D tal que
k
1
10
x <
k
1
+ 1
10
,
entonces
x =
k
1
10
+r
1
, con 0 r
1
<
1
10
.
Si r
1
= 0 detenemos el proceso y, tomando k
i
= 0 para cada i 2 ,
se tiene lo pedido. Por otra parte, si 0 < r
1
< 1/10 dividimos, a su vez,
el intervalo [0, 1/10[ en 10 partes iguales.
Sergio Plaza 219
1
100
1
10
5
100
1
10
4
100
1
10
3
100
1
10
2
100
1
10
7
100
1
10
9
100
1
10
8
100
1
10
6
100
1
10
1
10
1
10
Denotemos por k
2
el mayor elemento en D para el cual
k
2
10
2
r
2
<
k
2
+ 1
10
2
.
Si r
2
= k
2
/10
2
detenemos el proceso y tomamos k
i
= 0 para i 3 ,
con lo cual
x =

i=1
k
i
10
i
.
Por el contrario si k
2
/10
2
< r
2
< (k
2
+ 1)/10
2
, podemos escribir
r
2
=
k
2
10
2
+r
3
, con 0 r
3
<
1
10
2
.
Repetimos ahora el proceso con r
3
y as sucesivamente. De este modo
obtenemos que x puede escribirse como
x =

i=1
k
i
10
i
,
donde los coecientes k
i
D, para i = 1, 2, 3, . . .
Como en el Ejemplo anterior de la representacion de x = 2/3 , el
problema se reduce a examinar si la serie del lado derecho de esta ultima
igualdad es convergente.
Para mostrar esto notemos primero que k
i
/10
i
9/10
i
para cada
i 1 . Sea q
n
=

n
i=1
k
i
10
i
una suma parcial de la serie

i1
k
i

220 Teora de N umeros


10
i
y sea G
n
la correspondiente suma parcial de la serie geometrica

i1
9 10
i
. Aplicando la formula (5.2)?, y siendo los coecientes
de la serie geometrica positivos, se obtiene que
q
n
G
n

i=1
9 10
i
= 1,
lo cual muestra que la sucesion (q
n
)
nN
es acotada superiormente por
1.
Ademas la sucesion de sumas parciales (q
n
)
nN
es creciente, pues
cada vez estamos sumando terminos no negativos (mayores o iguales
que cero). Aplicando la Proposicion 5.3? se concluye que (q
n
)
n
es
convergente, esto es, la serie

i1
k
i
10
i
es convergente y su suma
x =

i=1
k
i
10
i
es un n umero real en el intervalo [0, 1] .
Teorema 6.1 Dado un n umero real x , con 0 x 1 y un n umero
> 0 existe un n umero racional q

tal que |x q

| < .
Consideremos el desarrollo decimal de x, esto es, escribamos x =

i=1
k
i
10
i
. Denamos para cada n umero natural n el n umero
x
n
=
n

i=1
k
i
10
i
.
Es claro que cada x
n
es un n umero racional (pues es una suma nita
de n umeros racionales). Ademas |x x
n
| satisface
|x x
n
| =

i=n+1
k
i
10
i

i=n+1
9 10
i
= 10
n
,
Sergio Plaza 221
de donde se deduce que |x x
n
| se aproxima a cero cuando n crece
denitivamente. Mas precisamente, eligiendo n
0
tal que 1/10
n
0
< y
deniendo q = x
n
0
=
n
0

i=1
k
i
10
i
, se obtiene lo pedido.
Tenemos, as que dado un n umero real x en el intervalo [0,1] hemos
construido una sucesion de n umeros racionales que aproxima a x.
Finalmente, veremos que si tenemos un n umero real x 1 tambien
podemos construir estas aproximaciones mediante n umeros racionales.
Para ello, reduciremos el problema al caso 0 x < 1 .
Escribamos x = [x] + (x) para x 1 . Como [x] es un n umero
natural el se puede representar de la forma
[x] =
N

i=0
d
i
10
i
, d
i
D,
con N el menor natural tal que 10
N
[x] < 10
N+1
. Por otra parte,
como 0 (x) < 1 , sabemos que
(x) =

j=1
k
j
10
i
, con k
j
D, j 1.
En resumen, x se puede representar como
x =
N

i=0
d
i
10
i
+

j=1
k
j
10
j
.
La primera suma es la representacion decimal del n umero natural [ x ]
y la segunda suma (que una serie) es la representacion decimal de la parte
fraccionaria (x) de x. Ahora, para cada n umero natural n denamos
x
n
= [x] +
n

i=1
k
i
10
i
.
222 Teora de N umeros
Cada x
n
es un n umero racional y en forma analoga al caso anterior
se demuestra que x
n
se aproxima cada vez mas a x cuando n crece
indenidamente.
Esta propiedad de los n umeros racionales en los n umeros reales es
llamada la densidad de los racionales en los reales.
Ejemplo 6.2 El n umero

2 = 1, , 414213... se puede escribir en la


forma

2 = 1 10
0
+
4
10
+
1
10
2
+
4
10
3
+
2
10
4
+
1
10
5
+
3
10
6
+
De este modo, utilizando la representacion (5.4)?, podemos escribir
cada n umero real positivo en su forma decimal y obtener de este modo
aproximaciones por n umeros racionales.
En general, la representacion decimal de un n umero no es unica, como
lo ilustra el siguiente ejemplo.
Ejemplo 6.3 El n umero 1 puede escribirse como
1 = 0 10
0
+

j=1
9
10
= 1 10
1
+

j=2
0
10
j
.
Cuando el denominador de la fraccion irreducible p/q no es potencia
de 10, la representaci on decimal de esta es periodica. Por otra parte, la
perdida de unicidad en la representacion decimal de un n umero real x
ocurre cuando x es de la forma p/q , con q una potencia de 10. Ob-
servemos tambien que un n umero irracional tiene representacion decimal
no periodica.
Sergio Plaza 223
6.2 Representacion en base p , p > 1
En la seccion anterior estudiamos la representacion decimal, esto es, en
base 10 de los n umeros reales no negativos. Ahora trataremos de imitar
tal construccion tomando como base un n umero natural p > 1 en vez
de la base 10 ya considerada.
Como en el caso anterior, comenzamos por denir nuestro conjunto
de dgitos D = {0, 1, 2, . . . , p 1} . Primero buscamos la representacion
en base p para los n umeros naturales, es decir, dado un n umero natural
n queremos representarlo como una suma nita de potencias de p y
coecientes en el conjunto D, esto es, expresar n como
n =
N

i=0
k
i
p
i
= k
0
+k
1
p + +k
N
p
N
,
donde para i = 0, 1, . . . , N los coecientes k
i
son elementos de D.
Para lograrlo procedemos en forma similar al caso de la representacion
decimal y aplicamos el logaritmo de la division con p en vez de 10.
Imitando lo realizado para p = 10 , bastara lograr dicha representacion
para los n umeros reales x en intervalo [0, 1[ . As, dividimos los inter-
valos [0, 1/p
n
] ( n 0) en p partes iguales. Siguiendo las mismas
directrices utilizadas para el caso p = 10 , se obtiene la representacion
requerida. Se concluye, en denitiva, que todo real x > 1 posee la
representacion
x =
N

j=0
d
j
p
j
+

i=1
k
i
p
i
.
La convergencia de la serie 6.1 en la ultima igualdad esta garantizada,
pues se compara con la serie geometrica de razon 1/ p ( p > 1 ).
224 Teora de N umeros
6.2.1 Representacion Binaria
Esta representacion consiste en tomar p = 2 y D = {0, 1} . En este
caso todo n umero real positivo x es representable como
x =
N

i=0
d
i
2
i
+

j=1
k
j
2
j
, con d
i
, k
i
D
Ejemplo 6.4
137
256
= 1 2
1
+0 2
2
+0 2
3
+0 2
4
+1 2
5
+0 2
6
+0 2
7
+1 2
8
.
Observemos que cualquier n umero real x en [0, 1[ tiene asociado una
sucesion de ceros y unos. Por ejemplo, a
137
256
se le asocia k
1
= 1, k
2
=
0, k
3
= 0, k
4
= 0, k
5
= 1, k
6
= 0, k
7
= 0, k
8
= 1 , y k
j
= 0 , para
j 9 .
Adaptando para la sucesion anterior la escritura 10001001 , tenemos
que el smbolo 1000100 representa el n umero
137
256
.
As, los smbolos 001 y 01011 representan los n umeros racionales
1
8
= 0 2
1
+ 0 2
2
+ 1 2
3
,
19
32
= 0 2
1
+ 1 2
2
+ 0 2
3
+ 1 2
4
+ 1 2
5
.
Un n umero irracional debe tener innitos unos en su expresion binaria
(de otra forma representara un n umero racional) y sus ceros y unos care-
cer de toda periodicidad. Es as como los smbolos 01001000100001 . . . ,
110111011110111110 . . . representan n umeros irracionales. El lector puede
intentar calcularlos.
Sergio Plaza 225
Para las computadoras, calculadoras y relojes analogicos los n umeros
son objetos de diferentes longitudes con ceros y unos. En particular,
como la longitud de los smbolos que estas maquinas pueden calcular
es nita (dependiendo de la capacidad de cada una), concluimos que
ellas trabajan solamente con n umeros racionales. Para maquinas que
procesan con 8 y 13 dgitos los resultados que generan para el n umero
irracional

2 son 1, 4142135 y 1, 414213562373 , respectivamente. Ob-


viamente, por lo que ya sabemos, estos valores son solo aproximaciones
racionales de

2 .
En las representaciones de n umeros reales expuestas en estas notas
hemos supuesto que tanto la base p (p > 1) y los dgitos D utilizados
son n umeros naturales. La verdad es que esto solo sirvio para simplicar
la exposicion y los calculos. En general, podemos construir representa-
ciones de los n umeros reales usando una base cualquiera p, con |p| > 1 y
un conjunto nito de dgitos D = {d
1
, d
2
, . . . , d
k
} . La condicion |p| > 1
es necesario para garantizar la convergencia de las series geometricas que
aparecen en tal caso.
6.2.2 Representacion triadica
Esta representacion consiste en tomar p = 3 , y por lo tanto, el conjunto
de dgitos D = {0, 1, 2} . En este caso todo n umero real positivo x es
representable como
x =
N

i=0
d
i
3
i
+

j=1
k
j
3
j
, con d
i
, k
i
D.
Por ejemplo,
38
81
= 1 3
1
+ 2 3
2
+ 2 3
3
0 3
4
+ + 0 3
n
+
226 Teora de N umeros
Aqu los coecientes son k
1
= 1 , k
2
= 2 , k
3
= 2 y k
j
= 0 para
j 3 .
Otros ejemplos de representaciones triadicas son
15 = 0 3
0
+ 2 3
1
+ 1 3
2
9
7
= 2 3
1
+ 1 3
2

2 = 1 3
0
+ 1 3
1
+ 0 3
2
+ 0 3
3
+ 2 3
4
+ (no periodica)
2
9
= 0 3
1
+ 2 3
3
= 1 3
1
+ 2 3
2
+ 2 3
3
+ + 2 3
k
+
Calcularemos en detalle el siguiente ejemplo
7
8
= 2 3
1
+ 1 3
2
+ 2 3
3
+ 1 3
4
+ 2 3
5
+ ,
donde los coecientes de subndice impar son iguales a 2 y los con
subndice par son iguales a 1. Para probar esta ultima igualdad procede-
mos a partir la serie en dos series, en una que agrupan los coecientes
par y en otra los coecientes impares. Aplicando la formula (5.2) se
tiene entonces que

j=0
2 3
(2j+1)
+

j=0
1 3
2j
=
2
3

j=0
9
j
+

j=1
9
j
=
2
3

9
8
+
1
8
=
7
8
.
En general, un n umero real x tiene una representacion nita en base
3, es decir,
x =
N

i=0
d
i
3
i
+
M

j=1
k
j
3
j
,
si y solo si x es de la forma m/3
n
, donde n y m son enteros positivos.
Sergio Plaza 227
Notemos que si el denominador de la fraccion irreducible p/q no es
una potencia de 3, la representacion en base 3 de p/q es periodica. Por
otra parte, n umeros irracionales poseen representaciones en base 3 no
periodicas.
Ejemplo 6.5
1
4
= 0 3
1
+ 2 3
2
+ 3
3
+ 2 3
4
+
Aqu los coecientes con ndice impar son cero y los coecientes con
ndice par son iguales a 2.
Ejemplo 6.6
1
7
= 0 3
1
+ 1 3
2
+ 0 3
3
+ 2 3
4
+ 1 3
5
+ 2 3
6
+ 0 3
7
+
El bloque formado por los coecientes k
1
= 0 , k
2
= 1 , k
3
= 0 ,
k
4
= 2 , k
5
= 1 , k
6
= 2 y k
7
= 0 en la expresion anterior se repite
periodicamente.
Ejemplo 6.7 Al igual que en el caso en base 10, cada n umero real
tiene una representacion en la forma (5.7)? y existen n umeros para
los cuales se tienen al menos dos representaciones distintas. Por ejemplo:
1
3
=
1
3
+
0
3
+
0
3
+ =
0
3
+
2
3
+
2
3
+
228 Teora de N umeros
6.3 Ejemplos
Ejemplo 6.8 Muestre que para cualquier entero n 1 , la suma del
n umero de dgitos de 4
n
y de 25
n
es un entero impar.
Solucion. Sean a
n
= 4
n
y b
n
= 25
n
. Tenemos as a
n
b
n
= 100
n
=
10
2n
. Sean r el n umero de dgitos de b
n
. Luego, 10
r1
a
n
< 10
r
y 10
s1
b
n
< 10
s
, por lo tanto 10
r+s2
a
n
b
n
< 10
r+s
. Por otra
parte, 10
2n
es una potencia de 10, luego debemos tener 10
2n
= 10
r+s1
,
de donde r +s 1 = 2n, es decir, r +s es impar.
Ejemplo 6.9 Encuentre todos los n umeros naturales x tales que el
producto de sus dgitos, en notacion decimal, es igual a x
2
10x 22 .
Solucion. Supongamos que x tiene la forma x = a
0
+a
1
10 +a
2
10
2
+
+a
n1
10
n1
, donde 0 a
k
9 y a
n1
= 0 .
Sea P(x) el producto de los dgitos de x, entonces debemos tener
que P(x) = x
2
10x 22 . Ahora, P(x) = a
0
a
1
. . . a
n1
9
n1
a
n1
<
10
n1
a
n1
x. La desigualdad estrictas ocurre cuando x tiene mas de
un dgito. Luego x
2
10x 22 x, y deducimos que x < 13 , por lo
tanto x tiene un solo dgito o x = 10 , x = 11 , x = 12 .
Si x tiene un dgito, entonces a
0
= x
2
10x22 , pero esta ecuacion
no tiene soluciones enteras. Si x = 10 , entonces P(10) = 0 , pero
x
2
10x 22 = 22 = 0 . Si x = 11 , entonces P(11) = 1 , pero
x
2
10x 22 = 11 = 1 .
Por lo tanto x = 12 es la unica solucion. En efecto, 12
2
101222 =
2 y P(12) = 1 2 = 2 .
Ejemplo 6.10 Sea A un n umero entero positivo, y sea A

un n umero
escrito con los mismos dgitos de A arreglados en alg un otro orden.
Pruebe que si A+A

= 10
10
, entonces A es divisible por 10.
Sergio Plaza 229
Solucion. Es claro que A y A

deben tener 10 dgitos. Sean A =


a
10
a
9
. . . a
1
y A

= a

10
a

9
. . . a

1
, donde a

i
es alg un a
j
. Ahora, A+A

=
10
1
0 si y solo si existe un j , con 0 j 9 para el cual a
1
+ a

1
=
a
2
+a

2
= = a
j
+a

i
, a
j+1
+a

i+1
= 10a
j+2
+a

j+2
.
Existe un j , con 0 j 9 para el cual a
1
+ a

1
= a
2
+ a

2
=
= aj + aj

, a
j+1
+ a

j+1
= 10 , a
j+2
+ a

j+2
= a
j+3
+ a

j+3
= =
a
10
+ a

10
= 9 . Observemos que j = 0 implica que no existen suma de
la forma a
j+k
+ a

j+k
, k 2 y j = 0 implica que no existen suma de
la forma a

+a

, 1 j . Adicionando todas esas sumas, obtenemos


a
1
+a

1
+a
2
+a

2
+ +a
10
+a

10
= 9 + 9(9 j).
Como los a

s
son permutaciones de los a
s
, vemos que el lado izquierdo
de la ultima igualdad es el n umero par 2(a
1
+a
2
+ +a
10
) , esto implica
que j debe ser impar. Pero esto implica que a
1
+a

1
= 0 , de lo cual el
resultado se sigue.
Ejemplo 6.11 Pruebe que cada entero positivo tiene un m ultiplo en el
cual aparecen, en su representacion decimal, todos los dgitos 0, 1, . . . , 9 .
Solucion. Sea n un entero positivo arbitrario con k dgitos. Sea
m = 123456789 10
k+1
Se tiene que los n n umeros consecutivos m + 1 , m + 2, . . . , m + n
comienzan con 1234567890 y uno de ellos es divisible por n.
Ejemplo 6.12 Escriba 6312 en base 2.
Solucion. Como 2
12
= 4096 < 6312 < 2
13
= 8192 , se tiene que
(6312)
2
= a
12
2
12
+a
11
2
11
+ +a
1
2 +a
0
.
230 Teora de N umeros
Tenemos 6312 = 2
12
+ 2216 . Ahora, como 2
11
= 2048 < 2216 <
2
12
= 4096 , se sigue que 6312 = 2
12
+ 2
11
+ 168 , y siendo que 2
7
=
128 < 168 < 2
8
= 256 , se tiene que 6312 = 2
12
+ 2
11
+ 2
7
+ 40 .
Ahora, 2
5
= 32 < 40 < 2
6
= 64 , as 6312 = 2
12
+ 2
11
+ 2
7
+ 2
5
+ 8 =
2
12
+ 2
11
+ 2
7
+ 2
5
+ 2
3
.
Ejemplo 6.13
137
256
= 1 2
1
+0 2
2
+0 2
3
+0 2
4
+1 2
5
+0 2
6
+0 2
7
+1 2
8
.
Ejemplo 6.14 Dado x
0
, con 0 x
0
< 1 , sea
x
n
=
_
_
_
2x
n1
si 2x
n1
< 1
2x
n1
1 si 2x
n1
1 .
para todo entero positivo n. Encuentre todos los x
0
tales que x
0
= x
5
(si existe alguno).
Solucion. Escribiendo x
0
en base 2 se tiene
x
0
=

k=1
a
k
2
k
, a
k
= 0 o 1
El algoritmo dado en el enunciado simplemente mueve el punto binario
una unidad a la derecha. Para que x
0
sea igual a x
5
, necesitamos que
0.a
1
a
2
a
3
a
4
a
5
a
6
a
7
= 0.a
6
a
7
a
8
a
9
a
10
a
11
a
12
. Esto ocurre si y solo si x
0
tiene una expansion periodica en base 2 con a
1
a
2
a
3
a
4
a
5
siendo el bloque
del perodo. Existen 2
5
= 32 de tales bloques. Pero si a
1
= a
2
= =
a
5
= 1 , entonces x
0
= 1 y no es posible. Por lo tanto solo quedan
Sergio Plaza 231
2
5
1 = 32 1 = 31 posibilidades, y ellas son faciles de construir. Por
ejemplo
a
1
a
2
a
3
a
4
a
5
1 0 0 0 0
0 1 0 0 0
etc
Ejemplo 6.15 La sucesion 1, 3, 4, 9, 10, 12, . . . consiste de todos los
enteros positivos que son una potencia de 3 o suma de distintas potencias
de 3. Por ejemplo, 12 = 3
2
+ 3 , 10 = 3
2
+ 1 , 4 = 3 + 1 , 1 = 3
0
,
13 = 3
2
+ 3 + 1 . Encuentre el termino 100 de esta sucesion.
Solucion. Si los terminos de la sucesion son escritos en base 3, ellos son
exactamente aquellos n umeros enteros positivos cuya expansion en base
3 no contienen el dgito 2. Luego, la sucesion escrita en base 3 y puesta
en orden creciente es
1, 10, 11, 100, 101, 110, 111, . . .
En base 2, estos n umeros no son otros que 1, 2, 3, 4, 5, 6, 7, 8, . . . .
Luego para obtener el termino 1000 de la sucesion basta escribir 100 en
binario, esto es, 100 = 1 2
6
+ 1 2
5
+ 0 2
4
+ 0 2
3
+ 1 2
2
+ 0 2
1
+
0 2
0
= (1100100)
2
y ponerlo en base 3, lo cual nos da (1100100)
3
=
3
6
+ 3
5
+ 3
2
= 981 .
Observemos que cualquier n umero real x, con 0 x < 1 tiene aso-
ciado una sucesion de ceros y unos. Por ejemplo, a
137
256
se le asocia
k
1
= 1, k
2
= 0, k
3
= 0, k
4
= 0, k
5
= 1, k
6
= 0, k
7
= 0, k
8
= 1, k
j
= 0, j 9 .
232 Teora de N umeros
Adaptando para la sucesion anterior la escritura 10001001, tenemos
que el smbolo 10001001 representa el n umero
137
256
.
As, los smbolos 001 y 01011 representan los n umeros racionales
1
8
= 0 2
1
+ 0 2
2
+ 1 2
3
,
19
32
= 0 2
1
+ 1 2
2
+ 0 2
3
+ 1 2
4
+ 1 2
5
Un n umero irracional debe tener innitos unos en su expresion binaria
(de otra forma representara un n umero racional) y sus ceros y unos care-
cer de toda periodicidad. Es as como los smbolos 01001000100001 . . . ,
110111011110111110 . . . representan n umeros irracionales. El lector puede
intentar calcularlos.
Ejemplo 6.16 Escriba 6312 en base 5.
Solucion. Tenemos que 5
5
= 3125 < 6312 < 5
6
= 15625 . Luego
(6312)
5
= a
5
5
5
+ a
4
5
4
+ a
3
5
3
+ a
2
5
2
+ a
1
5 + a
0
. Tenemos
6312 = 2 5
5
+62 , es decir, a
5
= 2 , a
0
= 62 , y a
1
= a
2
= a
3
= a
4
= 0 .
Ejemplo 6.17 Pruebe que 4.41 es un cuadrado perfecto en cualquier
base p > 1 .
Solucion. Escribiendo 4.41 en la base p tenemos
4 41 = 4 +
4
p
+
1
p
2
=
_
2 +
1
p
_
2
6.4 e es irracional
Teorema 6.2 e es irracional.
Sergio Plaza 233
Demostracion. Supongamos que e =
a
b
con a y b enteros positivos,
y sea
= b|e
b

j=0
b!
j!
=

j=b+1
b!
j!
(6.1)
Entonces
0 < <

j=1
1
(b + 1)
j
=
1
b
1
Por otro lado, la expresion central de (6.1) es un entero. Luego, tene-
mos una contradiccion y e es irracional.
Problema 6.1 Es
e
irracional? abierto.
Problema 6.2 Es

n=1
1
n
5
irracional?. abierto.
Observacion. Un n umero irracional elevado a un n umero irracional
puede ser racional. En efecto, considere x =

2
el cual es irracional.
Pero x

2
=
_

2
_

2
= (

2)
2
= 2 es racional.
6.5 Conjunto de Cantor
Este es uno de los ejemplos mas simples que sirven para ilustrar el
concepto de conjunto fractal. El conjunto de Cantor es obtenido con-
siderando la representacion en base 3 de los n umeros reales x, con
0 x 1 , usando como conjunto de dgitos permitido para la re-
presentacion el conjunto D = {0, 2} , es decir, el conjunto de Cantor
esta formado por todos los n umeros reales x, con 0 x 1 , tales que
en su representacion en base 3 no aparece el dgito 1. Geometricamente,
234 Teora de N umeros
este conjunto se construye como sigue. Consideremos un segmento de
recta de longitud 1. Dividimos el segmento inicial en 3 segmentos de
igual
longitud y eliminamos el segmento central, obteniendo dos segmentos
cada uno de longitud
1
3
. Enseguida dividimos cada segmento resultante
en la etapa anterior en 3 segmentos de igual longitud y eliminamos los
segmentos central, obteniendo 4 segmentos cada uno de longitud
1
9
.
Repetimos el proceso de division y eliminacion anterior a cada segmento
resultante en la etapa anterior, y continuamos el proceso indenida-
mente.
.................................................................................................................................................................................................................................................................................................................................................................................................................................................................................................................................................................................................................................................................
0 1
......................................................................................................................................................................................................................
0 1/3
......................................................................................................................................................................................................................
2/3 1
.......................................................................
0 1/9
.......................................................................
2/9 1/3
.......................................................................
2/3 7/9
.......................................................................
8/9 1
.
.
.
Conjunto de Cantor
El resultado nal es un conjunto C , llamado conjunto de Cantor,
el cual es no vaco y contiene tantos puntos como la recta real. De-
namos la funcion : {x : 0 x 1} C por (

j=1
a
j
2
j
) =

j=1
(2a
j
)3
j
. Es facil vericar que es una biyeccion, por lo tanto
se tiene lo pedido. Lo que acabamos de demostrar no es intuitivo ni facil
de aceptar.
6.5.1 Longitud del Conjunto de Cantor
Si en cada etapa de la construccion del conjunto de Cantor, medimos la
longitud del conjunto resultante, obtenemos la siguiente tabla
Sergio Plaza 235
Etapa Longitud
0 1
1
2
3
2
4
9
=
_
2
3
_
2
3
8
27
=
_
2
3
_
3
.
.
.
.
.
.
Intuitivamente el conjunto de Cantor debera tener longitud 0. Debido
a su construccion el conjunto de Cantor es autosimilar, es decir, cada
parte cuando la ampliamos se ve como el conjunto original.
Notemos que el conjunto de Cantor es de longitud cero y tiene tantos
puntos como el segmento inicial. Esto no es facil de aceptar ni intuitivo.
La construccion anterior del conjunto de Cantor es la clasica. Exis-
ten muchas construcciones de conjuntos de Cantor, es decir, de division
de un segmento en segmentos (no necesariamente en 3) y en propor-
ciones distintas (no necesariamente
1
3
) , y que nos llevan a un conjunto
de Cantor. Incluso se pueden construir conjuntos de Cantor con lon-
gitud positiva. En la actualidad a un se trabaja y se publican trabajos
profundos en matematica que tienen relacion con estos conjuntos.
Porque le dimos el nombre de fractal al conjunto de Cantor? No
solo porque posee la propiedad de ser autosimilar, si no mas bien por el
hecho que el ocupa en la recta mas espacio que un conjunto de puntos
aislados y menos que un segmento de recta Como podemos asegurar
tal hecho? Para esto deniremos el siguiente concepto.
236 Teora de N umeros
6.6 k Volumen
Consideremos un segmento de recta, que por simplicidad, suponemos
unitario (no es ninguna restriccion esta suposicion). Para calcular el
1 volumen del segmento unitario procedemos como sigue. Cubrimos
nuestro segmento por segmentos peque nos de longitud, digamos , ( >
0 un n umero). Denotemos por N() el n umero mnimo de intervalos de
longitud necesario para cubrir el segmento dado. Tenemos entonces
que N()
1
1 , donde el smbolo signica aproximadamente igual,
luego el 1volumen del segmento de recta unitario es igual a 1.
Este concepto se extiende de modo natural al caso dimension mayor
que 1.
Para calcular el 2volumen de un cuadrado, el cual para simplicar
suponemos unitario (es decir, de lado 1) cubrimos este por cuadrados
peque nos de lado ( > 0) . Denotemos por N() el n umero mnimo
de cuadrados de lado , necesarios para cubrir el cuadrado, entonces
N()
2
1 , luego el 2volumen del cuadrado unitario es igual a 2.
Ahora consideremos el conjunto de Cantor. Para calcular su 1
volumen, en cada paso de la construccion calculamos el 1 volumen
de los segmentos que conforman la parte que resta en cada etapa de la
construccion. Para n = 0, si tomamos = 1, tenemos N() = 1 , para
n = 1, si tomamos =
1
3
, tenemos N() = 2 , para n = 2, tomando
=
1
3
2
, tenemos N() = 2
2
, . . . , para n = k tomando =
1
3
k
, tenemos
N() = 2
k
. Y vemos que en la etapa n de la construccion del conjunto
de Cantor, tenemos la siguiente formula para calcular el 1volumen de
lo que resto; v
n
=
_
2
3
_
n
, luego el 1volumen del conjunto de Cantor
es obtenido haciendo crecer n indenidamente en la formula para v
n
,
consecuentemente es igual a cero como presentamos intuitivamente. Por
Sergio Plaza 237
otra parte, la ecuacion
N()
1

_
2
3
_
k
,
usada para calcular el 1volumen del conjunto de Cantor, es reem-
plazada por una ecuacion del tipo
N()
s
= 1
y buscamos el valor de s de modo que esto ocurra. En la etapa k ,
tenemos la ecuacion
2
k
_
1
3
k
_
s
= 1 ,
de esto,
2
k
= 3
ks
de donde, aplicando logaritmo en base e , nos queda k ln(2) = ks ln(3) ,
y por lo tanto obtenemos que s =
ln(2)
ln(3)
. Esto nos dice que el conjunto
de Cantor tiene s volumen igual a
ln(2)
ln(3)
= 0.63092975 . . . lo cual nos
indica que ocupa mas espacio que un conjunto de puntos aislados, pero
menos que un segmento de recta. Este tipo de k volumen es el que
dene el caracter de fractal del conjunto de Cantor, y de otros muchas
guras que son usadas en la literatura para ilustrar este concepto, muy
de moda en el ultimo tiempo.
Nota: La denicion que dimos de conjunto fractal
1
no es enteramente
correcta. Para dar la denicion correcta necesitamos de muchos otros
concepto que escapan muy lejos del objetivo de nuestro texto.
1
Para una introduccion a los Fractales y geometra fractal puede verse S. Plaza
Fractales y Generacion Computacional de Imagenes , monografa 16, IMCA, Per u,
2000. Este texto incluye tambien una peque na historia del desarrollo del concepto de
conjunto fractal.
238 Teora de N umeros
6.6.1 Triangulo de Sierpinski
Este es la segunda gura fractal de entre las mas conocidas en general.
Su construccion geometrica es sencilla y la describimos a seguir.
Consideremos una region triangular en el plano. Primero dividimos
los lados del triangulo frontera en sus puntos medios, uniendolos for-
mamos cuatro regiones triangulares, de las cuales eliminamos la region
triangular central. La gura muestra esta primera etapa.
etapa 1
En las regiones triangulares restantes repetimos el proceso anterior, y
as sucesivamente, como se muestra en las guras siguientes
etapa 2
Sergio Plaza 239
etapa 3 etapa 4
etapa 5 etapa 6
Este proceso es convergente y la gura obtenida es conocida como
triangulo de Sierpinski. El triangulo de Sierpinski tiene longitud innita
y ocupa un area cero en el plano, de hecho ocupa mas espacio que
una curva y menos que un plano. Una aproximacion mas renada al
resultado nal es mostrado en la siguiente gura
Triangulo de Sierpinski
240 Teora de N umeros
Un calculo directo, muestra que en cada etapa de la construccion del
triangulo de Sierpinski, es posible calcular su area y que esta tiende a
cero cuando avanzamos en la etapas de la construccion. Este calculo es
una aplicacion directa de la formula del area de un triangulo, y como
los tres triangulos que aparecen en cada uno de los triangulos de una
etapa para la siguiente son congruentes, basta calcular el area de uno de
ellos y notar que en la etapa 1 tenemos 3 triangulos, en la etapa 2 tene-
mos 3
2
, en la etapa 3 tenemos 3
3
triangulos, y as sucesivamente, en la
etapa n de la construccion tenemos 3
n
triangulos . Ahora, si consider-
emos las sucesivas etapas de la construccion del triangulo de Sierpinski,
vemos que en la etapa 1 necesitamos 3 triangulos para cubrir lo que
resto despues de eliminar el triangulo central, en la etapa 2 necesitamos
3
2
triangulos para cubrir lo que resto despues de eliminar el triangulo
central en cada uno de los 3 triangulos de la etapa 1, y en general, nece-
sitamos 3
n
triangulos para cubrir lo que resto despues de eliminar el
triangulo central en cada uno de los 3
n1
triangulos de la etapa n 1 .
Razonado como lo hicimos en el caso del conjunto de Cantor, vemos que
el s volumen del triangulo de Sierpinski es s =
ln(4)
ln(3)
.
6.6.2 Triangulo de Sierpinski y expansion en base 2
En el plano R
2
consideramos un sistema de coordenadas (u, v) con
origen en (0, 0) donde la recta de las abscisas coincide con el eje x y
la recta de las ordenadas forma un angulo de 60
0
con el eje x. Es facil
ver que las coordenadas en el plano (u, v) con 0 u 1 y 0 v 1
representan un punto en el triangulo de Sierpinski si y solo si la expansion
en base 2 de u y de v nunca tienen un 1 en la misma posicion.
Otra manera interesante de obtener una imagen del triangulo de Sier-
Sergio Plaza 241
pinski, es considerar el triangulo de Pascal, es decir, el triangulo formado
por los coecientes binomiales del desarrollo del binomio (x +y)
n
, con
n = 0, 1, 2, 3, . . . . Enseguida marcamos de color negro cada n umero
impar y marcamos de color blanco cada n umero par. Esto es hecho
asignando a 0 el color blanco y a 1 el color negro, y los n umeros en el
triangulo de Pascal los consideramos modulo 2, es decir, si k N en-
tonces k = 1 (mod 2) si y solo si k es impar y k = 0 (mod 2) si y solo
si k es par. La gura obtenida se ve como el triangulo de Sierpinski.
6.7 Problemas
Problema 6.1 Calcule
1. (1034)
5
+ (243)
5
2. (54302)
6
(21543)
6
3. (1230)
4
(3120)
4
.
Problema 6.2 Determine el valor de b si
1. (104)
b
= 8285
2. (30407)
b
= 12551 .
Problema 6.3 Si b > 1 es un entero. Demuestre que (111)
b
|(10101)
b
.
Problema 6.4 Sea t un n umero real positivo. Pruebe que existe un
entero positivo n tal que la expansion decimal de nt contiene al dgito
7.
Problema 6.5 Para cada entero positivo k , sea f
1
(k) =(suma de los
dgitos de k
2
). Para n 2 , sea f
n
(k) = f
1
(f
n1
(k)) . Encuentre
f
1988
(11) .
242 Teora de N umeros
Captulo 7
Fracciones continuadas
Para hacer matematica, esto es, en orden de entender y hacer contribu-
cione a esta disciplina, es necesario estudiar su historia. Matematica es
constantemente construida sobre descubrimientos pasados.
Veamos por ejemplo un poco de la historia y origen de fracciones
continuadas.
El origen de las fracciones continuadas es difcil de establecer, esto
debido al hecho que podemos encontrar ejemplos de esas fracciones a
traves de la matematica en los ultimos 2000 a nos, pero su verdadero
fundamento nos lleva hasta nes de 1600 e inicio de 1700.
El origen de fracciones continuadas es tradicionalmente puesto en el
tiempo de la creacion del algoritmo de la division de Euclides. El algo-
ritmo de Euclides, es usado para encontrar el maximo com un divisor de
dos n umeros. Por manipulaciones algebraicas del algoritmo, podemos
derivar la fraccion continuada simple de un n umero racional p/q . Es du-
doso si Euclides o sus predecedores de hecho hayan usado este algoritmo
de esa forma. Debido a su cercana relacion a fracciones continuadas,
la creacion del algoritmo de Euclides signico el desarrollo inicial de
243
244 Teora de N umeros
fracciones continuadas.
Por mas de mil a nos, todo libro que usaba fracciones continuadas es-
taba restricto a ejemplos especcos. El matematico ind u Aryabhata
(c.550 A.C.) uso fracciones continuadas para resolver una ecuacion lin-
eal en una indeterminada. Mas que generalizar su metodo, su uso de
fracciones continuadas es solo para ejemplos especcos.
A traves de los escritos matematicos griegos y arabes, encontramos
ejemplos y trazas de fracciones continuadas. Pero otra vez, su uso limi-
tado a ejemplos especcos.
Dos matematicos italianos, de la cuidad de Bologna, Rafael Bombelli
(B.C. 1530) y Prietro Cataldi (15481626) tambien contribuyeron al
estudio de fracciones continuadas proveyendo mas ejemplos. Bombelli
expreso la raz cuadrada de 13 como una fraccion continuada repetida.
Cataldi hizo lo mismo con la raz cuadrada de 18. Aparte de esos ejem-
plos, sin embargo, ning un matematico investigo las propiedades de frac-
ciones continuadas.
Fracciones continuadas se transforman en un campo de estudio por
si mismas a traves del trabajo de Hohn Wallis (1616-1703). En su li-
bro Arithmetica Inmtorum (1655), Wallis el desarrolla y presenta la
identidad
4

=
3 3 5 5 7 7
2 4 4 6 6 8 8
.
El primer presidente de la Royal society, Lord Brounker (16201684)
Sergio Plaza 245
transformo esta identidad en
4

= 1 +
1
2
2 +
3
2
2 +
5
2
2 +
7
2
2 +
9
2
.
.
.
Wallis tomo la iniciativa y comenzo a dar los primeros pasos para
generalizar la teora de fracciones continuadas.
En su libro Opera Mathematica (1645), Wallis hace algunas contribu-
ciones a la parte basica para fracciones continuadas. Explica como cal-
cular la convergente nesima y descubre algunas de las propiedades
familiares de las convergentes. Fue tambien en este trabajo en que el
termino fraccion continuada fue usado por primera vez.
El matematico y astronomo aleman Christiaan Huggens (16291695)
fue el primero en demostrar una aplicacion practica de fracciones con-
tinuadas. Escribio un artculo explicando como usar las convergentes de
una fraccion continuada para encontrar la mejor aproximacion racional
para las razones de engranajes.
Esas aproximaciones le permitieron tomar los engranajes con el n umero
correcto de dientes. Este trabajo fue en parte motivado por su deseo de
construir un planetario mecanico.
Mientras que los trabajos de Wallis y Huggens comenzaron el trabajo
sobre fracciones continuadas, el campo de las fracciones continuadas
comenzo a orecer cuando Leonard Euler (17071783), Johan Heinrich
Lambert (17281777) y Joseph Louis Lagrange (17361813) abarcan ese
topico. Euler contribuyo mucho a la teora moderna en su trabajo De
Fractionlous Continious publicado en (1737). En este trabajo Euler
246 Teora de N umeros
mostro que cada n umero racional puede ser expresado como una fraccion
continuada simple nita. Tambien dio una expresion para e en forma
de una fraccion continuada, y usa esa expresion para mostrar que e y
e
2
son irracionales. Tambien demuestra como pasar una serie a una
representacion en fraccion continuada para la serie, y recprocamente.
Lambert generaliza el trabajo de Euler sobre e y muestra que e
x
y tan(x) son irracionales si x es racional. Lagrange usa fracciones
continuadas para encontrar el valor de races irracionales. Probo tambien
que una raz real de una irracional cuadratica es una fraccion continuada
periodica.
El siglo IXX puede, probablemente, ser descrito como la edad de oro de
las fracciones continuadas. Claude Brezinki escribio en History of Con-
tinued Fractions and Pade Approximations, El siglo diecinueve puede
ser llamado el perodo popular para fracciones continuadas. Como
resultado, fue un crecimiento explosivo dentro dee este campo. La
teora de fracciones continuadas que signicativamente desarrollada, es-
pecialmente lo concerniente a las convergentes. Tambien fueron estudi-
adas fracciones continuadas con variables complejas como terminos. Al-
gunos de los matematicos mas prominentes que hicieron contribuciones
a este campo incluye a Jacobi, Perron, Gauss, Cauchy, and Stieljes. A
comienzo del siglo XX, la disciplina tiene grandes avances a partir del
trabajo inicial de Wallis.
A partir del comienzo siglo XX las fracciones continuadas hacen su
aparicion en otro campo. Por ejemplo, las fracciones continuadas han
sido usadas en algoritmos computacionales para calcular aproximaciones
a n umeros reales, as como tambien para resolver ecuaciones indetermi-
nadas.
A un cuando su desarrollo inicial parece haber tomado mucho tiempo,
Sergio Plaza 247
una vez que este comenzo, el campo y su analisis crecio rapidamente.
A un hoy en da, fracciones continuadas continuan siendo usadas e inves-
tigadas.
7.1 Fracciones Continuadas
Primero veamos las fracciones continuadas se usan para aproximar un
n umero real por n umeros racionales. Comenzemos con el siguiente ejem-
plo,
4
11
=
1
11
4
=
1
2 +
3
4
=
1
2 +
1
4
3
=
1
2 +
1
1 +
1
3
=
1
2 +
1
1 +
1
2 + 1
este desarrollo termina en un n umero nito de pasos. Una expresion
como la anterior se llama fraccion continuada nita.
Denici on 7.1 Una fraccion continuada nita es una expresion de la
forma
a
0
+
1
a
1
+
1
a
2
+
1
.
.
. +
1
a
n1
+
1
a
n
(7.1)
donde a
0
es un n umero entero cualquiera y para i = 1, 2, . . . , n, los a
i
son n umeros enteros positivos. Denotaremos la expresion 7.1 mediante
el smbolo
[a
0
; a
1
, a
2
, . . . , a
n
] o a
0
+ [a
1
, a
2
, . . . , a
n
] .
248 Teora de N umeros
Ejemplo 7.1
[1; 2, 3, 1] = 1 +
1
2 +
1
3 +
1
1
= 1 +
1
2 +
1
4
= 1 +
4
9
=
13
9
Es claro que toda fraccion continuada nita [a
0
; a
1
, a
2
, . . . , a
n
] repre-
senta un n umero racional. Recprocamente, se tiene el siguente resul-
tado.
Teorema 7.1 Todo n umero racional puede ser escrito como una fraccion
continuada nita.
Demostracion. Sea a/b un n umero racional positivo (caso a/b sea
negativo es analogo). Por el algoritmo de la division de Euclides, existen
n umeros enteros positivos a
1
, . . . , a
n
, tales que
a = ba
0
+r
1
, 0 < r
1
< b
b = r
1
a
1
+r
2
, 0 < r
2
< r
1
r
1
= r
2
a
2
+r
3
, 0 < r
3
< r
2
.
.
.
r
n2
= r
n1
a
n1
+r
n
, 0 < r
n
< r
n1
r
n1
= r
n
a
n
.
Esta cadena nita de igualdades se puede escribir de la forma siguiente
a
b
= a
0
+
r
1
b
= a
0
+
1
b
r
1
Sergio Plaza 249
b
r
1
= a
1
+
r
2
r
1
= a
1
+
1
r
1
r
2
r
1
r
2
= a
2
+
r
3
r
2
= a
2
+
1
r
2
r
3
.
.
.
r
n1
r
n
= a
n
Reemplazando las expresiones r
k
/r
k+1
, obtenemos
a
b
= a
0
+
1
a
1
+
1
a
2
+
1
.
.
. +
1
a
n1
+
1
a
n
en otras palabras hemos probado que
a
b
= [a
0
; a
1
, a
2
, . . . , a
n
] = a
0
+ [a
1
, a
2
, . . . , a
n
] .
Ejemplo 7.2 El n umero racional
943
414
, se descompone seg un el al-
gortmo de la division como
943 = 2 414 + 115
414 = 3 115 + 69
115 = 1 69 + 46
69 = 1 46 + 23
46 = 2 23 .
250 Teora de N umeros
Luego su fraccion continuada es
943
414
= 2 +
115
414
= 2 +
1
414
115
= 2 +
1
3 +
69
115
= 2 +
1
3 +
1
115
69
= 2 +
1
3 +
1
1 +
46
69
= 2 +
1
3 +
1
1 +
1
69
46
= 2 +
1
3 +
1
1 +
1
1 +
23
46
Sergio Plaza 251
= 2 +
1
3 +
1
1 +
1
1 +
1
2
= [2; 3, 1, 1, 2] .
Tambien es inmediato el calculo de 414/943 , puesto que
414
943
=
1
943
414
=
1
[2; 3, 1, 1, 2]
= [0; 2, 3, 1, 1, 2].
Esta representacion de un n umero racional como fraccion continuada
no es unica, por ejemplo, si [a
0
; a
1
, . . . , a
n
] es una representacion, con
a
n
2 , entonces la cola de la fraccion continuada es
a
n2
+
1
a
n1
+
1
a
n
la cual puede ser reescrita como
a
n1
+
1
(a
n
1) + 1
= a
n1
+
1
(a
n
1) +
1
1
y por lo tanto tenemos la igualdad de las fracciones continuadas
[a
0
; a
1
, . . . , a
n1
, a
n
1, 1] = [a
0
; a
1
, . . . , a
n
]
si a
n
= 1 , tenemos
a
n1
+
1
1
= a
n1
+ 1
252 Teora de N umeros
y entonces [a
0
; a
1
, . . . , a
n2
, a
n1
+ 1] = [a
0
; a
1
, . . . , a
n
] .
Ahora sea x un n umero real, no es racional, el Teoremaema 1 nos
dice que x no es representable por una fraccion continuada nita. Sin
embargo podemos tratar de aplicar el mismo algortmo anterior de una
manera espacial, para ellos introduciremos los conceptos parte entera y
parte fraccional de un n umero real.
Denicion 7.2 La parte entera, [x] , de un n umero real x es el mayor
n umero entero menor o igual que x. La parte fraccionaria, (x) , de x
es (x) = x [x] .
Observemos que 0 (x) < x [x] 1 . Por otra parte, tenemos que
x [x] = 0 si y solo si x es entero.
Ejemplo 7.3
_
943
414
_
= 2 y
_
943
414
_
=
115
414
.
Ahora sea x un n umero real no nulo. Usando lo anterior podemos
escribir x = [x] + (x [x]) = [x] + (x) . Llamemos r
0
= [x] y r
1
= (x) ,
entonces 0 < r
1
< 1 , y podemos escribir
x = x +r
1
= [x] +
1
1
r
1
,
1
r
1
> 1 .
Aplicamos el proceso anterior a 1/r
1
y podemos escribir
1
r
1
=
_
1
r
1
_
+r
2
, 0 < r
2
< 1
Sergio Plaza 253
donde r
2
= [1/r
1
] 1/r
1
es la parte fraccionaria de r
1
. Continuando
de este modo podemos escribir
x = a
0
+
1
_
1
r
1
_
+
1
_
1
r
2
_
+r
3
= a
0
+
1
_
1
r
1
_
+
1
_
1
r
2
_
+
1
_
1
r
3
_
+r
3
.
.
.
Este proceso termina cuando tenemos alg un r

= 0 , en cuyo caso el
n umero es racional como vimos arriba. Si el proceso no acaba en un
n umero nito de pasos, el n umero en cuestion es irracional.
El algoritmo anterior asocia a cada n umero real x una fraccion con-
tinuada nita
c
k
= [a
0
; a
1
, a
2
, . . . , a
k
]
donde a

= [1/r

] y r

= [1/r
1
] 1/r

.
A continuacion damos algunas proposiciones basicas sobre fracciones
continuadas, su demostracion ver [3] , [4] o [5]??.
Dados un n umero real x y un n umero natural k , consideremos la
fraccion continuada nita, c
k
= [a
0
; a
1
, a
2
, . . . , a
k
] asociada, es decir,
254 Teora de N umeros
c
k
= a
0
+
1
a
1
+
1
a
2
+
1
.
.
. a
k2
+
1
a
k1
+
1
a
k
(7.2)
donde
x = a
0
+
1
a
1
+
1
a
2
+
1
.
.
. a
k2
+
1
a
k1
+
1
a
k
+r
k
(7.3)
Asociado con c
k
, denimos los siguientes n umeros
p
2
= 0 , p
1
= 1 , p
i
= a
i
p
i1
+p
i2
, para i 2 (7.4)
y
q
2
= 1 , q
1
= 0 , q
i
= a
i
q
i1
+q
i2
, para i 2 (7.5)
Se tiene q
0
= 1, q
1
= a
1
q
0
q
0
, . . . , q
i
= a
i
q
i1
+ q
i2
q
i1
, es
decir, 1 = q
0
q
1
q
n
.
Teorema 7.2 Con las notaciones anteriores,
1.- Dados un n umero real r y un n umero entero n 0 , entonces
[a
0
; a
1
. . . , a
n1
, r] =
rp
n1
+p
n2
rq
n1
+q
n2
.
Sergio Plaza 255
2.- Para cada n umero entero n 0 , r
n
=
p
n
q
n
.
3.-
p
n
q
n1
p
n1
q
n
= (1)
n1
, n 1
r
n
r
n1
=
(1)
n1
q
n
q
n1
, n 1
p
n
q
n2
p
n2
q
n
= (1)
n
a
n
, n 2
r
n
r
n2
=
(1)
n
a
n
q
n
q
n2
, n 2
y mcd (p
n
, q
n
) = 1 .
Ahora mostraremos que un n umero real, no racional, x puede ser
aproximado tanto cuanto se desee por n umeros racionales.
Teorema 7.3 Sea x un n umero real arbitrario. Dado un n umero real
> 0 cualesquiera, existe un n umero racional q

tal que |x q

| < .
Demostracion. Sabemos que
x = [a
0
; a
1
, . . . , a
n
, r
n
] =
r
n
p
n1
+p
n2
r
n
q
n1
+q
n2
y
c
n
= [a
0
; a
1
, . . . , a
n
] .
Luego
x c
n1
=
r
n
p
n1
+p
n2
r
n
q
n1
+q
n2

p
n1
q
n1
256 Teora de N umeros
=
p
n1
q
n2
p
n2
q
n1
q
n1
(r
n
q
n1
+q
n2
=
(1)
n1
q
n1
(r
n
q
n1
+q
n2
)
.
Como la sucesion (q
n
)
nN
crece indenidamente y r
n
> 0 , se tiene
que xc
n1
es cada vez mas proxima de cero cuando n crece indenida-
mente. Por otra parte, es facil ver que dado > 0 existe un n umero
natural n
0
tal que
1
n
0
< . Tomamos n
0
sucientemente grande de
modo que |x c
n1
| <
1
n
0
< y eliguiendo q

= c
n1
se tiene lo
pedido.
Las fracciones continuada nitas c
n
, son llamadas las convergentes
de x.
Ejemplo 7.4 Busquemos el desarrollo en fraccion continuada de

2 .
Tenemos

2 = 1 + (

2 1)
= 1 +
1
1 +

2
= 1 +
1
2 +
1
1 +

2
= 1 +
1
2 +
1
2 +
1
1 +

2
.
.
.
Sergio Plaza 257
Por lo tanto

2 = [1; 2, 2, 2, 2, 2, 2, . . .] . Esta fraccion continuada es


periodica innita. Como ejercicio el lector puede desarrollar las aproxi-
maciones c
k
, k = 1, 2, 3, . . . para

2 .
Ejemplo 7.5 Consideremos la ecuacion cuadratica x
2
= ax+1 , donde
a es un n umero entero positivo. Tenemos
x = a +
1
x
= a +
1
a +
1
x
= a +
1
a +
1
a +
1
a +
1
x
.
.
.
luego x = [a; a, a, a, . . .] . Para a = 1 se tiene x =
1 +

5
2
, y por lo
tanto
1 +

5
2
= [1; 1, 1, 1, 1, . . .] . Ahora, si escribimos las convergentes
de la fraccion continuada de
1 +

5
2
, tenemos
1, 1 +
1
1
=
2
1
, 1 +
1
1 +
1
1
=
3
2
, 1 +
1
1 +
1
1 +
1
1
=
5
3
,
los siguientes terminos de esta sucesion son 8/5, 13/8, 21/13, . . . .
Observemos que los denominadores de la sucesion anterior vienen dados
258 Teora de N umeros
por la sucesion
1, 1, 2, 3, 5, 8, 13, 21, . . .
es decir, vienen dados por la sucesion
f
0
= 1, f
1
= 1 y f
n+1
= f
n
+f
n1
, n 2 ,
la cual no es otra que la sucesion de sucesion de Fibonacci
7.2 Aplicacion de Gauss
Al parecer Gauss fue el primero en preocuparse del estudio de la parte
fraccional en el desarrollo de un n umero en fraccion continuada, para
ello considero la aplicacion G : [0, 1[ [0, 1[ denida por
G(x) =
_

_
0 si x = 0 ,
1
x
(mod 1), si x = 0 .
Graco de G.
Con esta aplicacion, el algoritmo para encontrar la fraccion continuada
de un n umero real x , se escribe como sigue
r
k+1
=
_
1
r
k
_
= G(r
k
)
Sergio Plaza 259
a
k+1
=
_
1
r
k
_
, k = 0, 1, 2, . . . .
Ahora si x
0
= [ a
1
, a
2
, a
3
, a
4
, . . . ] [0, 1[ , entonces G(x
0
) = x
1
=
[ a
2
, a
3
, a
4
, . . . ] , G(x
1
) = x
2
= [ a
3
, a
4
, . . . ] y as sucesivamente. De-
notemos por G
k
la composicon de G consigo misma k veces, es decir,
G
k
= G G G, k veces. Decimos que un punto x
0
es periodico
de perodo k 1 para G, si G
k
(x
0
) = x
0
, cuando k = 1 deci-
mos que x
0
es un punto jo de G. Por ejemplo, x
0
= [ 1, 2, 1, 2, . . . ]
es periodico de perodo 2, pues x
1
= G(x
0
) = [ 2, 1, 2, 1, . . . ] , x
2
=
G(x
1
) = [ 1, 2, 1, 2, . . . ] , esto es, G G(x
0
) = x
0
.
Es facil ver que existen innitos puntos periodicos para G, para ello
basta tomar un bloque de longitud m de enteros positivos k
1
, k
2
, . . . , k
m
y formar el n umero x
0
= [ k
1
, k
2
, . . . , k
m
, k
1
, k
2
, . . . , k
m
, . . . ] . Un calculo
sencillo muestra que si x
0
es un punto periodico de G entonces es
solucion de una ecuacion cuadratica con coecientes enteros.
Mas general, se tiene
Teorema 7.4 (Galois) Un n umero real x tiene una fraccion continua-
da periodica, incluyendo el primer entero n
0
si y solo si x es solucion
de una ecuacion cuadratica con coecientes enteros. Ademas, la otra
raz pertenece al intervalo ] 1, 0[ .
Ejemplo 7.6 La razon a urea, =
1 +

5
2
, es solucion de la ecuacion

2
1 = 0 y como vimos arriba su fraccion continuada es =
1 + [ 1, 1, 1, 1, . . . ] y la de 1/ es 1/ = [ 1, 1, 1, 1, 1 . . . ] .
Decimos que x
0
es nalmente periodico, si para alg un entero positivo
k 2 , se tiene que x
k
= G
k
(x
0
) es periodico para G, por lo visto
260 Teora de N umeros
arriba su fraccion continuada tiene la forma
x
0
= [ a
1
, a
2
, . . . , a
k
, n
1
, . . . , n

, n
1
, . . . , n

, . . . ].
Estos punto tienen una caracterizacion sencilla dada por el siguiente
Teorema 7.5 (Lagrange) Un punto x
0
es nalmente periodico para
G si y solo si es solucion de una ecuacion cuadratica con coecientes
enteros.
Cuando x no es solucion de ninguna ecuacion cuadratica con coe-
cientes enteros, su fraccion continuada puede ser muy irregular, ver por
ejemplo el artculo de Carlos Tamm en este mismo n umero.?
7.3 Aproximaciones racionales por fracciones
continuadas
Probaremos primero que los denominadores de las convergentes de frac-
ciones continuadas crecen.
Lema 7.1 Sean a
0
, a
1
, . . . una sucesion de enteros, con a
k
> 0 para
todo k 1 . Denamos
p
0
= a
0
, q
0
= 1
p
1
= a
1
a
0
+ 1, q
1
= a
1
p
k
= a
k
p
k1
+p
k2
, q
k
= a
k
q
k1
+q
k2
, k 2 .
Entonces q
k+1
> q
k
para todo k > 0 .
Sergio Plaza 261
Demostracion. Sea k > 0 . Note que q
k1
es un entero positivo.
Luego,
q
k+1
= a
k+1
q
k
+q
k1
> a
k+1
q
k
1 q
k
= q
k
,
pues a
k+1
1 , ya que a
j
son todos enteros positivos para todo j 1 .
Las convergentes de una fraccion continuada oscilan alrededor del
valor lmite, y las convergentes son siempre fracciones en terminos menores.
De hecho, las convergentes con la mejor aproximacion racional al valor
de la fraccion continuada. Establecemos esto en forma mas precisa.
Teorema 7.6 Sea x un n umero irracional, y sea c
k
=
p
k
q
k
la kesima
convergente en la expansion de fraccion continuada de x. Supongamos
que p, q Z, con q > 0 , y
|q
x
p| < |q
k
x p
k
| .
Entonces q q
k+1
.
Antes de demostrar este teorema, veamos su signicado geometrico.
En el plano (t, y) dibujemos la recta de pendiente x a traves del origen.
Marquemos los puntos (p, q) y (p
k
, q
k
) . La hipotesis |qxp| < |q
k
xp
k
|
nos dice que las distancias verticales desde (q, p) a y = tx es menor
que la distancia vertical desde (q
k
, p
k
) para y = xt .
La conclusion dice que q > q
k+1
. De hecho, como q
k+1
> q
k
, q >
q
k
?? . El denominador de p/q es mayor que el denominador de
p
k
/q
k
. En otras palabras, la unica manera que el punto (p, q) sea mas
proximo a la linea y = xt es que su y coordenada es mayor. Podemos
reformular el teorema en la forma de un corolario en el cual podemos
ver las fracciones en cuestion aproximando a x.
262 Teora de N umeros
Corolario 7.1 Sea x un n umero irracional, y sea c
k
=
p
k
q
k
la kesima
convergente en la expansion en fraccion continuada de x . Supongamos
que p, q Z, q > 0 , y

x
p
q

<

x
p
k
q
k

.
Entonces q > q
k
.
Demostracion. Supongamos lo contrario, es decir, q < q
k
. Ahora,
como

x
p
q

<

x
p
k
q
k

,
multiplicando esas dos desigualdades obtenemos
|qx p| < |q
k
x p
k
| .
Del teorema 7.6 obtenemos q q
k+1
, de donde q
k
q q
k+1
, lo
cual contradice el hecho que los q
j
son crecientes. Por lo tanto, q > q
k
.
Este resultado dice que la unica manera que un n umero racional
p
q
puede a una fraccion continuada mejor que una convergente
p
k
q
k
es si la
fraccion tiene mayor denominador que el de la convergente.
Ejemplo 7.7 En la tabla siguiente se dan las convergentes de la ex-
pansion en fraccion continuada de .
Sergio Plaza 263
a
k
p
k
p
k
c
k
7 3 1 3
7 22 7
22
7
15 333 106
333
106
1 355 113
35
113
292 103993 33102
103993
33102
Ahora,
103993
33102
= 3.1415926530119026040 y vemos que

103993
33102

0.57789063 10
9
.
El teorema 7.6 dice que una fracci on
p
q
puede ser mayor aproximacion
a que
103993
33102
solo si q > 33102 .
El proximo resultado es una especie de recproco a los dos resultados
anteriores.

Este dice que si un n umero racional aproxima a un n umero
irracional x sucientemente bien, entonces el n umero racional debe
ser una convergente en la expansion en fraccion continuada de x.
Teorema 7.7 Sea x un n umero irracional, y sea
p
q
un n umero racional,
con mcd(p, q) = 1 , y q > 0 . Supongamos que

x
p
q

<
1
2q
2
.
Entonces
p
q
es una convergente en la expansion en fraccion continuada
para x .
Demostracion. Como q
k
k para k 0 , la sucesion (q
n
)
nN
es una
sucesion de enteros positivos estrictamente creciente.
264 Teora de N umeros
Por lo tanto para alg un k , se tiene que q
k
q q
k+1
. Como
q < q
k+1
, del teorema anterior obtenemos
|q
k
x p
k
| |q
x
p| = q

x
p
q

< q
1
2q
2
=
1
2q
.
Luego

x
p
k
q
k

<
1
2qq
k
.
Supongamos que
p
q
no es una convergente en la expansion en fraccion
continuada para x. En particular,
p
q
=
p
k
q
k
, luego qp
k
= pq
k
y |qp
k

pq
k
| es un entero positivo.
Como |qp
k
pq
k
| 1 , se tiene
1
qq
k

|qp
k
pq
k
|
qq
k
=

p
k
q
k

p
q

p
k
q
k
x +x
p
q

p
k
q
k
x

x
p
q

<
1
2qq
k
+
1
2q
2
de donde
1
qq
k
<
1
2qq
k
+
1
2q
2
, restando
1
2qq
k
a ambos lados de esta
desigualdad obtenemos
1
2qq
k
<
1
2q
2
,
luego q < q
k
. Pero hemos supuesto que q
k
q . Esto es una con-
tradiccion.
Sergio Plaza 265
Por lo tanto,
p
q
es una convergente en la expansion en fraccion con-
tinuada para x .
Ejemplo 7.8 La fraccion
355
113
es la mejor aproximacion a por una
fraccion que tiene denominador menor que 1000.
En efecto, supongamos que
p
q
es una fraccion irreducible con q <
1000 . Supongamos ademas que


p
q


355
113

.
En otras palabras, supongamos que
p
q
es al menos una buena aproxi-
macion como
355
113
a . Como q < 1000 , se tiene 2q
2
< 2000000 , luego
1
2q
2
>
1
2000000
= 5 10
7
. Pero


355
113

2.6676418 10
7
.
Luego
1
2q
2
> 5 10
7
>


355
113

>


p
q

.
Las hipotesis del teorema son satisfechas, luego p/q debe ser una
convergente en la expansion en fraccion continuada para x. Por lo que
supusimos, ella aproxima al menos tan bien como
355
113
. Por otra
parte, las otras convergentes 3 ,
22
7
,
333
106
con denominadores menores
que 1000 son aproximaciones a mas pobres que la aproximacion
355
113
.
Luego la unica posibilidad es
p
q
=
355
113
.
266 Teora de N umeros
7.4 Fracciones Continuas y Geometra
Como una aplicacion de las fracciones continuas, vamos a probar que

2 es irracional. Consideremos la gura siguiente,


en la gura, AD y AE son segmentos de una secante a traves del cculo
con centro en C . AB es tangente al arco con centro en C . Por lo tanto
tenemos AB
2
= AE AD de donde
AB
AD
=
AE
AB
,
BC = AB ,
AE = AD +DE = AD + 2BC ,
De estas igualdades se obtiene que
BC
AD
=
AB
AD
=
AE
AB
=
AD + 2BC
AB
=
AD + 2AB
AB
= 2 +
AD
AB
Ahora dado un cuadrado unitario y arcos de crculos como en la gura,
tenemos, AC
2
= AB
2
+ BC
2
= 1
2
+ 1
2
= 2. Por lo tanto, AC =

2 .
Sergio Plaza 267
Luego,
AC
BC
=

2
1
por lo tanto,

2 =
AC
BC
=
CD +AD
BC
= 1 +
AD
BC
= 1 +
1
BC
AD
= 1 +
1
2 +
AD
AB
= 1 +
1
2 +
1
AB
AD
= 1 +
1
2 +
1
2 +
AD
AB
repitiendo el proceso anterior con AD/AB, se ve que la fraccion con-
tinua anterior no termina en un n umero nito de paso, por lo tanto, el
n umero

2 no puede ser racional.


7.5 Ecuaciones de Pell
Observemos que si P es un n umero entero que no sea un cuadrado
perfecto entonces

P = [a
1
; a
2
, . . . , a
n
, 2a
1
, a
2
, . . . , 2a
1
, a
2
, . . .]
para alg un n umero natural n.
Una ecuacion de Pell es una ecuacion cuadratica de la forma
x
2
Py
2
= 1 (7.6)
donde P es un n umero natural. Ya estudiamos ecuaciones lineales de
este tipo (ecuaciones Diofanticas).
Proposicion 7.1 Existen soluciones enteras x e y para la ecuacion
de Pell.
268 Teora de N umeros
Demostracion. Tenemos que

P = [a
1
; a
2
, . . . , a
n
, 2a
1
, a
2
, . . .],
para alg un n umero natural n, es decir,

P = a
1
+
1
a
2
+
1
a
3
+
1
.
.
. a
n1
+
1
a
n
+
1
a
1
+a
1
+
1
a
2
+
1
.
.
.
= a
1
+
1
a
2
+
1
a
3
+
1
.
.
. a
n1
+
1
a
n
+
1
a
1
+

P
escribiendo, a
n+1
= a
1
+

P , se tiene
c
n
= r
n+1
/s
n+1
=

P =
a
n+1
r
n
+r
n1
a
n+1
s
n
+s
n1
reemplazando el valor de a
n+1
= a
1
+

P en esta ultima ecuacion


obtenemos,

P =
(a
1
+

P)r
n
+r
n1
(a
1
+

P)s
n
+s
n1
esto es,

P
_
(a
1
+

P)s
n
+s
n1
_
= (a
1
+

P)r
n
+r
n1
Sergio Plaza 269
s
n

P(a
1
+

P) +s
n1

P = a
1
r
n
+r
n

P +r
n1
a
1
s
n

P +s
n

P +s
n1

P = a
1
r
n
+r
n1
+r
n

P
s
n
P + (a
1
s
n
+s
n1
)

P = a
1
r
n
+r
n1
+r
n

P (7.7)
como

P es irracional, de la ecuacion (3.14) se tienen las ecuaciones


s
n
P = a
1
r
n
+r
n1
r
n
= a
1
s
n
+s
n1
.
De estas ultimas se derivan las siguientes ecuaciones para r
n1
y s
n1
r
n1
= s
n
P a
1
r
n
(7.8)
s
n1
= r
n
a
1
s
n
. (7.9)
Por otra parte, sabemos que r
n
s
n1
s
n
r
n1
= (1)
n
y reemplazando
los valores de r
n1
y s
n1
dados por las ecuaciones (3.15) y (3.16) en
esta ultima igualdad se tiene que
r
n
(r
n
a
1
s
n
) s
n
(s
n
P a
1
r
n
) = (1)
n
r
2
n
a
1
r
n
s
n
s
2
n
P +a
1
s
n
r
n
= (1)
n
r
2
n
Ps
2
n
= (1)
n
es decir, u = r
n
y v = s
n
son soluciones de la ecuacion de la ecuacion
x
2
Py
2
= (1)
n
. Por lo anterior, para encontrar soluciones enteras
de la ecuacion x
2
Py
2
= (1)
n
, buscamos la expansion en fraccion
continua de

P y formamos la tabla de las convergentes. Si n es el


n umero de termino en la expansion en fraccion continua de

P antes
que vuelva a aparecer por primera vez el termino 2a
1
, llamando x e y
al numerador y denominador de la n-esima convergente de la expansion
en fraccion continua de

P , se tiene que ellos son n umeros enteros que


satifacen la ecuacion x
2
Py
2
= (1)
n
. Por lo tanto, de lo anterior
270 Teora de N umeros
tenemos que si n es par, entonces x e y son soluciones de la ecuacion
(3.13).
Ademas si n es impar, tomamos x = r
2n
e y = s
2n
en la expansion
anterior y tenemos que ellos son soluciones de la ecuacion de Pell, pues
para todo n N , el n umero 2n es par y (r
2n
)
2
P(s
2n
)
2
= (1)
2n
= 1 .
Captulo 8
Problemas resueltos
Problema 8.1 Pruebe que la ecuacion
x
3
+ 1991y
3
= z
4
tiene innitas soluciones.
Solucion.
de lo cual se obtiene que x = 1992.
Entonces el tro x
0
= 1992 , y
0
= 1992 , z
0
= 1992 es una solucion
de la ecuacion.
Claramente, si k es un n umero entero positivo y u
0
, v
0
, w
0
, es
un tro solucion de la ecuacion, entonces el tro k
4
u
0
, k
4
v
0
, k
3
w
0
es
tambien una solucion. Como k es arbitrario, se ha encontrado una
innidad de ellas.
Problema 8.2 Sean p y q enteros positivos. Demuestre que 2
p
+1 =
q
2
implica que p = q = 3 .
271
272 Teora de N umeros
Solucion. Notemos que encontrando q se obtiene inmediatamente p
y que la igualdad puede ser escrita como 2
p
= (q 1)(q + 1) . Esto
signica que ( q 1) divide a 2
p
. Aplicando el Teorema Fundamental
de la Aritmetica (R.F.A) se obtiene que necesariamente ( q 1) es una
potencia de 2. En resumen, se tiene que (q 1) = 2
n
, con n p , y por
lo tanto la igualdad se transforma en
2
p
= 2
n
(2
n
+ 2) = 2
n
2(2
n1
+ 1) = 2
n+1
(2
n1
+ 1).
De esta igualdad se deduce que (2
n1
+1) debe ser una potencia de 2
(por T.F.A), y esto sucede su y solamente si n = 1 . Por lo tanto q = 3
y p = 3.
Problema 8.3 Sea n un n umero entero mayor que 1. Demuestre que
4
n
+n
4
no es primo.
Solucion. Si n es par la expresion z = 4
n
+n
4
es divisible por 2, luego
no es primo. Supongamos que n = 2k + 1 , con k un entero k 1 .
Entonces z = (2
2k+1
)
2
+ (n
2
)
2
. Sumando 2n
2
2
2k+1
se completa el
cuadrado del binomio, es decir,
z + 2n
2
2
2k+1
= (2
2k+1
+n
2
)
2
.
Despejando z en esta igualdad se obtiene que
z =
_
2
2k+1
+n
2
_
2
2
2(k+1)
n
2
=
_
2
2k+1
+n
2
_
2

_
2
k+1
n
_
2
=
_
2
2k+1
+n
2
n
2
2
k+1
n
_

_
2
2k+1
+n
2
+ 2
k+1
n
_
Sergio Plaza 273
Para nalizar basta con probar que las expresiones entre parentesis
de la derecha en la ultima igualdad son mayores que 1. Claramente
la segunda expresion es mayor que uno. Examinaremos la primera de
ellas. Supongamos que 2
2k+1
+ n
2
2
k+1
n = 1 . Entonces se obtiene
que
_
n 2
k
_
2
+ 2
2k
= 1 , lo cual se cumple solamente si k = 0 , lo cual,
a su vez, no esta permitido por hip otesis.
Problema 8.4 Determine todos los n umeros enteros positivos que son
soluciones de la ecuacion x
3
y
3
= 602 .
Solucion. Algebraicamente se tiene la descomposicion
x
3
y
3
= (x y)(x
2
+xy +y
2
) .
Ademas x y < x
2
+xy +y
2
si x, y son enteros positivos. Notemos
que la descomposicion primaria de 602 = 2 7 43 . Se debe resolver,
entonces,
x y = A
x
2
+xy +y
2
= B,
donde A < B. Experimentando con los pares (A, B) posible (1, 602) ,
(2, 301) , (7, 86) y (14, 43) , se obtiene que el unico que produce solu-
ciones enteras es (2, 301) , a saber x = 1 e y = 9 .
Problema 8.5 El producto de ciertos tres n umeros pares consecutivos
esta dado por 88xxxxx2 , donde cada x representa un dgito. Determine
los dgitos faltantes.
274 Teora de N umeros
Solucion. Se tiene que 88 10
6
< (n 2)n(n + 2) = n
3
4n < n
3
.
Como 440
3
< 88 10
6
< 450
3
= 91.125.000 , el n umero n es superior o
igual a 442 .
Tres n umeros pares consecutivos terminan en una de las cinco formas
posibles: 0, 2, 4; o 2, 4, 6; o 6, 8, 0; u 8, 0, 2; y la unica forma en
que el ultimo dgito del producto sea 2 es 4, 6, 8. Resulta entonces que
los n umeros son 444, 446, 448,cuyo producto es 88714752. Los dgitos
faltantes son, por tanto, 7, 1, 4, 7 y 5.
Problema 8.6 Determinar para cuales n umeros primos p se cumple
que 2
p
+p
2
es primo.
Solucion. Notemos que p = 2 y p = 3 producen los n umeros 8 y 17,
compuesto en el primer caso y primo en el segundo. Basta considerar
entonces primos p > 2 .
Consideremos congruencia modulo 3. Sabemos que p debe satisfacer
una y solo una de las congruencias siguientes
p 0 (mod 3), p 1 (mod 3), p 1 (mod 3) .
Claramente, el primer caso solo se puede dar si p = 3 , puesto que de
otra manera p sera un n umero compuesto.
Aplicando las propiedades de congruencia en cualquiera de los dos
casos restantes se obtiene que p
2
1 (mod 3) .
Por otro lado, como 2 1 (mod 3) se obtiene 2
p
(1)
p
(mod 3) .
Ademas p es impar, luego 2
p
1 (mod 3).
En resumen, 2
p
1 (mod 3) y p
2
1 (mod 3) . Aplicando las
propiedades de las congruencias con respecto a la suma se obtiene nal-
mente que
Sergio Plaza 275
_
2
p
+p
2
_
0 (mod 3) .
Pero entonces
_
2
p
+p
2
_
es siempre divisible por 3 si p > 3 . Luego
el unico caso es p = 3 .
Problema 8.7 Determinar todos los enteros positivos n para los cuales
la expresion 2
n
+ 1 es divisible por 3.
Solucion. Consideremos congruencia modulo 3. Entonces, 2 1 (mod 3)
implica que 2
n
(1)
n
(mod 3) , luego 2
n
+ 1 [(1)
n
+ 1] (mod 3)
Luego, si n es impar, 2
n
+ 1 0 (mod 3) , es decir, 2
n
+ 1 es
divisible por 3 para todo n impar. Ademas, si n es par se obtiene que
2
n
+ 1 2 (mod 3) . Luego 2
n
+ 1 no es nunca divisible por 3 si n es
par.
Problema 8.8 Sean a y b n umeros naturales tales que su maximo
com un divisor es d . Probar que hay exactamente d n umeros del con-
junto {a, 2a, 3a, . . . , (b 1)a, ba} que son divisibles por b .
Solucion. Si d = mcd (a, b) entonces d|a y d|b , es decir, existen
enteros r, s tales que a = rd y b = sd , con mcd (r, s) = 1 . Luego el
conjunto en cuestion puede describirse como sigue
{rd, 2rd, 3rd, , (b 1)rd, brd} = {krd : k = 1, 2, , b} .
Al dividir cada n umero del conjunto por b = sd , se obtiene resto
cero si y solamente si s divide a k (notar que mcd (r, s) = 1 ). Como
b = sd , esto sucede exactamente d veces.
276 Teora de N umeros
Problema 8.9 Una sucesion de n umeros a
1
, a
2
, a
3
, . . . es formada de
acuerdo a la siguiente regla a
1
= 19 , a
2
= 77 , y
a
n
=
1 a
n1
a
n2
, para n > 2 .
Calcule el termino a
1992
de esta sucesion.
Solucion. Como a
1
= 0 y a
2
= 0 , se tiene que a
1
+a
2
= 1 . Calculemos
los primos terminos de la sucesion
a
3
=
1 a
2
a
1
a
4
=
1 a
3
a
2
=
(a
1
+a
2
1)
a
1
a
2
a
5
=
1 a
1
a
2
a
6
= a
1
, a
7
= a
2
.
.
.
Como cada termino solo depende de los dos inmediatamente anteri-
ores, se sigue que ella se repite en ciclos de cinco terminos. Ahora, como
1992 2 (mod 5) , se tiene que a
1992
= a
2
= 77 .
Problema 8.10 Pruebe que, para todo n umero natural n 2 ,
1 +
1
2
+
1
3
+ +
1
n
no es n umero entero.
Solucion. Denotemos por A(n) el conjunto de los primeros n n umeros
naturales, es decir,
Sergio Plaza 277
A(n) = {1, 2, 3, . . . , n}
Podemos suponer que n es un n umero que se encuentra entre 2

y
2
+1
para alg un > 1 , es decir, n = 2

+k , con 0 k < 2

. Luego los
n umeros de la forma 2
m
, con m 1 , estan en dicho conjunto, y ademas
ellos son divisores de un n umeros z si y solo si en la descomposicion
primaria de z aparecer 2

.
Para sumar la expresion pedida se necesita calcular el mnimo com un
m ultiplo de los elementos del conjunto A(n) . Este mnimo com un
m ultiplo es el menor entero divisible por todo elemento de A(n) , luego,
por el T.F.A., el debe tener la forma 2
e
llb , con b impar.
Sumando obtenemos que
1 +
1
2
+
1
3
+ +
1
n
=
a
2

b
Basta con probar que a no es divisible por 2. Reescribiendo esta
igualdad y multiplicandola por 2

b , se tiene que
2

b +
2

b
2
+
2

b
3
+ +
2

b
2

+
2

b
2

+ 1
+
2

b
2

+ 2
+ +
2

b
2

+k
= a .
Puesto que 2

b es divisible por todos los n umeros de A(n) , todas


las fracciones en el lado izquierdo de la ultima igualdad son n umeros
enteros. Ademas, son n umeros pares, pues la maxima potencia de 2 que
puede aparecer en la descomposicion primaria de cualquier n umero del
conjunto A(n) (que son los denominadores de tales fracciones) es .
Luego a debe ser impar.
278 Teora de N umeros
8.1 Problemas Propuestos
1. Determine todas las soluciones enteras de la ecuacion
x
2
+ 15
a
= 2
b
.
con a, b n umeros enteros.
2. Demuestre que la fraccion
21n + 4
14n + 3
es irreducible para todo n umero natural n.
3. Sean x, y, z enteros tales que x
3
+y
3
z
3
es m ultiplo de 7. Pruebe
que uno de esos n umeros es m ultiplo de 7
4. Determine el n umero de cuadrados perfectos que hay entre 40.000
y 640.000 que son m ultiplos simultaneamente de 3, 4 y 5.
5. Un entero de la forma 4
a
(8b+7) , con a y b enteros no negativos,
no puede ser una suma de tres cuadrados.
6. Encontrar todos los n umeros enteros positivos n para los cuales
2
n
1 es divisible por 7.
7. Demostrar que no existen n umeros enteros positivos n para los
cuales 2
n
+ 1 es divisible por 7
8. Sea k un entero positivo tal que
k(k+1)
3
es un cuadrado perfecto.
Demuestre que
k
3
y (k + 1) son cuadrados perfectos.
Captulo 9
Problemas Clasicos No
Resueltos
Problema 9.1 Existen n umeros perfectos impares?
Problema 9.2 Sea f(n) = (n) n. Dena la sucesion f
0
(n) = n y
f
k+1
(n) = f(f
k
(n)) para k = 1, 2, . . . permanece esta sucesion acotada
para cada n? (Poulet).
Problema 9.3 x
n
1
+ x
n
2
+ + x
n
n1
= x
n
n
soluble para cada n > 2 ?
(Euler)
Problema 9.4 Es
e
irracional?
Problema 9.5 Para que valores de k es x
2
+k = y
3
?
Problema 9.6
Problema 9.7
Problema 9.8
279
280 Teora de N umeros
Problema 9.9
Problema 9.10
Problema 9.11
Problema 9.12
Problema 9.13
Referencias
[1] O. Barriga, V. Cortes, S. Plaza, G. Riera. Matematicas y Olimpadas.
Soc. de Matematica de Chile, 1994.
[2] R. M. Corless, G. W. Frank, J. G. Monroe. Chaos and Continued
Fractions. Physica D 46 (1990), 241253.
[3] A. Y. Khintchin. Continued Fractions. Noordho, Groningen, 1963.
[4] G. H. Hardy, E. M. Wright. An Introduction to the Theory of Num-
bers. Oxford Univ. Press, 1971.
[5] W. B. Jones, W. J. Thron. Continued Fractions: Analytic Theory
and Applications. AddisonWesley, Reading, 1980.
281

Você também pode gostar